You are on page 1of 90

CRIMINAL LAW SUMMARY

powerfully expressive, concern to make express what we think is good and just
principles find expression in the law through outcomes, enforcement, philosophical
objectives
o Substantive v. Procedural
AR + MR (Defences) = Criminal Law (Substantive CL)
how do you get to trial, police (Criminal Procedure)

I. SENTENCING
Aims and Purposes of Sentencing
the remedies in criminal law is sentencing; 1995/6 made revisions, putting in
objectives as a general critique, not much guidance
CRIMINAL CODE OF CANADA
718
The fundamental purpose of sentencing is to contribute, along with crime prevention initiatives,
to respect for the law and the maintenance of a just, peaceful and safe society by imposing just
sanctions that have one or more of the following objectives:
(a) to denounce unlawful conduct;
(b) to deter the offender and other persons from committing offences;
(c) to separate offenders from society, where necessary;
(d) to assist in rehabilitating offenders;
(e) to provide reparations for harm done to victims or to the community; and
(f) to promote a sense of responsibility in offenders, and acknowledgment of the harm
done to victims and to the community.
718.0 When a court imposes a sentence for an offence that involved the abuse of a person under the
1
age of eighteen years, it shall give primary consideration to the objectives of denunciation
and deterrence of such conduct.
718.0 When a court imposes a sentence for an offence under subsection 270(1), section 270.01 or
2
270.02 (assault of peace officer) or paragraph 423.1(1)(b) (intimidation), the court shall give
primary consideration to the objectives of denunciation and deterrence of the conduct that
forms the basis of the offence.
718.1 Fundamental Principle of Proportionality - A sentence must be proportionate to the
gravity of the offence and the degree of responsibility of the offender.
718.2 A court that imposes a sentence shall also take into consideration the following principles:
(a) a sentence should be increased or reduced to account for any relevant aggravating
or mitigating circumstances relating to the offence or the offender, and, without
limiting the generality of the foregoing,
o (i) evidence that the offence was motivated by bias, prejudice or hate based on
race, national or ethnic origin, language, colour, religion, sex, age, mental or
physical disability, sexual orientation, or any other similar factor,
o (ii) evidence that the offender, in committing the offence, abused the offenders
spouse or common-law partner,
o (ii.1) evidence that the offender, in committing the offence, abused a person
under the age of eighteen years,
o (iii) evidence that the offender, in committing the offence, abused a position of
trust or authority in relation to the victim,
o (iv) evidence that the offence was committed for the benefit of, at the direction of
or in association with a criminal organization, or
o (v) evidence that the offence was a terrorism offence
shall be deemed to be aggravating circumstances;
(b) a sentence should be similar to sentences imposed on similar offenders for similar
offences committed in similar circumstances;
(c) where consecutive sentences are imposed, the combined sentence should not be

unduly long or harsh;


(d) an offender should not be deprived of liberty, if less restrictive sanctions may be
appropriate in the circumstances; and
(e) all available sanctions other than imprisonment that are reasonable in the
circumstances should be considered for all offenders, with particular attention to the
circumstances of aboriginal offenders.
o NOTES: reasons of opportunity, poverty, abuse, historical injustice, systemic bias,
etc.; SCC remedial (reduce incarceration); reference question from Parliament,
otherwise have to wait for case to come to court; if just, peaceful and safe society
(s. 718), then have to consider systemic
Objectives are not prioritized and therefore judges have broad discretion to det.
Sentences
Doubt about efficacy of some objectives, esp deterrence and rehabilitation
Problems
in
CCC gives no guidance to judges about whether a custodial or non-cust sentence is
Sentencin
appropriate and if yes, what length; no avg starting points given
g
Existing sentencing laws say that ppl dont serve full sentence given by judge; elgible
for parole after 1/3 and usually released after 2/3; actual length det by subsequent
executive decision mkrs
1. Absolute and conditional discharge
2. Probation
3. Fine
4. Restitution and/or community work
Sentencin
5. Intermittent Sentence of Imprisonment (eg. 90 days or < served on wknds)
g Options
6. Conditional Sentence of Imprisonment (served in community under conditions e.g.
house arrest)
7. Imprisonment
8. Specific Prohibitions
9. Long term offender designation and dangerous offender designation
Aboriginal
Account for 2% of pop, 14% in fed penitentiaries
Offenders
S. 718.2(e) meant to respond to overrep of Ab ppl in prisons

S.
718.2(e)
and Other
Racialized
Groups

Gender
and
Sentencin
g

R v Borde (2003 Ont CA) and R v Hamilton (Ont SCJ 2003) potentially broaden scope of
718.2(e) to include offenders from racialized groups other Aboriginal

Sentences are less for women who fit traditional female s/t (two parent fam,
dependent, gentle, compliant); women who dont conform to this tend to be treated as
men
Discrim common agst young girls; more likely than young men to be brought to youth
court and detained for their own protection
Even with equal sentences, impact tends to be > on women less likely to be able to
pay same fines as men, imprisonment psychologically harder for women, fewer
womens prisons means further from support systems, more likely to have suffered
physical and emotional abuse prior to conviction
For same crim behaviour, poor more likely to be arrested, charged, convicted and sent
to prison, longer sentences, etc than middle and upper class
Often directly related to employment of police discretion; surveillance heavier in low
income areas
More likely to self represent
on gender, poverty and sentencing: black and Aboriginals overrepresented idea of
fixing young girls, women who are poor mothers shoplifting, fewer womens prisons
self injurious behaviour, separation from child discriminates against the poor and
harms as many people as it helps

The Poor
and
Sentencin
g

R v GLADUE (SCC 1999)


Ratio:

1. Purpose of s. 718.2(e) is remedial, aimed at reducing overrep of Ab ppl in prisons and


encourage restorative approach to sentencing.
2. Sentencing judges must consider
a) systemic factors that may have brought Ab to courts
b) appropriate types of sentencing procedures and sanctions given Ab heritage;
3. s. 718.2(e) not to be construed as automatically giving shorter sentences for Abs;
4. Applies equally to on and off-reserve Abs in rural and urban areas
5. Based on these factors, jail term for Ab offender may in some cases be less than non-Ab
offender, h/e more serious crime, more likely sentence will be same.
Facts: Ab woman pled guilty to manslaughter for killing CL husband after suspecting him of
having affair with her sister; sentencing judge concluded that sentence of 3 yrs in prison was
appropriate and that there were no special circumstances related to her Ab status that should
be considered. Gladue appeal sentence; CoA dismissed.
Issues: How should s. 718.2(e) be applied by judges?
Decision: Appeal dismissed; new hearing ordered that considers circumstances as an Ab
offender
Reasons: see above; also trial judge and CoA erred in not considering systemic factors and
limiting application of s. 718.2(e) to Abs living on reserve or in rural areas
R v IPEELEE [2012 SCC 13]:
Ratio:
1. Consideration of systemic factors doesnt offer an excuse but rather a context to determine
a fair and just sentence
2. Gladue principles need to be considered in EVERY case involving Ab offenders, including
those involving serious offences
Facts: Ab offender who was given longterm offender supervision (LTSO) order of 10 years;
breached order by committing fraud while intoxicated; given three year prison sentence
Issues: Are trial courts properly applying Gladue principles when dealing with s. 718.2(e)?
Decision: Sentence changed to 1 year
Reasons: Courts below did not properly consider Gladue principles.
Over a decade has passed since this Court issued its judgment in Gladue. As the
statistics indicate, section 718.2(e) of the Criminal Code has not had a discernable
impact on overrepresentation of Aboriginal people in the criminal justice system.
Granted, the Gladue principles were never expected to provide a panacea. There is
some indication, however, from both the academic commentary and the jurisprudence,
that the failure can be attributed to some extent to a fundamental misunderstanding
and misapplication off both 718.2(e) and this Courts decision in Gladue. (para. 63, per
LeBel J.)
Systemic and background factors do not operate as an excuse or justification for the
criminal conduct. Rather, they provide the necessary context to enable a judge to
determine an appropriate sentence. (para. 83)
error of law not to consider Gladue principles (2012)
Precedent: Dealt with the fact that Gladue didnt have much impact of lived realities of
Aboriginal ppl
R v BORDE (Ont CA 2003)
Ratio: Leaves open the possibility of considering background factors of black youth in urban
areas in other cases but needs to be addressed at trial and link for African Canadian
communitys connection to restorative justice is missing
Facts: Young, black male from Regent Park, record of violent crime, alcohol abuse, bad family
life; plead guilty to charges of agg assault and firearms; given sentence of 5y2m; on appeal,
argued that sentence should be reduced bc of systemic factors affecting young, black males
raised in urban ghettos

Issues: Can s. 718.2(e) be broadened to consider systemic and background factors of other
racialized groups?
Decision: Appeal dismissed; background factors not considered bc of seriousness of offence;
sentence reduced by one yr bc trial judge didnt give proper consideration to his age (18)
Reasons: Crime here was very serious but judge held that systemic racism and background
factors faced by black youth in Toronto are impt and could affect sentence in another case
R v HAMILTON (Ont SCJ 2003)
Facts: Hamilton and Mason acted as drug mules from Jamaica; both were single, black
mothers with 3 children and on social assistance, no prior crim record;
Issues: Is a conditional sentence appropriate once systemic and background factors are
considered?
Decision: Conditional sentences are appropriate given the circumstances; Hamilton 20m,
Mason 24m
Reasons: Hill, J -- In the Brampton area, 1) blacks of either gender or disprp charged w
importing cocaine compared to rest of popn 2) black, single mothers charged and sentenced
to pen for cocaine importing disprop to their percentage of the popn - race/poverty/gender
are proven factors
R v HAMILTON (Ont CA 2004)
Doherty, J Hill, J took on role of advocate in sentencing hearing and conditional sentences
generally not a fit sentence for drug couriers who import hard drugs even if the courier is
preyed on by drug lords; sentences increased to 2y in prison; where race/poverty/gender are
proven factors to the commission of the crime, those can be properly considered as possible
mitigating factors arriving a just sentence.

II. LIMITS OF CRIMINAL LAW


conceptual and institutional framing

Harm
Principle
(John Stuart
Mill On
Liberty)

Critiques of
Harm
Principle

Only basis on which you can validly use state power is to prevent people
from doing harm to others
So if youre not doing any harm to others, then the state should have no
right to interfere in personal affairs, least of all physical boundaries
Liberal response; criminal law should be based on the harm principle
There is deeply ingrained in the political and
philosophical tradition in western societies, the belief
that the fundamental value is the freedom of
individuals to develop their own view of life, without
direction from the state or from the majority.... Fraser
Committee Report
Two main critiques against harm principle:
1) Anything can be a harm
a. (indecency, harm to a vulnerable community, health care costs
(what counts as harm depends on what you value) it begins with the
question this matters)
2) has to do with the very theory of the state
a. May not believe that role of state is to leave us alone; if you believe
that state must act and create a community of equality, then there
is a positive duty to build liberty state help out not state keep
out
b. (A number of feminist theories). Limits to the criminal law is a

framing question
Devlin

Criminal law can do more than just prevent harm

Hart (Liberal)

Society isnt a society if its only based on harm; societies are also about
community values as well; criminal law plays a fundamental role in holding
those values together
disgust on omnibus
Defends harm principle and says that moral codes wont collapse without
criminal law; most violent form of intervention isnt required (ie. criminal
law)
Criminal law isnt reacting legitimately unless its responding to a harm
done from one to another (Liberal position)

R v MALMO-LEVINE 2003 SCC


marijuana case defines that harm is not principle of fundamental justice
Ratio: It is NOT unconstitutional to pass a law that doesnt cause harm to others. Causing
harm is not a necessary requirement for an act to be criminal.
PFJ must meet 3 criteria:
1) is it a legal principle?
2) is there sig societal consensus that it is fundamental to the way the legal system should
operate?
3) Is it identified with sufficient precision to yield a manageable standard against which to
measure deprivation of life, liberty and security of the person?
Facts: Challenge to the criminalization of marijuana. M-L argues that criminal law is limited to
conduct that causes harm and that the harm principle should be a principle of fundamental
justice.
Issues:
1)Does parliament have legislative authority to criminalize simple possession of marijuana?
2) If so, is power contrary to Charter by offending a PFJ under s. 7?
Section 7 of the Charter: Everyone has the right to life, liberty and security of the person
and the right not to be deprived thereof except in accordance with the principles of
fundamental justice.
3) Is the harm principle a principle of fundamental justice?
Decision: 1) Parliament w/i jurisdiction to criminalize marijuana under NCA 2/3) Causing harm
is not a necessary requirement for an act to be criminal. Crimes without harm are
constitutionally justifiable and do not offend the principle of fundamental justice under S. 7 of
the Charter
PFJ must meet 3 criteria:
1) is it a legal principle?
2) is there sig societal consensus that it is fundamental to the way the legal system should
operate?
3) Is it identified with sufficient precision to yield a manageable standard against which to
measure deprivation of life, liberty and security of the person?
Analysis:
1) No. Harm principle is an important state interest, not a legal principle - the state may
sometimes be justified in criminalizing conduct that is either not harmful (in the sense
contemplated by the harm principle), or that causes harm only to the accused... (para. 114)
2) No societal consensus; presence of harm to others may justify legislative action, but
absence of harm does not create barrier to legislative action; no general prohibition against
criminalizing harm to self (e.g. seat belts)
3) No. Cannot yield a precise definition; anything can be a harm; people will differ on what
counts as a harm

Dissent:
A law that has the potential to convict a person whose conduct causes little or no reasoned
risk of harm to others offends the principles of fundamental justice and, if imprisonment is
available as a penalty, such a law then violates a persons right to liberty under s. 7 of the
Charter. (para. 190)
Arbour Harm principle is a PFJ; where state seeks to imprison, there must be clear and
tangible harm to others; law that has no potential to harm others and has imprisonment as a
penalty offends PFJ and contrary to Charter
R v LABAYE 2005 SCC
Ratio: To be criminal, the harm must be one which society as a whole recognizes as
incompatible with proper functioning
TEST FOR INDECENCY: Indecent criminal conduct est where Crown proves BRD that:
1) By its NATURE, the conduct at issue causes harm or presents a significant risk of harm to
individuals or society in a way that undermines or threatens to undermine a value reflected in
an thus formally endorsed through the Constitution or similar fundamental laws by, for
example: confronting members of the public with conduct that significantly interferes with
their autonomy and liberty, predisposing others to anti-social behaviour, physically or
psychologically harming persons involved in the conduct
2) Harm is of a DEGREE that is incompatible with proper functioning of society (Objective test)
Facts: Group sex club, limited to members who were interviewed and paid an annual
membership fee. Doorman manned the main door to admit only members and group sex only
took place on 3rd floor which was separate from rest of clubone door marked Prive and the
other locked with numeric key pad.
Charge: s. 210(1) - keeping common bawdy-house undera place kept for purposes of acts of
indecency.
Issues: Were the acts committed acts of indecency w/i the meaning of the criminal law? (ie.
what does indecency mean for the CCC)?
Decision: Acquitted. On test for indecency, 1) Crown did not est that nature of harm
threatened a value in Constitution 2) Unnecessary to move on to step 2, but if did, no
evidence that degree of harm incompatible with functioning of society
Reasons:
TEST FOR INDECENCY: Indecent criminal conduct est where Crown proves BRD that:
1) By its NATURE, the conduct at issue causes harm or presents a significant risk of harm to
individuals or society in a way that undermines or threatens to undermine a value reflected in
an thus formally endorsed through the Constitution or similar fundamental laws by, for
example: confronting members of the public with conduct that significantly interferes with
their autonomy and liberty, predisposing others to anti-social behavior, physically or
psychologically harming persons involved in the conduct
3) Harm is of a DEGREE that is incompatible with proper functioning of society (Objective test)
Dissent: Bastarache, LeBel -- Maintain the community standard of tolerance test; harm is a
factor for determining an acceptable level of tolerance; situation here would not meet
Canadian community standard for tolerance and would therefore be indecent
R v CARTER BCSC 2012 on appeal to SCC
Ratio: The absolute prohibition against physician assisted suicide is unconstitutional and
should be replaced with a highly regulated scheme
Facts: Const challenge brought by chronically ill patient, Gloria Taylor, and Carter and Johnson
who helped arrange a doctor in Switzerland; Shoichet a family doc in BC who would perform
PAS if legal

Charge: s. 241(b) aiding or abetting suicide


Issues: Do the CCC provisions prohibiting physician-assisted dying violate ss. 7 and 15 of the
Charter?
Decision: Yes. CCC provisions declared invalid, suspended for one year; during year, Gloria
Taylor given const exemption to permit option of physician-assisted death provided meet
criteria (enduring, serious physical or psych, intolerable, cannot be alleviated, competent,
voluntary)
Reasons: Section 15: the law does not prohibit suicide, h/e disabled are denied the option to
do so bc s. 241(b) prohibits asst suicide; creates a more burdensome effect of those with
physical disabilities and therefore create a distinction not justified by s. 1 (no min imp)
Section 7: Deprives s. 7 rights inconsistent with PFJ legislation is overbroad and absolute
prohibition is grossly disproportionate to the objective; not justified under s. 1
Ethical Debate: 1) if law permitted it, would it be ethical for a physician to assist a
competent and informed patient to hasten death in an individual case? 2) Even if it were
ethical, would a change in law or policy pose a threat to other people or societal values?
Conclusions 1) There are physicians who would in some circumstances find it ethical to assist
patients in hastening death 2) There are no ethical distinctions bw physician assisted suicide
and other end of life practices whose outcome is likely death 3) There is no ethical distinction
bw suicide and assisted suicide physical provides a means for doing something which itself is
ethically permissible 3) No clear societal consensus
Precedent: Rodriguez only determined that s. 241(b) engages s.7 (SOP and liberty),
legislation not arbitrary and the absolute prohibition is meant to protect vulnerable persons
from being induced to commit suicide
CANADA (ATTORNEY GENERAL) v. BEDFORD
Ratio:
Issues: Whether the laws Parliament has enacted on how prostitution may be carried out
1) Keeping a Common Bawdy House
2) Living off the Avails of Prostitution
3) Communicating for the Purpose of Prostitution
violate s.7?
7. Everyone has the right to life, liberty and security of the person and the right not to be
deprived thereof except in accordance with the principles of fundamental justice.
Decision: Provisions unconstitutional suspended declaration of invalidity for legislature to
respond.
Reasons:
1. Do they deprive security of the person, engaging s.7?
By imposing dangerous conditions on prostitution... They prevent people engaged in a risky
but legal activity from taking steps to protect themselves from the risks. (para. 60)
2. Is this deprivation in accordance with principles of fundamental justice?
The principles of fundamental justice set out the minimum requirements that a law that
negatively impacts on a persons life, liberty, or security of the person must meet. (para 94)
The s. 7 analysis is concerned with capturing inherently bad laws: that is, laws that take away
life, liberty, or security of the person in a way that runs afoul of our basic values. The
principles of fundamental justice are an attempt to capture those values. (para 96) in this
case concerned with arbitrariness, overbreadth, and gross disproportionality to the
legislative goal laws that violate these norms are not in accordance with PFJ one
of these effects on one person is enough to establish s.7 breach (qualitative, not
quantitative)
3. Are infringements justified under s.1? not saved

Note: VERTICAL/HORIZONTAL PRECEDENT


Vertical:
A lower court is not bound to follow a prior higher court decision when a new legal
issue is raised, or if there is a significant change in the circumstances or evidence.
decision from Prostitution Refernecein 1990, but everything shifted
Horizontal:
The SCC may itself depart from its previous decisions where it thinks it is necessary in
light of a balancing exercise in which the Court must weigh correctness against
certainty. balance need for certainty vs. how important to get it right

III. CODIFICATION OF THE CRIMINAL LAW


Constitution Act, 1867
s. 91 federal parliament can only pass laws which are in pith and substance federal (includes
peace, order and good government clause)
s. 92 provincial laws

Prior to Charter in criminal law, div of powers was one of the main ways of protecting civil
liberties; constrained provincial incursions into things that were constraints on civil right
breadth produces conflict w provincial legislation ie. property and civil rights provincial
legislation against theft; heart of constitutional law was to sort out powers one thing to
resolve is pith and substance test what is this really about

FEDERAL
s. 91(27) - Criminal law and procedure
s. 91(28) Penitentiaries

PROVINCIAL

92(6) - Reformatory prisons


92(7) - Asylums (mental hospitals)
92(13) - Property and civil rights
92(14) - Administration of justice
92(15) - Fines, penalties

SCOPE of Criminal Law Power (Firearms)

To be
1.
2.
3.

a valid criminal law for purposes of 91(27):


prohibition
penalty
criminal law purposes aimed at some evil or injurious effect (e.g. public peace, health,
security, order, morality or some other legitimate purpose) (ie. marijuana casemorality)
federal criminal law is extremely broad

VALIDITY of Legislation
1. Must be in jurisdiction (Ebling, Morgentaler)
2. Must not conflict with Charter

SWITZMAN V ELBLING 1957 SCC


Pre Charter
Ratio: If a law is in pith and substance about criminal law, then it is federal jurisdiction. A
provincial law that tries to regulate that is ultra vires (outside the power) the provincial
government and therefore unconstl.
Facts: Provincial law, Act Respecting Commmunistic Propaganda said you could be imprisoned
for communicating about communism or bolshevism.
Issues: Is this in pith and substance a criminal law matter and therefore outside the power of
the province to regulate?
Decision: Yes. Law is unconstitutional
Reasons: Its in pith and substance criminal and therefore federal jurisdiction
Precedent: Today would be dealt with under the Charter (freedom of expression).

A. DIVISION OF POWERS
R v MORGENTALER 1994 SCC
Charter case under s. 7 moved from pie to bagel rights protected zone
Ratio: Laws that are in pith and substance about criminal law are federal jurisdiction
Facts: Nova Scotia created a provincial law under civil and property rights powers that
prohibited women from obtaining abortions in hospitals.
Issues: Are Nova Scotia Medical Services Act and its abortion regulations ultra vires the
province of Nova Scotia on the ground that they are in pith and substance criminal law?
Decision: Yes. Struck down as unconstitutional
Reasons: Provincial invasion of federal criminal law power

B. SCOPE OF CRIMINAL LAW POWER


REFERENCE RE FIREARMS ACT 2000 SCC
Ratio: Three prerequisites for a law to be classified as criminal: 1. Prohibition 2. Penalty 3.
Criminal law purpose aimed at some evil or injurious effect
Facts: Alberta challenged federal governments constitutional authority to enact Firearms Act,
which required all holders of firearms to obtain licenses and register their guns; govt argued it
fell w/i criminal law (s. 92(14)); prov argued it fell under property and civil rights (s. 92(13))
Issues: Whether parliament has the constitutional authority to enact the law. (Must look to the
division of powers in Constitution for the answer and determine pith and substance).
Decision: Law is within federal jurisdiction and therefore constitutional
Reasons: Pith and Substance of the law is dedicated to enhancing public safety by controlling
firearmstherefore, it comes within Parliaments jurisdiction over criminal law.

C. COMMON LAW DEFENCES


AMATO V THE QUEEN 1982 SCC
Ratio: Common law offences are prohibited under s 9(a) of CCC, but defences, excuses or
justifications do exist at common law under s. 8(3)
Facts: s. 7(3) of CCC holds that every common law rule that renders any circumstance a
justification, excuse or defence will prevail wrt CCC provisions.
Issues: Is there a common law defence of entrapment through s. 7(3)?
Decision: This case ruled that s. 7(3) (now 8(3)) of CCC can be interpreted as giving the
common law defence of entrapment.

D. COMMON LAW OFFENCES


FREY V FEDORUK 1950 SCC
Ratio: Common law offences are no longer recognized. No one can be convicted of a crime that
isnt already an offence under the Crim Code or other federal criminal statute (exception:
contempt of court); the idea of a code is to be exhausted, give good notice, and make it rational
clear and predictable code in this sense in 1953
Nullum Crimen Sine Lege Nulla Poena Sine Lege maxim of no crime, no punishment
Note: common law still used to interpret what crimes are about/mean
Facts: Frey was caught by Fedoruk looking into window of womans house.
Issues: Can someone be convicted of a crime that is not recognized as an offence under the
CCC? Should courts continue to expand list of common law offences?
Decision: Courts will no longer use their authority to expand list of offenses at common law b/c
courts arent democratically elected and shouldnt create laws based on values. Parliament is

where laws are created.


Precedent: Later, ammendments to crim code created: offence of trespassing at night (1953)
and voyeurism (2005)
JOBIDON V THE QUEEN 1991 SCC
Crime understood in conjunction with common law
Ratio: At common law, you are not allowed to consent to serious bodily harm. Consent is not a
defence to assault. (used here to show how words in CCC get interpreted at CL); Only exception
from s. 9 is contempt of Court
Facts: Jobidon got into a consensual fight with the victim outside a bar. Jobidons first punch
rendered the victim unconscious and after a subsequent flurry of punches, the victim lay limp
and later died of contusions to the head. The trial judge acquitted b/c says there was no
unlawful act because the victim consented to the fight; the appeal court overruled this and
convicted Jobidon of manslaughter. The accused appealed.
Charge: manslaughter 222.5 culpable homicide resulting in death by a) unlawful death b) crim.
negligence
Issues: Is consent a defence to assault?
Decision: No. Conviction upheld
Reasons: Gonthier - There are common law limitations on consentconsent between
adults is vitiated in the case of fist fights (Jobidon) but this doesnt apply to sporting events if
actions are within the rules and regulations of the game. Also, must think about what victim
consented tono way you can give consent when unconscious.
Dissent: Sopinka - No new offences are supposed to be made at common law and majoritys
application creates a new offence intentional application of force with the consent of the
victim. Use of CL to eliminate an element of the offence that is required by statute is more than
just interpretation

STATUTORY INTERPRETATION - VAGUENESS, OVERBREADTH AND


CERTAINTY
i) Vagueness
(Nova Scotia
Pharmceutical
Society)

ii)
Overbreadth
(R v. Heywood)
iii) Strict
Construction
(Certainty)
(R v. Par)

Laws arent clearly defined


To determine vagueness, court must ask:
1. Is the law capable of delineating a zone of risk? Of giving a workable
standard by which individuals can guide their conduct? (Fair Notice)
2. Is the law precise enough to constrain the actions of an enforcer or
does it create a standardless sweep? (Limitation of Enforcement
Discretion)
3. Does the impugned law give a basis for legal debate and analysis a
grasp for the judiciary in interpreting the provision?
Judge is interpreting after the fact; If a judge wants to be able to
interpret a law, they will; vagueness arguments are therefore incredibly
difficult to win
Laws are too sweeping in relation to the objective - too much
conduct captured
Laws cannot be broader than necessary to accomplish their goal lest
they become arbitrary and disproportionate: Heywood 1994.
If a provision is reasonably capable of two interpretations, that
interpretation which is the more favourable to the accused must be
adopted (least impact on liberty).
o This does NOT mean that a word which has two accepted
meanings must always be given the more restrictive meaning.
o To determine the correct meaning, Court must first endeavour to

10

i)

determine the sense in which Parliament used the word from the
context in which it appears.
ALWAYS ASK:
o What is the purpose of the statute?
o What kind of conduct is it designed to cover.

Vagueness

R v NOVA SCOTIA PHARMACEUTICAL SOCIETY 1992 SCC


Ratio: Vagueness can be raised under s. 7 since it is a PFJ that laws cannot be too vague.
Laws that are too vague are unconstitutional
To determine vagueness, court must ask:
1. Is the law capable of delineating a zone of risk? Of giving a workable standard by which
individuals can guide their conduct? (Fair Notice)
2. Is the law precise enough to constrain the actions of an enforcer or does it create a
standardless sweep? (Limitation of Enforcement Discretion)
3. Does the impugned law give a basis for legal debate and analysis a grasp for the
judiciary in interpreting the provision?
Facts: Challenge to s. 32(1) of Combines Investigation Act which made it an offence to
lessen, unduly, competition
Issues: Is the use of the term unduly too vague thereby making the law unconstitutional?
Decision: No. In this cases unduly is not impermissibly vague.

ii) Overbreadth
R v HEYWOOD
Ratio: Laws that are too sweeping in relation to their objective are overbroad and therefore
unconstitutional. Different from vague laws, where the meaning is ambiguous; both are result
of lack of sufficient precision by legislature in means to accomplish the objective.
Facts: Accused was a convicted sex offender charged with vagrancy, by being a person who
had been convicted of a sexual offence and found loitering at or near a school ground,
playground, public park, or bathing area which violated s. 179(1)(b) of CCC. He was also
observed with camera at playground, taking pictures.
Issues: Is the law overbroad and therefore unconstitutional?
Decision: Yes. S. 179(1)(b) restricts liberty more than needed to accomplish its goal of
preventing re-offense and is void for overbreadth.
Reasons: Corey -- Law is overbroad in 1) geographical scope bc it embraces all public parks
and beaches no matter how devoid of children 2) temporal aspect as prohibition applies for life
w/o review 3) number of persons it encompasses 4) may be enforced w/o any notice to the
accused.
Dissent: Gonthier Law is not overbroad; should read into the law activities with a
malevolent or ulterior purpose related to reoffending

ii)

Strict Construction (Certainty)

R v PARE 1987 SCC


Ratio: Strict construction is used only if ambiguity still exists after purposive methods
of interpretation. Must first look at the intention of parliament and the acts the law is
designed to cover in determining how to interpret ambiguous language.

11

NOTE: rule has little bite - judges first figure out intention and infer what meant only rule
against strict construction if cant find out; rule comes from a time when death was result for
most convictions; still serves a couple purposes ensures liberty only affected if parliament is
clear- serves purpose of giving notice- in favour of the accused
Goulis under obligation to disclose assets, charged with fraudulent concealment bc of
shoes does concealing require active hiding or failure to disclose court of appeal said
agree with result but dont need rule of strict construction to get there- interpret as
active concealment - only in presence in genuine ambiguity
Facts: The accused indecently assaulted a boy, threatened to kill him, and killed him 2
minutes later.
Charge: s. 214 (5) (now s. 231(5)) first degree murder occurs when the death is caused
while committing an indecent assault (now sexual assault)
Issues: Did accused kill victim while committing indecent assault?
Decision: Pare found guilty of first degree murder. Entire course of conduct was one single
transaction.
Reasons: Wilson if you look at intention and purpose of statute, the approach that requires
you to interpret while as exactly simultaneous is problematic: 1) difficult to define beginning
and end of an assault 2) leads to distinctions that are arbitrary and irrational (ie. if murdered
two min earlier, clear 1st degree) 3) where acts leading up to death are all one single
transaction, the death was caused while committing indecent assault. and apply notion that
you would reach an absurd result of a less serious crime rather than more serious, there is no
ambiguity and principle of strict construction is not needed; In addition, policy considerations
underlying the provision (ie. increasing penalties in such acts) suggest that single transaction
analysis is better approach.

IV. EXCLUSION OF EVIDENCE


Two Key Rights:

Section 7: Everyone has the right to life, liberty and security of the person and the right
not to be deprived thereof except in accordance with the principles of fundamental
justice.
Section 15(1): Every individual is equal before and under the law and has the right to
the equal protection and equal benefit of the law without discrimination and, in
particular, without discrimination based on race, national or ethnic origin, colour,
religion, sex, age or mental or physical disability

Key Constitutional Provisions


o Section 52(1) [Constitution Act, 1982]:
The Constitution of Canada is the supreme law of Canada, and any law that
is inconsistent with the provisions of the Constitution is, to the extent of
the inconsistency, of no force or effect.
o Section 1 [Charter]:
The Canadian Charter of Rights and Freedoms guarantees the rights and
freedoms set out in it subject only to such reasonable limits prescribed by
law as can be demonstrably justified in a free and democratic society.
o Section 24(2) [Charter]:
Where, in proceedings under subsection (1), a court concludes that
evidence was obtained in a manner that infringed or denied any rights or
freedoms guaranteed by this Charter, the evidence shall be excluded if it is
established that, having regard to all the circumstances, the admission of it
in the proceedings would bring the administration of justice into disrepute.

12

CHARTER CLAIM IN THE CRIMINAL LAW


There is a breach of a Charter right

What caused the breach?



A Law
State Action

Is
the legally prescribed breach justifed under s.
1?
The remed
is that the evidence obtained by the state action might be excluded under s. 24(2) of the
Charter.

Yes
No

Since it is a justified breach, the law


is constitutionally sound. No remedy is
necessary
The offending law is
inconsistent with the
constitution and so is of no
force or effect (s. 52)

OAKES TEST
Government has the burden of proving, on a balance of probabilities, that a breach is a
reasonable limit, prescribed by law, and justified in a free and democratic society.
1. Is the limit on the right (the breach) prescribed by law?
2. Is there a pressing and substantial objective/purpose for the limit on the right?
3. Is the offending law proportionate?
a. Is the limit rationally connected to the purpose/objective?
b. Does the limit minimally impair the right in order to achieve the objective?
c. Are the deleterious effects of the limit proportionate to its salutary effects? heart is
proportionality

EXCLUSION OF EVIDENCE (POST-GRANT)


Evidence is obtained in breach of a Charter right

Section 24(2)

KEY QUESTION: Would a reasonable person, informed of all the relevant circumstances and
the values underlying the Charter, conclude that the admission of the evidence would bring
the administration of justice into disrepute?

THREE LINES OF INQUIRY:


1. The seriousness of the Charter-infringing state conduct (Worried about message
that j.s. condones serious state misconduct). Consider: inadvertent or minor? Wilful or
reckless disregard for Charter rights? Good faith by police? Part of a pattern of abusce?
2. The impact of the breach on the Charter-protected interests of the accused
(Worried about message that individual rights count for littler) Consider: interests
engaged by infringed right; degree to which violation impacted interests (e.g. high
expectation of privacy?)
3. Societys interest in the adjudication of the case on its merits (Worried about
impact of failing to admit the evidence on the truth-seeking function of the criminal

13

trial) Consider: is the illegally obtained evidence reliable? How important is the evidence
to the Crowns case? Seriousness of the offence (cuts both ways)

Given the assessments under each of these three lines of inquiry, would the admission
of evidence obtained by Charter breach bring the admin of justice into disrepute?

Yes No

Evidence excluded pursuant to s. 24(2)


The evidence is admitted
Types of Evidence (from Grant)
a) Statements
b) Bodily Evidence
c) Non-Bodily
d) Derivative
Made By Accused
1 is fact-specific; 2 Physical Evidence Evidence 1 fact
Concern for proper
greater intrusion,
1 fact-specific,
specific and more
police conduct,
more likely to
depends on if conduct serious state conduct,
centrality of protected exclude; 3 usually
was egregious or
more likely
interests affected and favour admission bc
deliberate; 2 privacy
undermines public
questionable
evidence generally
is the main interest
conf; 2 if evidence
reliability favour
reliable
engaged, depends on discoverable, more in
exclusion
REP 3 usually
favour of admission; 3
reliable so tends to
evidence is usually
weigh in favour of
reliable so in favour of
admission
admission
R V. OAKES
Ratio:
Government has the burden of proving, on a balance of probabilities, that a breach is a
reasonable limit, prescribed by law, and justified in a free and democratic society.
1. Is the limit on the right (the breach) prescribed by law?
2. Is there a pressing and substantial objective/purpose for the limit on the right?
3. Is the offending law proportionate?
a. Is the limit rationally connected to the purpose/objective?
b. Does the limit minimally impair the right in order to achieve the objective?
c. Are the deleterious effects of the limit proportionate to its salutary effects? heart is
proportionality
Facts: Oakes challenged s. 8 of the Narcotics Control Act, which held that if the court finds the
accused in possession of a narcotic, he is presumed to be in possession for purpose of
trafficking. Oakes was found was weed and hash oil. Oakes argued that NCA violates right to
be presumed innocent under s. 11(d) of the Charter bc law places burden on accused to prove
that s/he is not in possession for purpose of trafficking.
Issues: Is s. 8 of NCA constitutional?
Decision: No. Law should be struck down.
Reasons: S. 8 compels the accused to prove that he is not guilty and therefore violates his
right to be presumed innocent. The infringement is not justified under s. 1. There was a
pressing and substantial concern (conviction of drug traffickers), h/e it fails on the rational
connection test. Irrational to assume a person has intent to traffic on basis of possessing a
small quantity of narcotics.

STATE ACTION & EXCLUSION OF EVIDENCE


R V COLLINS 1987 SCC
Ratio: Collins Test established to determine whether admission of evidence would bring the
administration of justice into disrepute.

14

1. Would admitting the evidence adversely affect the fairness of the trial?
a. Is the evidence conscriptive? (yes, then go to b; no, then go to 2)
b. Is the evidence discoverable? (yes, then go to 2; no, then admission of evidence
will render trial unfair and it will generally be excluded w/o considering stages 2
and 3).
2. How serious was the Charter violation?
3. What would be the effect of excluding the evidence on the administration of justice?
Issues: When is it ok to admit evidence obtained through a Charter breach?
Precedent:
Courts ended up interpreting Collins in a different way than what was intended
At the first stage, fairness really meant did the police get conscriptive, non-discoverable
evidence
o If yes, the trial cannot be fair and evidence must be excluded
In practice, it was an automatic exclusion at stage one and never went on to consider 2
and 3.
Courts became increasingly uncomfortable bc evidence was sometimes being excluded for
very minor breaches but never went on to next steps.
Ended up excluding evidence where breach seemed really small bc at CL courts created a
precedent that evidence of a certain kind (ie. consc, non-disc) would automatically be
excluded
Eventually the SCC came up with new test Grant test
R V. STILLMAN 1997 SCC
Ratio: Provided an analytical framework for the first stage of the Collins test. Once shown that
evidence is obtained in breach of Charter:
1. Classify evidence as conscriptive or non-conscriptive based on manner it was obtained.
2. If evidence is conscriptive and Crown fails to show on balance of prob that it would have
been discovered, then its admission will render the trial unfair.
3. If evidence is conscriptive and Crown shows on bal of prob that it was discoverable, then its
admission will generally not render the trial unfair. Will then consider remaining two steps.
Facts: Stillman, 17, charged with murder of 14 yr old girl. Lawyers advised police that he was
not giving any bodily samples. After lawyers left, Police took samples under threat of force
(removed hair, teeth impressions and mucou from Kleenex).
Issues: Should the evidence obtained through police force be excluded under s. 24(2)
Reasons: Conscriptive evidence will generally render a trial unfair unless the evidence was
discoverable.
Vocabulary:
Conscriptive evidence evidence that requires you to incriminate yourself at the behest of
the state by means of a statement, the use of the body or production of bodily samples.
Derivative Evidence Another kind of conscriptive evidence; evidence that is not a
statement, use of body or bodily sample, but is found as a result of these things.
Discoverability Conscriptive evidence might be discoverable if it was capable of being
found by other constitutional means either through other sources or it was inevitable that it
would be found anyway.
R V. GRANT 2009 SCC
Ratio: See new Grant test for determining exclusion of evidence.
Main question: would admitting the evidence bring the administration of justice into disrepute?
Judge must consider three lines of inquiry from a longterm, forward-looking and societal
perspective:
Seriousness of Charter-infringing conduct
Impact of the Charter breach on the protected interests of the accused

15

What is societys interest in the adjudication of the case on its merits?


After that, ask would the admission of evidence bring the administration of justice into
disrepute?
No overarching rule governs how the balance is to be struck. (para 86) essentially trying to decide if reasonable person informed of circustances and
values under charter would conclude admission of evidence would bring
admin of justice into disrepute

Facts: Police patrolling near Toronto school. Grant, a young, black man was walking down the
street and came to attention of two plainclothes officers. They say him fidgeting with jacket
and told a third, uniformed officer to chat with him on the street. He was asked for name and
address and after adjusting his jacket, asked to keep hands in front. Two additional officers
came back. When Grant was asked if he had anything he shouldnt, he replied a small bag of
weed and a firearm. conscriptive statement led to evidence
Issues: Was there a violation of ss. 8, 9, 10(b) of the Charter? If so, should evidence obtained
through a Charter breach be excluded?
Decision: Held. The gun should be admitted into evidence under s 24(2)
Reasons: Gun is derivative evidence. 1. police conduct not at the most serious end of the
spectrum 2. impact on liberty not egregious 3. Gun is highly reliable evidence and value of
evidence is extremely high. It is essential to a determination of the case on its merits.
Weighing all the concerns, the admission of the evidence would not bring the admin of justice
into disrepute.
Precedent: The weed and firearm are conscriptive, derivative evidence the kind that would
have been excluded under Collins/Stillman
R V HARRISON 2009 SCC
Ratio: Application of Grant test
Facts: Accused travelling across Canada in rental car. Police noticed the car had no front
license plate and was going speed limit, which was unusual on that road. No grounds to stop,
but did so bc of concern about integrity of police to observers if didnt. Accused had
suspended license and was charged. Police searched car incident to arrest to find license.
Found 35 kg of cocaine.
Issues: Was there a Charter breach? If so, should the evidence obtained through the breach
be excluded?
Decision: Search breached ss 8 and 9 of Charter. Evidence should be excluded.
Reasons: Charter breach initial stop should not have been made and search was not
incidental to arrest. Exclusion of evidence Egregious police conduct and disregard for rights.
They knew it was an illegal search but did it anyway.
What will Grant mean?
The trial judge drew the line where the police had continually shown systematic disregard for the law
and the Constitution. The trial judge did not err in concluding that the courts must not tolerate this sort
of behaviour by those sworn to uphold the law. He took the only course open to him in order to prevent
the administration of justice from falling into further disrepute by condoning this disturbing and aberrant
police behaviour. (R. v. Cot, 2011 SCC 46, para. 4)

V. SEARCH AND SEIZURE


Due Process v. Crime Control (Herbert Packer): moved toward due process (ie. Cole
horrible crime, crucial evidence but conduct is so bad that we dont want to associate
with that systematic disregard for rights); feminist critique is involved here
16

underneath law, have power and relationships values communicated arent neutral
reasonable or objective how generalizable is this really
Crime Control Model:
o Metaphor: the assembly line things should flow along well
o Goal: the conviction of the guilty
o Core Value: truth
Due Process Model:
o Metaphor: the obstacle course
o Goal: the protection of rights
o Core Value: fairness

POLICE POWERS
When dealing with
exercise of police
powers, ask:
(Collins)

1. By what legal authority did the office act? (i.e. statutes)


2. Did the officer comply with the legal authority?
3. Even if so, is the legal authority constitutionally sound?

Section 8

Everyone has the right to be secure against unreasonable search or


seizure. qualified right = protected from un not reasonable

REASONABLE EXPECTATION OF PRIVACY


1. Is there a
reasonable
expectation of
privacy?

Section 8 is only engaged when the accused had a REP. If no REP,


then no protection of s. 8 of Charter.
REP determined on the totality of the circumstances
(Edwards)

Considerations include:
1. Type of privacy interest at stake (i.e. personal, territorial,
informational privacy Tessling)
2. Whether the accused has a subjective expectation of privacy and
whether that expectation is objectively reasonable. (Patrick)
3. For property or territorial searches, factors to consider may
include (Edwards):
Presence at the time of the search
Possession of control of the property or place searched
Ownership of the property or place (Belnavis, Edwards)
Historical use of the property or item
Ability to regulate access, including the right to admit or
exclude others from the place
REASONABLE EXPECTATION OF
NO REASONABLE EXPECTATION OF PRIVACY
PRIVACY
Your own bodily samples (Stillman)
Garbage bags left for collection at the edge of
ones property (Patrick)
Entry to your home (Feeney)

The exterior heat signature of your house


Business records in your place of
(Tessling)
business (Hunter v Southam)
A girlfriends apartment (Edwards)
Closed areas of your car when you are
Regulatory seizure of documents (Thomson
the owner/operator (Blenavis; Harrison)
Newspaper)
Rented locker at a bus depot (Buhay)

The contents of a car when one is a


A public even in a private hotel room
passenger (Belnavis)
(Wong)

Sexual activity in a public washroom(LeBeau


Private conversations (Duarte)
and Lofthouse)
Bags, purses, pockets (Grant)

17

Sniffer dog searches of a backpack in a


school or public place (A.M. and KangBrown)
2. Based on this assessment, is there a REP?
If no REP
Section 8 is not engaged and the search is reasonable
If a REP exists
Section 8 is engaged. Determine if the search conducted was reasonable.

REASONABLE SEARCH
In order to be
reasonable under
s. 8, a search or
seizure must
satisfy three
preconditions:
(Collins)

When is a search
reasonable?
(Hunter v
Southam)

3. Assuming a REP
exists, was the
search reasonable?

1. It must be authorized by law


Searches incident to investigative detention, dog sniff searches
and searches incident to arrest are common law powers
Other search powers are derived by statute.
2. The law itself much be reasonable
3. It must be carried out in a reasonable manner
**This is the broader framework for assessing reasonableness under s. 8.
Apply to all searches regardless of the context. Hunter v Southam, etc
only apply in criminal and quasi-criminal cases (provides specific
ingredients for #2 above).
Warrantless searches presumptively breach s. 8 (Hunter v Southam)
When a REP exists, a search is reasonable when:
1. Prior authorization from an impartial judicial or quasi-judicial
body (ie. a warrant)
Standard: reasonable and probable grounds to believe that the
proposed search or seizure will produce evidence of a crime.
(Reasonable suspicion standard sometimes used for less intrusive
searches)
EXCEPTION: Exigent circumstances (e.g. to protect officer safety
or destruction of evidence)
2. There are reasonable and probable grounds, established
under oath, to believe that an offence has been committed
and that there is evidence to be found at the place of the
search
3. Conducted reasonably and within the scope of the warrant
Apply Hunter v Southam:
When a REP exists, a search is reasonable when:
1. Warrant? No presumptively unreasonable. Exigent circumstance
that made it impossible to get warrant?
2. Reasonable & probable grounds?
3. Conducted reasonable w/i warrant?

4. Did the
circumstances
allow for a lower
standard of
suspicion than
Hunter v Southam?
(Kang-Brown, A.M.)

If all three of the above do not exist, it may still be a legal search if
you can prove the search was reasonable under the circumstances.
o Does it fit into another category of search (e.g. search incident
to arrest, sniffer dog)?
Sniffer dog searches (Kang-Brown, A.M.)
These types of searches are reasonable w/o a warrant if they are
based on reasonable suspicion (rather than reasonable and
probable grounds)
Seen as being less intrusive and undertaken in response to on-thespot observations. Sniffer dogs are also highly accurate.
Search must still be conducted reasonably (ie. dog must be properly

18

trained)

Was there
reasonable
suspicion?
5. If search IS
reasonable
5. If search IS NOT
reasonable
7. Was there
reasonable
suspicion?
should the
evidence obtained
be excluded under
s. 24(2)?

What constitutes reasonable suspicion?


Suspicion is an expectation that targeted individual is possibly
engaged in criminal activity. Reasonable suspicion means
something more than mere suspicion and something less than
belief based on reasonable and probable grounds. (Binnie KangBrown)
Requires police officers subjective belief to be backed up by objectively
verifiable indications
YES No Charter breach
NO Breach of s. 8
No Charter breach
Breach of s. 8. Should the evidence be excluded?
YES No Charter breach
NO Breach of s. 8

Apply Grant exclusion of evidence test.

HUNTER V SOUTHAM 1984 SCC


Ratio: Section 8 imposes the requirement that, in the absence of exigent circumstances,
before the police can conduct a search, they must have:
What is a reasonable search? The criteria:
1. Prior authorization from an impartial judicial or quasi-judicial body (a warrant) - Warrantless
searches are presumptively unreasonable and, therefore, presumptively unconstitutional
- says where feasible need a warrant (sometimes not realistic referred to as exigent
circumstances)
2. There are reasonable and probable grounds, established under oath, to believe that an
offence has been committed and that there is evidence to be found at the place of the
search
3. Conducted reasonably and within the scope of the warrant
Facts: Govt of Alberta began investigation of Southam Newspaper under authority of
Combines Investigation Act. Investigators entered Southams offices to examine documents.
Southam asked court for injunction until they could challenge constitutionality of Combines
Investigation Act. Argued that s. 10(1) of Act violated s. 8 of Charter.
Issues: Is the statute authorizing the search and seizure constitutional?
Decision: No. Combines Investigation Act violates s. 8 of the Charter bc it does not meet
standard for administering warrants.
Reasons: Dickson Section 8 is triggered when an individual has a reasonable expectation of
privacy. Must be a balancing of interests of privacy and need for law enforcement and effective
investigation of crime.
R v DUARTE1990 SCC
Ratio: There is a REP in private conversations. A warrant is required to record them as it
constitutes and electronic search
Facts: Undercover police officers recorded conversations they had with suspects.
Issues: Is there an REP? Does that amount to a search w/i meaning of s. 8?

19

Decision: Yes. Suspects had a REP over information and police require a warrant for such a
recording.
Reasons: There is a REP over private conversations wrt state surveillance. Permitting
unrestricted video surveillance by state agents would seriously diminish the degree of privacy
we can reasonably expect to enjoy in a free and democratic society.
R v WONG 1990 SCC
Ratio: No REP in a private hotel room used for public purposes
Facts: Wong was charged with keeping a common gaming house. He put up posters inviting
the public to gamble in his hotel room. Police put up a video camera in the hotel. Crown
argued that there was no search bc he had no REP. Wong argued that videotape amounted to
an illegal search and violated s. 8.
Issues: Did Wong have an REP in a private hotel used for public gambling? If so, was the
search in violation of s. 8 of the Charter?
Decision: A REP of privacy exists and the videotaping amounted to a warrantless, thereby
violating s. 8 of the Charter. (Evidence, h/e, was not excluded under s. 24(2))
Reasons: LaForest You dont give up your REP in a private hotel room even if you take the
risk of making it available to the public. A REP is with respect to state action. There is a great
distinction bw exposing ourselves to risks wrt to other individuals and with the state. Threat to
privacy from state is much greater.
Concurring: Lamer In most cases, a private hotel room is a place where individuals have a
REP. H/e when Wong invited the public into the hotel he gave up his REP. A reasonable person
would not expect that strangers, and possibly the police, would not be in the room. No REP
and therefore no search w/i meaning of s. 8.
R v EDWARDS 1996 SCC
Ratio: REP depends on the totality of the circumstances In determining property or
territorial REPs, factors to consider may include:
Presence at time of search
Possession or control of the property or place searched
Ownership of the property of place
Historical use of the property of item
Ability to regulate access, include right to admit and exclude
Existence of subjective expectation of privacy
Objective reasonableness of the expection.
Facts: Drugs found during search of Edwards girlfriends apt after she cooperated with the
police. Edwards argued that it was an illegal search.
Issues: Does a man have a REP in his girlfriends apartment?
Decision: No REP
Reasons: Cory -- In considering totality of circumstances, accused had no REP at gfs apt. The
fact that he had a key and sometimes stayed over outweighed by his lack of authority to
regulate access to the premises. This meant he could not be free from intrusion or
interference. He was no more than a privileged guest.
R v BELNAVIS 1997 SCC
Ratio: No REP if you are a passenger in a car. Application of Edwards factors.
Facts: Three women were stopped for speeding. Police officer noticed bags of clothes in back
with tags on them. Upon search, more found in trunk. Girls all charged with possession of
stolen property. Accused argued that search was illegal and in violation of Charter.
Issues: Does a passenger in a car have a REP wrt search of bags and the trunk of the car in
which shes travelling?
Decision: No. Passengers in cars have no REP. Evidence should not be excluded under 24(2).

20

Reasons: No REP bc she did not own the vehicle or have any control over it. No relationship
to driver. Did not control access to vehicle. No subjective evidence that she had expected to
have privacy in the vehicle.
R v TESSLING 2004 SCC
Ratio: No REP over heat signature from home. The type of information obtained is important
to determining whether a REP exists (in this case, heat signature deemed meaningless).
Facts: Police used thermal imaging device to take heat picture of Tesslings home from an
overhead plane. Recorded relative distributions of heat over the surface of the building. Taken
with other evidence, led police to believe there was a marijuana grow-op. Tessling argued that
technology amounted to a warrantless search of his home and violated s. 8.
Issues: Do you have a REP over the heat signature of your home?
Decision: Appeal allowed. No REP over heat signature of your home and therefore no warrant
needed to take it.
Reasons: Binnie -- Applied Edwards totality of circumstances factors The FLIR technology
provides a crude image, which doesnt tell you enough on its own to provide sufficient grounds
for a search warrant. Merely tells police there are heat generating activities in the home. At
this point in time, FLIR technology is non-intrusive and mundane in data it produces. Not the
same as having police inside the home.
R v PATRICK 2009 SCC
Ratio: No REP over garbage bags left at edge of property.
Facts: Police suspected Patrick of operating and ecstasy lab and seized garbage bags left for
collection at rear of his property. The police didnt step foot onto property but reached across
the property line. Used evidence to secure a warrant. Patrick argued that seizure of garbage
violated s. 8 bc he retained a privacy interest in bags.
Issues: Is there a REP over garbage bags left at edge of property?
Decision: No. Section 8 is not engaged.
Reasons: Binnie -- Patrick abandoned privacy interest in contents of bag when he placed the
bags for collection in a location where any passing member of the public had access. Viewed
in totality of circumstances, gave up REP (placed in open container at back of property,
unprotected, in easy reach of passerby)
R v SPENCER 2014
Ratio:

Facts: computer someone was using to store and share child porn; asked Shaw for identity of
persona associated with IP address charged w possession and making available convicted
on possession and argues that he had privacy
Issues:
Decision:
Reasons:
1. The subject matter of the alleged search
a. name and address; Spencer says they were also searching intimate details of
private life revealed in association with particular activities Court says abt
person and associating it with pattern of usage on Internet
2. The nature of the claimants privacy interest in the subject matter
a. in Tessling, Court says privacy can be personal, territorial or information this
was territorial, but SCC says it was informational and abt heat coming off home
much lower privacy interest had no reasonable explanation
b. the closer it is to you, the more important Court says this is informational
could be secrecy, control of info or anonymity interest pretty high level of
informational privacy this actually reveals great deal about you

21

3. The claimants subjective expectation of privacy in the subject matter accused thought
through the state
4. Whether this subjective expectation of privacy was objectively reasonable having
regard to the totality of the circumstances.
a. in Edward how much control over territory, do you own, can you regulate acess
what are totality of circumstances
b. in Spencer, gov said we all sign IS agreements which says might have to give
info to police- contract of adhesion one person creates and you say yes or no
no negotiating room these things in term of privacy are informative but not
dispositive
c. REP, since police x warrant, was unconstitutional; b to s.8 rights but evidence was
admitted, but principally bc law was so unclear and thought this was fine, were
letting this in
Note: Privacy analysis is laden with value judgments which are made from the independent
perspective of the reasonable and informed person who is concerned about the long-term
consequences of government action for the protection of privacy. (para 14)
R v KANG-BROWN; R V A.M.
Ratio:
a) Sniffer dog searches are an exception to Hunter v Southam standard and establish a new
common law police power.
b) There is a REP over smells emanating from personal belongings. Police intrude on a REP
when use a drug-sniffing dog to detect the odour of narcotics coming from an individuals
person or luggage.
c) HOWEVER, sniffer dog searches have a lower standard or suspicion/belief. Sniffer dog
searches are reasonable if the officer has reasonable grounds to suspect (as opposed to
reasonable and probable grounds) that the accused has drugs.
d) Hunter v Southam is the gold standard. Lower standard is given after considering all
circumstances, including minimal intrusion, contraband-specific and high accuracy rate of flyby sniff. Creates a proper balance bw individual and societal interests.
Facts: Kang-Brown: Undercover RCMP at bus station noticed accused get off bus and then
glance at police officer over shoulder. Officer found bahviour suspicious and approached
accused and asked if he was carrying narcotics. Accused said no and snatched bag away as
officer reached out to touch it. RCMP called for sniffer dog who signalled that drugs were in the
bag. Accused searched and arrested.
A.M: Principle of high school gave a standing invitation to police to bring sniffer dogs to school
to search for drugs. Police showed up with dogs. Dog alerted officer to drugs in one of the
bags. Backpack was searched and AM charged.
Issues: Under what circumstances are police allowed to conduct a sniffer dog search of an
individual and his/her belongings?
Decision: Accused acquitted in both. No sufficient grounds for search. Violation of s. 8 and
evidence excluded under s. 24(2)
Reasons: a) Accused had a REP in bags sufficient to trigger s. 8 b) Police are entitled as a
matter of common law to conduct sniffer dog searches w/o a warrant on basis of reasonable
ground to suspect the presence of contraband c) However police did not have sufficient
grounds to conduct the searches
Dissent: LeBel dissented on creation of new standard. Risk that in practice critical elements
of the rights guaranteed by s. 8 will be jeopardized (ie. legal basis in statute or common law,
judicial authorization, reasonable and probable cause). Creation of such CL rules is not an
appropriate exercise of judicial power.
What is reasonable suspicion?
Justice Karakatsanis in Chehil (2013):
it is about the reasonable possibility, not probability, of crime. (para. 27)

22

You look at the constellation of objectively discernible facts that are said to give the investigating
officer reasonable cause to suspect that an individual is involved in the type of criminal activity
under investigation. (para. 29).

VI. ARREST AND DETENTION


JJ Harper and Osborne Harper was stopped bc Aboriginal; outcome was tragic; unless police
are given authority general rule Is that when police officer and citizen meet, the officer can
legitimately ask question and information, but citizen can legitimately refuse to participate of
stop only exception historically was when arrested; could not be arrested unless good reason
reasonable and probable grounds to think person has committed offense (Storrey case)

ARREST POWERS
General Rule
How was the person
arrested? (ie. How
does an accused
become drawn into
system?)

What is needed for a


lawful arrest?

Upon arrest, what are


the police entitled to
do?

When a police officer and an individual meet, unless there is an arrest,


the individual has the option NOT to interact with the police
1. Through Warrant
Results in summons to court to answer to a charge or warrant for
arrest.
Police must show court reasonable and probable grounds that
person committed the crime in order for a warrant to be issued
(Storrey). Otherwise, for less serious offences, summons is issued.
2. Warrantless Arest
Power to make a warrantless arrest turns on the following:
1) Who is making the arrest
a. Citizens Arrest (s. 494 CCC)
- May arrest w/o warrant a) person committing an indictable
offence b) person who on reasonable grounds has committed a
criminal offence and is escaping police c)person found committing
a criminal offence on or in relation to your property
b. Police (s. 495 of CCC)
- a) has committed an indictable offence or on reasonable grounds
is about to; b) person found committing a criminal offence
2) What kind of offence it is
a. Indictable
b. Summary conviction
c. Hybrid (Crown has choice on proceeding by either indictable
or sc
With warrant
Must show judge reasonable and probable grounds that person
committed offence.
Without warrant
1. Criminal Code requires that arresting officer must subjectively
have reasonable and probable grounds on which to arrest.
2. Grounds must also be justifiable from objective perspective. (i.e.
Would a reasonable person in the position of the officer conclude
there were reasonable and probable grounds for the arrest?)
Common Law Power to Search Incident to Arrest (Cloutier v
Langlois)
This is a power, not a duty
Must be for a valid objective in pursuit of the ends of criminal
justice:

23

EXCEPTIONS to
search incident to
arrest

What rights do
individuals have upon
arrest?

o Safety of police
o Securing evidence
o Preventing escape
Must not be done in abusive manner
1. Bodily samples (Stillman)
2. Strip Searches (Golden)
Police must show reasonable and probable grounds to justify the
strip search, and additional grounds for justifying strip search done
in the field, as opposed to the police station.
s10. Everyone has the right on arrest or detention
(a) to be informed promptly of the reasons therefore;
(b) to retain and instruct counsel without delay and to be informed
of that right; and
(c) to have the validity of the detention determined by way of
habeas corpus and to be released if the detention is not lawful.

CLOUTIER V LANGLOIS 1990 SCC


Ratio: Police have a common law power to search incident to arrest. 1) It is a power not a
duty. 2) The search must be for a valid objective in pursuit of the ends of criminal justice why
searching?: a) safety of the police, b) securing evidence or c) preventing escape. 3) Search
must not be abusive and does not extend to bodily substances
Facts: Lawyer charged two police officers with assault after he was frisked following arrest for
unpaid parking tickets.
Issues: Was the frisk lawful?
Decision: Frisk incidental to arrest was lawful.
Reasons: LHeureux-Dube -- Search of the lawyer, who was being verbally abusive, was
justified for the police officers safety and not conducted with excessive force or constraint.
Frisk search incidental to lawful arrest balances public interest in law enforcement and
freedom and dignity of indl.
R V GOLDEN 2001 SCC
Ratio: Strip searches are NOT included in the police power to search incident to arrest.
Reasonable grounds are needed to justify the strip search (additional to establishing grounds
for the arrest). Must be done in police station except in exigent circumstances, which will
require further reasonable and probable grounds that it is necessary to conduct strip search in
field (e.g necessary to search for weapons or threat to safety of accused, officers or others)
Strip searches are only constitutionally valid (reasonable) when:
1. Conducted following a lawful arrest
2. For the purpose of discovering weapons or evidence related to the arrest
3. The police establish reasonable and probable grounds justifying the strip search (beyond
those required for the initial arrest)
4. It is conducted in a reasonable manner
Facts: Strip search took place in Subway restaurant where suspect was bent over table and
cleaning gloves were used to do a cavity search.
Issues: Are strip searches allowed as part of the common law police power to search incident
to arrest?
Decision: No. Strip searches are not included in the police power to search incident to arrest.
Reasons: Arbour (5to4) Strip searches are inherently humiliating, more intrusive and
dangerous than other searches and therefore require additional reasonable grounds to justify
strip search.
Dissent: Bastarche Same requirements justifying conduct of a search incident to arrest
should apply for all circumstances, including strip searches. Categorizing searches on degree

24

of intrusion creates confusion. This decision bypasses Cloutier, which says that reasonable and
probably grounds are not prerequisite to police power to search.

INVESTIGATIVE DETENTION
Sections 9 and 10
9. Everyone has the right not to be arbitrarily detained or imprisoned.
10. Everyone has the right on arrest or detention
o (a) to be informed promptly of the reasons therefor;
o (b) to retain and instruct counsel without delay and to be informed of that right; and
o (c) to have the validity of the detention determined by way of habeas corpus and to
be released if the detention is not lawful.
before no common law authority before 1993 said arrested or free to go

History/Context

1. Was the person


detained? To
determine, apply the
Grant test. (applied in
Suberu)

If NO Detention

General Rule, Pre-Charter and Pre-Mann: Short of arrest, the


police never possessed legal authority at common law to detain
anyone against his or her will for questioning or to pursue an
investigation. (R. v. Dedman 1985)
Mann: Middle ground emerged bw liberty and arrest; CL police power
of investigative detention.
Clayton: General detention power when reasonably necessary in
circumstances.
Progression to more expansive and intrusive police powers has
taken place in Charter-era through expansive reading of the
common law rather than legislation. Generally weakening
protection for civil liberties.
Detention under ss. 9 and 10 of the Charter refers to a suspension
of the individual's liberty interest by a significant physical or
psychological restraint. (Grant)
TEST FOR PSYCHOLOGICAL DETENTION:
1. Psychological detention is established either where the individual
has a legal obligation to comply with the restrictive request or
demand, or a reasonable person would conclude by reason of the
state conduct that he or she had no choice but to comply.
Test: objective reasonable person
You are NOT detained when police are simply gathering information
or requesting assistance (Suberu)

2. Where no physical restraint or legal obligation, to determine


whether the reasonable person in the individual's circumstances would
conclude that he or she had been deprived by the state of the liberty
of choice, the court may consider:
a) The circumstances giving rise to the encounter as would
reasonably be perceived by the individual. (Consider: whether
the police were providing general assistance; maintaining general
order; making general inquiries regarding a particular occurrence; or,
singling out the individual for focussed investigation).
b) The nature of the police conduct (Consider: language used; the
use of physical contact; the place where the interaction occurred; the
presence of others; and the duration of the encounter).
c) The particular characteristics or circumstances of the
individual where relevant, (Consider: age; physical stature;
minority status; level of sophistication.)
No Charter breach

25

2. If DETENTION,
what are the police
entitled to do once a
person is detained?

1. Common Law Power of Investigative Detention (Mann)


Police may detain an individual for investigative purposes if there
are reasonable grounds to suspect on the totality of the
circumstances that the individual is connected to a particular
crime, and that such a detention is necessary.
Detention must be brief and no duty to answer questions of police.
2. Common Law Power to Search Incident to Investigative
Detention (Mann)
A search incident to investigative detention must be a protective
search:
a) only a pat-down search
b) must be a search for weapons in order to protect officer or
public safety
c) Only permissible when there are reasonable grounds to
believe that safety at risk
d) Must be conducted in a reasonable manner

3. If DETENTION,
What are the rights of
the detainee?

3. General Detention Power (Clayton)


Police have a common law power to detain an individual so long as
detention is reasonably necessary in the totality of the
circumstances.
To determine whether a detention is reasonably necessary,
consider:
1. The nature of the situation, including the seriousness of the
offence,
2. The information known to the police about the suspect or
the crime, and
3. The extent to which the detention was reasonably
responsive or tailored to these circumstances, including its
geographic and temporal scope.
Then balance the seriousness of the risk to public or individual
safety with the liberty interests of members of the public to
determine whether the nature of the detention is no more intrusive
than is reasonably necessary.
Charter
9. Everyone has the right not to be arbitrarily detained or imprisoned.
10. Everyone has the right on arrest or detention
(a) to be informed promptly of the reasons therefore;
(b) to retain and instruct counsel without delay and to be informed
of that right; and
(c) to have the validity of the detention determined by way of
habeas corpus and to be released if the detention is not lawful.

s. 7 includes the right to remain silent. It is a residual right included in


the principles of fundamental justice in s. 7. Related to the notion of
voluntariness (confessions) and principle against self incrimination
RIGHT NOT TO BE ARBITRARILY DETAINED (s. 9)
4. Did the detention
A detention w/o reasonable suspicious or reasonable necessity is
violate s. 9?
arbitrary and in violation of s. 9
RIGHT TO COUNSEL (s. 10(b))
5. Did the detention
Police must advise detainee of right to counsel (informational
violate s. 10(b)? (ie.
component) (Manninnen)

26

What obligations are


placed on the police
by s. 10(b)

Individuals have a right to counsel immediately upon being


detained (Suberu) (immediacy only subject to concerns for officer
or public safety.)
Police must give detainee reasonable opportunity to exercise right
to retain and instruct counsel (implementational component)
(Manninnen), including access to legal aid and duty counsel
(informational component) (Brydges)
Police must cease questioning until detainee given reasonable
opportunity to exercise right (Manninnen), or unequivocally waived
the right to counsel (Brydges)
S 10(b) does NOT require a lawyer to be present during
interrogation (Sinclair)
Police are only required to provide additional opportunities to
consult counsel when there are material changes in the
circumstances (e.g. additional charges, didnt understand waiving
right to counsel, new procedures) (Sinclair)
Must use reasonable diligence

6. Did the detainee

use reasonable
conduct in exercising
their rights?
RIGHT TO SILENCE (s. 7)
7. Did the detention
1. Police can carry on persuading and questioning a detainee
violate s. 7? What are
EVEN after they have explicitly stated they want to stand
the police entitled to
on their right to silence AS LONG as the questioning doesnt
do?
render them involuntary. (Singh)
2. Right to silence applies only after detention (Hebert)
3. The police can continue to question and persuade the accused
once he has consulted counsel (Hebert). They are not obligated to
provide access to counsel again unless there were material
changes in the circumstances (Sinclair)
4. The right does not affect voluntary statements made to fellow cellmates (Hebert)
Use of undercover agents to observe the accused in cells is okay as
long as police dont engage in active elicitation designed to undermine
the accuseds choice not to speak to police. (Hebert)
Was the statement
COMMON LAW CONFESSION RULE (Oickle)
rendered involuntary? If a statement of the accused is made to person of authority (i.e.
police), the Crown must prove beyond a reasonable doubt that the
statement was voluntary
Consider the totality of the circumstances
Confessions can be rendered involuntary because of:
1) Threats or promises that induce the confession (Ibrahim,
Boudreau)
2) An atmosphere of oppression that compels a suspect to speak in
order to bring ordeal to end (Hobbins) (deprivation of food, etc)
3) Where suspect lacked operating mind (Ward v. the Queen;
Horvath v. the Queen; R. v. Clarkson)
If the common law confessions rule is breached, it results in
the automatic exclusion of the statement
7. Was the detention
YES NO Charter breach
lawful?
NO Charter breach (either s. 7, 9 or 10)
8. If a Charter breach Apply Grant exclusion of evidence test.
was found, should the

27

evidence be excluded
under s. 24(2)?
R V SIMPSON 1993 Ont CA
Ratio: Police have authority for investigative detention with articulable cause to link person to
offence.
Facts: Police stopped a vehicle on basis that person was driving away from what they were
told was a crack house; Police stopped and searched Simpson and found crack.
Issues: Do police have authority to detain someone without arrest?
Decision: In this case, arrest was improper bc not enough grounds for detention. Police do
have authority to do investigative detention with articulable cause.
Reasons: Not enough articulable cause.
Precedent: Represented a profound change in Canadian law, which did not generally
authorize police to detain individuals short of arrest (even though that was the common
practice).
R V MANN 2004 SCC
Ratio:
1) Investigative Detention: There is a common law police power of investigative detention.
Police officers may detain an individual for investigative purposes if there are reasonable
grounds to suspect on the totality of the circumstances that the individual is connected
to a particular crime, and that such a detention is necessary. It must be brief and no duty to
answer questions of police.
2) Search Powers Incident to Investigative Detention: Police have authority to search
incident to investigative detention. It must be a protective search, meaning: a) only a pat
down search b) must be a search for weapons in order to protect office or public safety c)
Only permissible when there are reasonable grounds to believe that safety at risk d) Must
be done reasonably.
Facts: Police received call about break and enter with description of suspect. At scene, Mann
walking and stopped by police who said matched to a tee the description. Officer did a pat
down search and felt something soft in pocket. Officer reached in a pulled out a bag of
marijuana. In another pocket, found two valium.
Issues: Do police have authority for investigative detention? If so, do police have search
powers incident to investigative detention?
Decision: 1) Officers had reasonable grounds to detain Mann. 2) There were reasonable
grounds for a protective search of Mann. 3) Seizure of marijuana unlawful. Officer went too far
by reaching into Manns pocket. Evidence excluded under 24(2)/
Reasons: 1) He closely matched the description and was near the scene of crime which led
officers to reasonably suspect he was involved in the crime and should be investigated further.
2) Logical possibility that Mann, suspected of the crime, had B&E tools that might endanger
officer safety. On balance, pat down was justified. 3) Should have stopped when felt something
soft. No reason to believe safety was in question any longer and violated REP over pocket
contents.
Post-Mann: Decision received negative criticism. Filling a gap in police powers with common
law rather than leaving matter to parliament to address. No mention at all about way police
powers impact marginalized communities. Racial minorities and economically disadvantaged
are more frequently detained
R V CLAYTON 2007 SCC
Ratio: Police have a common law power to detain an individual so long as detention is
reasonably necessary based on the totality of the circumstances. Deciding whether a
detention is reasonably necessary in given circumstances will depend on factors like:
1. The nature of the situation, including the seriousness of the offence,

28

2. the information known to the police about the suspect or the crime, and
3. the extent to which the detention was reasonably responsive or tailored to these
circumstances, including its geographic and temporal scope.
All of which involves balancing the seriousness of the risk to public or individual safety with
the liberty interests of members of the public to determine whether the nature of the stop is
no more intrusive than is reasonably necessary.

Confirms that, as established in Mann, searches will only be permitted where the
officer believes that his or her safety, or that of others, is at risk.

Facts: Police set up vehicular blockade minutes after receiving 911 call from a strip club that
10 black males were outside and 4 had guns. Caller identified four vehicles. When a vehicle
attempted to leave by rear exit (not one of the vehicles described in the call), police stopped it
stopping every car even if it didnt match. Farmer driving and Clayton passenger. Police
asked men to get out of car bc suspicious, esp bc passenger was wearing leather gloves and it
was a warm day. When Clayton got out of the car, he ran. Police caught him and found a
loaded gun. Upon searching car, found a second firearm. Clayton argued that police had no
authority to make the initial stop. Unlike in Mann, the police were unable to say that people
detained closely matched the description of the perpetrators.
Issues: Do the police have a common law power to conduct this kind of investigative
detention?
Decision: Yes. Police have a common law power to detain an individual so long as detention is
reasonably necessary in the totality of the circumstances.
Reasons: There is a common law police power to conduct criminal investigative roadblock
stops of vehicles and their occupants. Such roadblocks must be tailored to the information
possessed by police, seriousness of the offence being investigated, and the temporal and
geographic connection bw the situation being investigated and the timing and location of the
roadblock.
Dissent: Mann required a clear nexus to crime, so this extension should be linked to this
specific set of facts around vehicular blockades to find firearms in a certain location.
Precedent: Police powers in Mann are supplemented with a general detention power; even
broader backdrop to Mann, which gives clear authority in those circumstances
R V. GRANT 2009 SCC
Ratio:
1. Detention under ss. 9 and 10 of the Charter refers to a suspension of the individual's
liberty interest by a significant physical or psychological restraint. Psychological detention is
established either where the individual has a legal obligation to comply with the restrictive
request or demand, or a reasonable person would conclude by reason of the state conduct
that he or she had no choice but to comply.
2. In cases where there is no physical restraint or legal obligation, it may not be clear whether
a person has been detained. To determine whether the reasonable person in the individual's
circumstances would conclude that he or she had been deprived by the state of the liberty of
choice, the court may consider:
a) The circumstances giving rise to the encounter as would reasonably be perceived by the
individual: whether the police were providing general assistance; maintaining general
order; making general inquiries regarding a particular occurrence; or, singling out the
individual for focussed investigation.
b) The nature of the police conduct, including the language used; the use of physical
contact; the place where the interaction occurred; the presence of others; and the
duration of the encounter.
c) The particular characteristics or circumstances of the individual where relevant,
including age; physical stature; minority status; level of sophistication.
Facts: Police patrolling near Toronto school. Grant, a young, black man was walking down the

29

street and came to attention of two plainclothes officers. They say him fidgeting with jacket
and told a third, uniformed officer to chat with him on the street. He was asked for name and
address and after adjusting his jacket, asked to keep hands in front. Two additional officers
came back. When Grant was asked if he had anything he shouldnt, he replied a small bag of
weed and a firearm.
Issues: Were Grants s. 9 rights violated? When is a person considered detained for
purposes of the Charter?
Decision: Yes. Grant was detained w/o reasonable grounds and therefore it violated his s. 9
rights.
Reasons: McLaughlin -- Grant was detained when he was told to keep his hands in front of
him. There were no legal grounds to detain him and therefore it violated his s. 9 rights.
Dissent: Binnie -- Worries that with the majority approach, encounters with police that
average citizens would consider left them with no choice but to comply are denied the status
of detentions through the device of putting in their place an artificially robust and assertive
reasonable person. Creating a reasonable person who is assertive, unfearful of police and
knowledgeable of their rights. Important for visible minorities who may feel especially unable
to assert rights to police. Says the focus should be on what the police were intending rather
than what the reasonable person perceived to be happening.
R V SUBERU
Ratio: From the moment someone is detained, section 10(b) is engaged and the police have
an obligation to inform the detainee of right to counsel. Immediacy only subject to concerns
for officer or public safety. Once engaged, s 10(b) imposes informational duty (must inform
detainee of right) and implementational (must provide detainee with reasonable opportunity
to retain and instruct counsel). Also requires police to refrain from eliciting incriminatory
evidence until s/he has a reasonable opportunity to reach a lawyer, are unequivocally waived
right to do so.
Facts: Suberu and friend shopping using stolen credit card. LCBO employee, having been
alerted, called police. Suberu walked past police officer and said he did this, not me, so I
guess I can go. Officer followed Suberu outside and questioned him. Officer determined he
had reasonable and probable grounds for arrest after seeing shopping bags in back of car.
Upon arrest, gave him his rights, including right to counsel. Issues is whether right shouldve
been given at outset of interaction.
Issues: Was Suberus s 10(b) rights violated by failing to inform him of the right upon
detention? Is the police duty to inform an individual of right to retain and instruct counsel
under s. 10(b) of the Charter triggered at the outset of investigative detention?
Decision: 1) Suberus s 10(b) right was not violated. Yes. From the moment someone is
detained, section 10(b) is engaged and the police have an obligation to inform the detainee of
right to counsel.
Reasons: McLaughlin 1) No detention occurred when officer spoke to Suberu in van. Applied
Grant detention analysis. A) Circumstances giving rise to encounter: Officer was trying to
figure out if Suberu was involved rather thank] deprive him of liberty. B) Police conduct: He
was doing preliminary investigative questioning that fell short of detention. C) Individuals
Personal Circumstances: No evidence that Suberu believed he couldnt leave or evidence of
his emotional or personal circumstances. Therefore no detention and his right to counsel was
not yet triggered. 2) Concerns about compelling self-incrimination and interference with liberty
that 10(b) seek to protect are present as soon as detention occurs without delay means
immediately
Dissent: Binnie Majority interpretation requires court to underestimate coercive powers of
police and over estimate resilience of Cdn population in face of such commands. Would have
found on Grant detention test that Suberu was detained. Police mean what they say when
they tell someone to stay put thus Suberu correctly assumed that he had no choice but to
comply. Police were not doing general questioning but responding to a specific incident.
Would have found detention arbitrary and violation of s. 9

30

VII. CONFESSIONS, RIGHT TO COUNSEL AND RIGHT TO


SILENCE
A. CONFESSIONS
Common Law
Confessions Rule
(Oickle)

If a statement of the accused is made to person of authority (i.e.


police), the Crown must prove beyond a reasonable doubt that the
statement was voluntary
Consider the totality of the circumstances

Confessions can be rendered involuntary because of:


1) Threats or promises that induce the confession (Ibrahim,
Boudreau)
2) An atmosphere of oppression that compels a suspect to speak in
order to bring ordeal to end (Hobbins) (deprivation of food, etc)
3) Where suspect lacked operating mind when speaking (ie.
serious car accident -- Ward v. the Queen; put in a hypnotic state
by police -- Horvath v. the Queen; intoxicated -- R. v. Clarkson;
4) Where police engaged in trickery that is shocking to the
community.
If the common law confessions rule is breached, it results in the automatic
exclusion of the evidence.
Before a court will conclude that common law confession rule has been breached, the
defence will have to show that there has been HUGE interference on the part of the police;
very high standard
o

The burden is on the Crown to prove, beyond a reasonable doubt, that the statement was
voluntarily made

BOUDREAU V THE KING 1949 SCC


Ratio: Fundamental question about admissibility of a confession made to a person in authority
(ie. police) is whether it is voluntary
Voluntariness Evolves:
Threats, promises, inducements (Ibrahim we cant trust if think on behalf of state,
induced threatened or made promises, Boudreau)
Oppression (Hobbins)- circumstantially depriving of food, etc.; SCC says we wonder if
statement holds up
Operating Mind
o Ward v. the Queen - accused in state of shock after car accident; cant make
voluntary statement
o Horvath v. the Queen -accused integrated after police hypnosis o R. v. Clarkson- in orbiter said could be fairness
o R. v. Whittle - not full appreciation for consequences, but you know what saying
and understand could be used against you- not just knowing reliability, but could
be used against you also about fairness notion of voluntariness broadened

R V OICKLE 2000 SCC - the Modern Rule


Ratio: Common law confessions rules extends to protect a broader conception of
voluntariness that focuses on protection of accuseds rights and fairness in the criminal
process. The totality of the circumstances must be considered in deciding whether the Crown
established beyond a reasonable doubt that suspect made a voluntary statement to police.

31

Confessions can be rendered involuntary: 1) because of threats or promises that induce the
confession 2) due to an atmosphere of oppression that compels a suspect to speak in order to
bring ordeal to end 3) where suspect lacked operating mind when speaking 4) where police
engaged in trickery that is shocking to the community.
Facts: Oickle given Charter rights but many thought police had exerted oppressive
circumstances encouraged him, promised not to subject fianc to polygraph
Issues: How high is the standard for involuntariness? How much is required before will is
overborne?
Decision: This did not beach CLCR.
Reasons: For statements made by an accused to a person in authority to be admissible, the
Crown must establish, beyond a reasonable doubt, that the statement was voluntary in the
reasonable doubt, that the statement was voluntary in the sense that it was not the product of
a will overborne by threats, promises, or inducements; by oppressive circumstances, or the
lack of an operating mind.
R V SPENCER 2007
Ratio: standard for involuntariness - influence must be so overbearing that it can only be said
that the detainee has lost any meaningful independent ability to choose to remain silent, and
has become a mere tool in the hands of the police.
Facts: Accused and his girlfriend arrested Accused confesses to robberies after requesting
lenient treatment for girlfriend and being permitted to visit with her Whether statements
voluntary
Issues: whether the accused had an effective choice and whether his will was overborne
Decision: Voluntary used Oickle
Reasons: A promise renders a statement involuntary only if the quid pro quo provides a
strong enough inducement to raise a reasonable doubt about whether the will of the suspect
was overborne. Accordingly, while a quid pro quo is an important factor in establishing the
existence of a threat or promise, it is not by itself determinative. It is the strength of the
inducement, having regard to the particular individual and his or her circumstances, that is to
be considered in the overall contextual analysis into the voluntariness of the accuseds
statement.
Fish J - the will of the detainee is said to have been overborne only in the sense that he or
she would not otherwise have given a statement but was persuaded to do so in order to
achieve an expected result to avoid threatened pain or achieve promised gain. A statement
thus given is the result of a calculated decision by an operating mind; it is nonetheless
considered involuntary (para. 32) dissent - worried about not GRANT Grant is remedy
for beach of right this is common law if we breach right to counsel or silence may or may
not be excluded bc have to go to 24(2); if you breach CLCR, it is OUT appealing bc 1. burden
is on Crown, whereas right breach is on applicant; 2. if cant show voluntary, no more analysis
struck; at common law only statement is excluded, not stuff found bc of statement here
youre better of going to charter if concerned about derivative

R V CLARKSON 1986 SCC


Ratio: Operating mind analysis can be cast narrowly (understand own words) or broadly
(understand potential use and consequences of statement). Trial judge must balance probative
value of statement and prejudice of accused. Police cant question until right to consult
counsel
Facts: Clarkson phoned sister saying she shot her own husband. Police charged her, and took
her to station for questioning in presence of her aunt. She had blood alcohol level
210mg/100ml. She ignored her right to counsel and even though aunt interjected several
times, questioning continued until she had effectively confessed.

32

Issues: Is a confession given by an intoxicated person and without benefit of counsel


admissible?
Decision: Confession was improperly obtained and in violation of s. 10(b). Evidence excluded.
Reasons: Wilson -- Operating mind could be cast two ways: narrow, which is whether you
understand your own words or broadly, which is full conscious awareness about statements
potential use and consequences. Judge must balance tension bw probative value of confession
and prejudice to accused. Case dealt with on s 10(b) issue. Accused did not waive her right to
counsel with a full awareness of the consequences. Police should have at least delayed
questioning.

B. RIGHT TO COUNSEL
Charter s 10(b)
Components of
right (Sinclair)
What obligations
are placed on the
police by 10(b)?

What obligations
are on the
accused?
Can you waive
your right to
counsel?
Can the accused
choose counsel?
Can you have
access to legal aid
for counsel?

Everyone has the right on arrest or detentionto retain and instruct


counsel without delay and to be informed of that right.
1. Informational Detainee must be advised of right to counsel
2. Implementational Detainee must be given reasonable opportunity to
exercise right to retain and instruct counsel.
Police must advise detainee of right to counsel (informational
component) (Manninnen)
Individuals have a right to counsel immediately upon being detained
(Suberu) (immediacy only subject to concerns for officer or public
safety.)
Police must give detainee reasonable opportunity to exercise right to
retain and instruct counsel (implementational component)
(Manninnen), including access to legal aid and duty counsel
(informational component) (Brydges)
Police must cease questioning until detainee given reasonable
opportunity to exercise right (Manninnen), or unequivocally waived
the right to counsel (high standard) (Brydges)
S 10(b) does NOT require a lawyer to be present during interrogation
(Sinclair)
Police are only required to provide additional opportunities to consult
counsel when there are material changes in the circumstances (e.g.
additional charges, didnt understand waiving right to counsel, new
procedures) (Sinclair)
Detainee must exercise reasonable conduct in exercising their rights
reasonable diligence
Yes, you can waive the right to counsel but the threshold is very high. In
waiving the right, must have a full and voluntary waiver in full awareness
of the consequences (Brydges)
Yes. Only if chosen counsel not available in reasonable amount of time
should they be expected to call another lawyer
Yes. Part of informational component includes access to legal aid and
duty counsel (Brydges) (e.g. Brydges Line 24hr phone line)

R v MANNINNEN [1987]
Ratio: Section 10(b) imposes following duties on police: 1) Informational police must inform
detain of right to counsel and 2) Implementational police much provide detainee with a
reasonable opportunity to exercise the right to retain and instruct counsel without delay. 3)
Duty to cease questioning or attempting to elicit evidence from detainee until given a
reasonable opportunity to retain and instruct counsel.

33

Facts: Two plain clothes officers drove to site acting on information given two days after a
robbery. Manninnen arrived in what was described as stolen car used armed robbery. Accused
was arrested and told his rights. Said he wasnt saying anything until he saw his lawyer. Police
continued to question and accused said enough for conviction. There was a telephone in the
room and police did not offer for accused to use it.
Issues: Was Manninens s 10(b) right violated? What duties are imposed on the police by s.
10(b)
Decision: Yes. Violation of s. 10(b). Conviction quashed, evidence excluded and new trial
ordered.
Reasons: Police had a duty to provide opportunity to exercise right to counsel and didnt offer
him a phone even though one was available. Detainee doesnt need to explicitly ask. Police
should have ceased questioning until opportunity to exercise right was given.
R v BRYDGES 1990 SCC
Ratio: Access to Legal Aid or duty counsel is part of the informational component of s. 10(b)
right to counsel. Police must tell detainees about existence of such services and hold off
questioning until they have had reasonable opportunity to reach counsel. A detainee may,
either explicitly or implicitly waive their right to counsel, but the standard will be very high
where alleged waiver is implicit.
Facts: Accused charged with second-degree murder and asked if they had any free legal aid
available bc wouldnt be able to afford a lawyer. Police asked if there was a reason for wanting
to talk to a lawyer and he replied no. Accused then continued to answer questions and gave
prejudicial statements until he eventually asked for a lawyer and was given one.
Issues: Was Brydges waiving his right to counsel when he first said that he didnt need to
speak to a lawyer?
Decision: No. Brydges did not waive his right to retain counsel under s. 10(b).
Reasons: The standard for waiving the right to counsel is very high. The failure of the police
to inform accused about the existence of legal aid or duty counsel was a restriction on his right
to counsel
Precedent: Following case, Ontario set up 24 hr Brydges Line so detainees can contact duty
counsel or counsel paid by legal aid.
R v PROSPER 1994 SCC
Ratio: S. 10(b) does not impose a positive obligation on govts to fund duty counsel programs
for those detained outside regular business hours. However, where detainee is prevented from
exercising 10(b) right bc of absence of duty counsel program, 10(b) requires police to hold
questioning until detainee has a reasonable opportunity to reach counsel.

B. RIGHT TO SILENCE
Charter s. 7

Limits on right to
silence (What are
the police entitled
to do?)

Everyone has the right to life, liberty and security of the person.
A residual right to silence is included under s. 7 in PFJ (Hebert) -- 1)
notion of voluntariness and 2) principle against self incrimination
A person in the course of the criminal process has the right to choose
whether to speak to the police or remain silent
1. Police can carry on persuading and questioning a detainee
EVEN after they have explicitly stated they want to stand on
their right to silence AS LONG as the questioning doesnt
render them involuntary. (Singh)
2. Right to silence applies only after detention (Hebert)
3. The police can continue to question and persuade the accused once
he has consulted counsel (Hebert). They are not obligated to provide
access to counsel again unless there were material changes in the
circumstances (Sinclair)

34

Did the
questioning render
the accused
involuntary?

4. The right does not affect voluntary statements made to fellow cellmates (Hebert)
5. Use of undercover agents to observe the accused in cells is okay as
long as police dont engage in active elicitation designed to
undermine the accuseds choice not to speak to police. (Hebert)
Determined by the Common Law Confession Rule (Oickle)
If a statement of the accused is made to person of authority (i.e. police),
the Crown must prove beyond a reasonable doubt that the statement
was voluntary
Consider the totality of the circumstances
Confessions can be rendered involuntary because of:
1) Threats or promises that induce the confession (Ibrahim,
Boudreau)
2) An atmosphere of oppression that compels a suspect to speak in
order to bring ordeal to end (Hobbins) (deprivation of food, etc)
3) Where suspect lacked operating mind (Ward v. the Queen;
Horvath v. the Queen; R. v. Clarkson)
If the common law confessions rule is breached, it results in
the automatic exclusion of the statement

R v HEBERT 1990 SCC


Ratio: Right to silence is a principle of fundamental justice and therefore protected under s. 7
of the Charter. Once in police custody, accuseds right to silence cannot be undermined
through trickery. However, if accused volunteers information to undercover officers of own free
will, statements could be used against them.
Facts: Accused arrested for robbery and after speaking with counsel advised police he did not
wish to make a statement. Put in holding cell with undercover police officer who engaged in
questioning of accused. Accused made incriminating statements that implicated him in the
robbery.
Issues: Do statements made by detainees to undercover police violate the accuseds right to
silence?
Decision: Yes, but with certain limitations. Here, violation of s. 7 was found and statements
excluded.
Reasons: Essence of right to silence of choosing whether or not to speak to police. Accused
clearly chose not to and when he later spoke to undercover officer, was not reversing that
choice.
R v SINGH 2007 SCC
Ratio: Police can continue to persuade detainees to break silence after they have (repeatedly)
exerted their right to silence under s. 7 as long as they dont cross a line which renders the
statements involuntary. Main issue is whether accused used free will in choosing to make a
statement.
Facts: Accused charged with murder after killing innocent person in gun fight. Evidence
included testimony of doorman and fuzzy photograph. Police arrested Singh, gave Charter
rights and he spoke with counsel. Police began questioning and he said he knew nothing,
didnt want to talk and wanted to go back to cell. Officer would say you dont need to speak
but I have obligation to put evidence before you This happened 18 times. After 18 th time he
finally admitted that it was him in the photo. Accused concedes voluntary confession, but
undermined right to silence.
Issues: What is the scope of a detainees pre-trial right to silence under s. 7? How do Hebert
and common law confessions rule (Oickle) intersect? If you insist on silence, do police have to
obey you?
Decision: Dismissed Singhs appeal. The police questioning did not deprive him of his free will

35

to choose whether or not to speak and therefore did not violate his right to silence under s. 7.
Section 7 does not require the police to refrain from using reasonable persuasion to encourage
detainees to speak.
Reasons: Charron -- Provided that the detainees rights are adequately protected, including
the freedom to choose whether to speak or not, it is in societys interest that the police
attempt to tap this valuable resource. (para. 45) The ultimate question is whether the
accused exercised free will by choosing to make a statement.
Dissent: Fish -- A right that need not be respected after it has been firmly and unequivocally
asserted any number of times is a constitutional promise that has not been kept. (para. 70);
more draconian initiatives might prove more effective still; Charter must go further than
CL confession rule. If you say you want to stand on your right to silent, police should have to
respect that.
Precedent: Oickle & Hebert intersection -- If Crown proves beyond a reasonable doubt that
statement is voluntary, there can be no violation of s. 7 right to silence. Must balance freedom
to speak or not, and societys interest in solving crime. The majority found
that Oickle enhances, rather than subsumes, the s. 7 right to silence; at common law, the
protection afforded by the confessions rule also guards against the Charter concern of
potential state abuse over a detained suspect.
R v SINCLAIR 2010 SCC
Ratio: 1) Right to counsel under s. 10(b) does not require lawyer to be present throughout
interrogation by police 2) Where a detainee has already received legal advice prior to police
interrogation, s. 10(b) only requires the police to provide a reasonable opportunity to consult a
lawyer again if there are material changes in the circumstances. Material changes include: a)
new procedures involving detainee (e.g line up) b) change in jeopardy (e.g new charges) c)
indication that a detainee who initially waived right to counsel may not have understood that
right.
Facts: Sinclair arrested for murder and upon arrest, declined his right to speak with counsel.
When asked again, he accepted and spoke with lawyer for 3 min. Police interrogated him for 5
hours during which he expressed discomfort about being questioned w/o presence of lawyer
and wanted to speak to lawyer again. Police told him he had the choice as to whether or not to
speak and refused to let him see lawyer again.
Issues: Are police required to give detainees access to counsel beyond the initial consultation
with duty counsel?
Decision: No. Police were not in violation of his rights by not allowing another consultation
with lawyer.
Reasons: McLachlin and Charron Unless there is a material change in the detainees
situation, the initial consultation with the lawyer satisfies s. 10(b). Section 10(b) does not
mandate presence of defence counsel during interrogation. Section 10(b) should be
interpreted in a way that fully respects its purpose of supporting s. 7 right to silence this is
achieve through initial lawyer consultation which gives detainee enough information to choose
whether or not to speak with police.
Dissent: Binnie Police should provide reasonable access to counsel from time to time as
circumstances evolve. Would have found a breach of 10(b). - You have reached counsel. Keep
your mouth shut. Press one to repeat this message. (para. 86)
LeBel and Fish Detainees should have right to counsel whenever they want on request.
People who are not detained have that right, so no logical grounds for saying that those
detained should have less access.

TRIAL PROCESS AND ROLE OF COUNSEL


Wrongful

Criminal justice system is compulsively worried about error. Evidence:

36

Convictio
ns
(Marshall)

Racial
Bias

Presence of appellate mechanisms. Criminal cases where they is a


dissent at the appeal level automatically leave to appeal to SCC.
o Presumption of innocence
o Burden of proof of beyond a reasonable doubt (Lifchus)
o Obligation to give reasons
When there is a wrongful conviction, the criminal justice system has not only
failed to address that wrong and bring order to situation, but compounded
victimhood with victimhood -- suffering at hands of state.
Aboriginal ppl & the justice system
Ontario report
Systemic discrimination is best addressed through increased representation of
black communities in the criminal justice system.
Manitoba report (trial of Helen Betty Osborne)
Traditional legal system is not appropriate for resolution of conflict for Ab.
Argued for a hiving off criminal justice Aboriginal courts, police, juries;
applying own version of their law.
Self determination that is at the root of being able to address systemic
discrimination
o

USA V BURNS AND RAFAY


Ratio: Extradition to face the death penalty is a violation of s. 7.
Facts: Murder in Washington state after which both fled to Canada. Faced extradition and
death penalty if sent back to Washington State.
Issues: Is it constitutional to extradite someone where the consequence is the death penality.
Decision: No. Under s. 7 one can no longer be extradited to face the death penalty. Must get
assurances from state that person will not be executed.
Reasons: Legal systems have to live with the possibility of error. (1 st sent of decision).
There is extensive evidence of wrongful convictions (Sophonow, Truscott, Milgaard, etc)
despite all of the safeguards in place to protect the innocent.
R V MARSHALL
Ratio: Marshall Inquiry showed that outcomes of criminal justice system turn on small,
mundane decisions. Criminal justice system is fragile and mistakes compound over time,
which can lead to these terrible outcomes. Major lesson from inquiry was that much of these
issues resulted from systemic discrimination and bias against Marshall, an Aboriginal person.
Facts: Marshall, Aboriginal man, charged and convicted with stabbing death of Sandy Seale.
Tried and convicted and appeal dismissed.
Decision: Marshall ultimately acquitted and an inquiry held.
Reasons: Police presumed guilt of Marshall, they didnt investigate witnesses, had tunnel
vision; Crown didnt fully disclose all the information they had, including inconsistent
statements; shaped the testimony of witnesses; Defence counsel didnt do anything to defend
Marshall bc they felt he was guilty; Appeal judge didnt point out error of law (they tried to
blame Marshall for not telling what happened), trial judge made evidentiary rulings that were
incorrect; Other witnesses not pushed in the way they should have been.
bias institutional bias and systemic/structural bias- inequalities deep rooted in system,
exclusion of voices and ppl; also speaks to intersectionality Aboriginal women specifically
(Manitoba article); in Ontario, response is more integration judges, jury; from Manitoba
self determination
R V S.(R.D)
Facts: 15yr old black youth charged with assaulting a white police officer. He allegedly
interfered with arrest of other youth. Trial judge said that police officers had been known to

37

overeat with regard to non-white groups. Crown alleged reasonable apprehension of bias
against J Sparks
Decision: Acquittal restored. No reasonable apprehension of bias from trial judge.
Reasons: McLachlin (+3) Comments reflected the context of the case, which was well
known to the judge and any informed community member. (Iacobucci, Cory) -- Comments
were regrettable but do not meet the standard of bias. Majority (6): decision was justified
Dissent: Lamer, SopinkaStereotyping police not helpful for progress; conclusions not based
on evidence.
Role of Crown
Counsel

Role of
Defence
Counsel
Plea Bargains

Crown prosecutors are ministers of justice assisting in its administration,


not advocates. They have a pseudo-public role. Obligation is not to secure
a conviction, but to present the evidence firmly and fairly and if the
evidence shows that they dont win the case, it should be considered they
have done job effectively. (Boucher)
Crown counsel have an obligation to disclose information obtained through
investigation to the defence. Failure to disclose is a breach of accuseds
right to full answer and defence under s. 7. (Stinchcombe)
Not a minister of justice, a fearless advocate.
No obligation to assist the Crown or disclose evidence.
Obligations flow from being an officer of the court (ie. LSUC rules)
Reduced sentence for admission of guilt. Offer at Crowns discretion.
Accounts for 70-90% of the way that cases get resolved in cjs.
Cant run the system w/o them already buckling under pressure of 20%
cases in court

ROLES OF CROWN COUNSEL


BOUCHER V THE QUEEN
Ratio: Authority on role of Crown counsel in a criminal trial. Crown prosecutors are ministers
of justice assisting in its administration, not advocates. They have a pseudo-public role.
Obligation is not to secure a conviction, but to present the evidence firmly and fairly and if the
evidence shows that they dont win the case, it should be considered they have done job
effectively.
R V STINCHCOMBE
Ratio: Crown counsel have an obligation to disclose information obtained through
investigation to the defence. Failure to disclose is a breach of accuseds right to full answer
and defence under s. 7.
Facts: The Crown interviewed a witness but then decided not to get person to testify so they
wouldnt disclose the statements to the defence. Defence challenged that it was a violation of
accuseds Charter rights.
Decision: Crown counsel have a duty to disclose.
Reasons: Sopinka Fruits of investigation are not the Crowns property to secure a conviction,
but the property of the public to be used to ensure that justice is done. The right to make a full
answer and defence is a PFJ so there is a duty to disclose under s. 7 and arguably under s.
10(d). Includes the disclosure of all relevant, non-privileged information, including evidence
that Crown wants to rely on and evidence that it doesnt. Exception: Evidence that is
privileged(e.g. informer privilege, solicitor-client privilege).
Precedent: Prior to Stinchcombe, disclosure as a matter of sharing the evidence by Crown
with defence was ad hoc, voluntary and inconsistent. Might be the biggest change to cjs since
Charter

38

CRIMINAL LIABILITY
actus non facit reum, nisi mens sit rea - an act does not make a person guilty unless his mind
(or intention) is also guilty
Criminal Liability AR + MR Defences = CL
1. Actus reus (physically voluntary act or omission)
2. Mens rea (guilty mind; fault element)
3. Absence of any defences
Affirmative defences (have AR & MR but shouldnt be liable. e.g. self
defence)
Negative defences (mistake of fact - did not have MR e.g. intoxication)
Exemptions to criminal liability (e.g. children, mental disabilities)
Crown bears the burden of est. AR and MR beyond a reasonable doubt.
Principle of
AR and MR must occur at the same time.
Contemporaneit Rule is not applied strictly and relaxed where it would produce absurd
y
results. (Fagan v commissioner of metropolitan police, Miller, Cooper,
Meli)
Principle of contemporaneity doesnt require complete coincidence of AR
and MR as long they are part of a continuous transaction. Determination
depends on the nature of the act. (Cooper).
Defining
To determine the definition of an offence, look to:
Offences
1. Statute does criminal code define term?
2. Common Law what have courts said?
If neither of those, then you will need to engage in statutory
interpretation. (e.g. What does obscene mean?)
Actus Reus -- look at crime itself, interpretation portion and/or the
definitions section at beginning, section 2
Status Offences
Crimes punished with no act or omission. Punished for a status or for
being a certain way. Have fallen into disrepute and are not a
meaningful part of our contemporary criminal law. Now, if you found a
true status offence, it would likely be unconstitutional under s. 7.

ACTUS REUS
Elements of AR

1. Voluntariness

2. Omissions

1.
2.
3.
4.

A physically voluntary
Act or omission;
Sometimes in certain circumstances;
And sometimes causing certain consequences.
Physical voluntariness must be element of Actus Reus of any offence
(Larssoneur). Key issue is autonomy. author of own misfortune
Punishing involuntary actions is unjust because it conflicts with the
assumption in criminal law that individuals are autonomous and freely
choosing agents (Ruzic).
A person will not be criminally liable for failing to act unless he or she is
under a legal duty to act. Two basic circumstances:
1. Specific Omission Offences -- Where the offence specifies and
criminalizes a particular omission to exercise a legal duty.
S. 50(b) failure to report high treason
S. 127 failure to obey court order
S. 129(b) omitting to assist a police officer when requested
S. 252(1) failing to stop and assist after involved in accident
S. 254(5) failing to provide breath sample.
2. General Omission Offences -- Where offence states generally that its

39

predicated on breach or failure to preform a legal duty.


S. 180 common nuisance
S. 219 criminal negligence (causing death or bodily harm)
Fails to discharge legal duty or duty to do is a general duty to act
that creates obligations. Must then find a legal duty to plug into the
general placeholder.
Possible sources of duties:
1. Statutory Law (Browne, Thornton)
2. Provincial Statute (e.g. In QC, duty to help someone if wont be harmed)
3. Common Law (Coyne, Popen, Nixon, Thornton)
Duty imposed by common law could snap in to common nuisance
or criminal negligence.

3.
Circumstances

4.
Consequences

Causation
Factual Cause:
Legal Causation:

Is the crime first


degree murder
under s. 231(5)?
Smithers Test for
Legal Causation
(affirmed in
Cribbin)

Duties that ground liability for general omission offences (imposed


by Criminal Code)
S. 79 DOC handling explosives
S. 215(1) Duty to provide necessities of life
S. 216 DOC for those who undertake to administer surgical or medical
treatment or other lawful act that may endanger lives of others to have
and use reasonable knowledge, skill and care in doing so. (Thornton)
S. 217 Duty for those who undertake an act to actually do it, if
omission can be dangerous to life. (Browne)
S. 217.1 Duty of those directing work to take reasonable steps prevent
bodily harm
Some acts require that certain circumstances exist
Circumstances in actus reus are the things that turn otherwise noncriminal or less serious conduct into criminal or more serious criminal
offences. (e.g. consent is the lynchpin of assault)
For some offenses, the actus reus requires causing of certain
consequences: assault causing bodily harm (267), crim negl causing
harm/death (220/221), all homicides (222, 224-228), wilful damage to
property (430), arson (433), dangerous operation of a vehicle causing
bodily harm/death (249 (3)(4)), and impaired driving (255(2)(3)).
As part of the Actus Reus, Crown will have to prove the consequence
beyond a reasonable doubt.
Was there a causal link between the accuseds actions and the prescribed
consequence? (Winning)
Asks whether the factual cause is significant enough to justify criminal
liability. legal causation is a narrowing concept which funnels a wider
range of factual causes into those which are sufficiently connected to a
harm to warrant legal responsibility. (Maybin, para. 16)
Yes Harbottle causation
Not Smithers causation
Crown must establish that the accuseds actions are a
contributing cause of death, outside the de minimus range. (ie.
whether or not the accuseds actions were a non-trivial or not
insignificant cause of the prescribed result. )
Thin skull rule applies (Smithers)
significant contributing cause rather than non trivial cause, not

40

insignificant or de minimis can be used to mean the same (Nette).


Harbottle Test
for Legal
Causation

Multiple
Contributg
Causes
Intervening
Causes

In first degree murder cases under s. 231(5) and other sections where
the language caused by that person is used, legal causation test is:
the actions of the accused must form an essential, substantial, and
integral part of the killing of the victim (Harbottle)
Will usually involve actual physical involvement in the killing (Harbottle).
The higher standard used for first degree murder under 231(5) and
similar language ONLY. Otherwise, Smithers standard applies (Nette).
Multiple causes are not a problem. All that matters is if the accuseds
actions were a contributing cause to the outcome. Smithers standard
still applies.
Generally where actions of intervener are a direct result of the accuseds
actions, it will not sever the chain of action. It is only where the
accuseds actions do not precipitate the direct cause. (Pagett pregnant
gf, Blaue JW blood transfusion)
TEST:
Were the actions of the accused a significant contributing outcome?
(Maybin) Two approaches for analyzing the intervening cause:
a) Reasonable foreseeability: Was the general nature of the
intervening act reasonably foreseeable? (Maybin)
b) Independence: Was the intervening act sufficiently
independent and overwhelming to sever the impact of the
accuseds actions? (Maybin)

CONTEMPORANEITY
Fagan v. Commissioner of Metropolitan Police (1969)
Ratio: Not always necessary for guilty act and the intent to be completely concurrent.
Principle of contemporaneity can be massaged to create a fair result.
Facts: Fagan accused of assaulting police officer after accidently driving onto his foot and
then not moving the car had intent after driven onto foot Fagan argues cant be convicted bc
of principle of contemporaneity. Crown generates continuing act theory assault carried on
and mens rea attached when came in
Decision: Guilty. Assault continued and the MR attached to assault, therefore making it
punishable. Events werent discreet enough to violate principle of contemporaneity.
R. v. Miller (HL 1982)
Ratio: Unintentional act followed by intentional omission to rectify can be regarded in total as
an intentional act. Does not violate the principle of contemporaneity.
Facts: Smoking while lying down on mattress. Falls asleep and cig falls onto mattress. He sees
it but does nothing, goes to another room and goes back to sleep. Charged with arson.
Issues: Was it a continuous act?
Decision: Guilty.
Reasons: Dorothys Dog applies unintentional act followed by intentional omission to rectify
can be regarded in total as an intentional act. It should be read as all part of the same
behaviour. HL also said that you could deal with this as an omission problem (in addition to
coincidence)
R. v. Cooper (1993 SCC) adopts contemporaneity
Ratio: Principle of contemporaneity doesnt require complete coincidence of AR and MR as
long they are part of a continuous transaction. Determination depends on the nature of the
act.

41

Facts: Accused charged with murder for strangling a woman. He remembers getting angry,
grabbing her by neck and intending to do physical harm. Then testified he lost consciousness
and didnt recall causing her death black out, no mind.
R. v. Meli (1954)
Ratio: AR and MR for murder coincided as part of a continuing transaction.
Facts: Meli lured a man, beat him and threw him off a cliff, thinking he was dead. Autopsy
showed that he died of exposure over cliff death came after.
Issues: Were the AR and the MR for murder contemporaneous?
Decision: Guilty of murder.
Reasons: This was a continuing transaction where he intended to kill and dispose of the
person. (Could also have used causation theory ultimately caused the death of the victim.)

VOLUNTARINESS -

physical voluntariness requires low standard of will this is


principle of fundamental justice; cant be deprived of liberty unless we know act was
voluntary
Oliver Wendell Holmes in The Common Law (1881):
The reason for requiring the act is, that the act implies a choice and that it is felt to
be impolite and unjust to make a man answerable for harm, unless he might have
chosen otherwise.

R. v. Larsonneur (1933 UK) BAD LAW!


Ratio: Bad law now bc basic physical voluntariness wasnt there. Actus reus of ANY offence
requires that the person charged voluntarily performed the act in question. In this case, the
woman was forced to go back to England and wasnt in basic control of her actions where you
could say that was her act. She wasnt the author of her actions.
Facts: French citizen charged with immigration offence for being found in UK at a time when
she was denied entry. After being required to leave England, Irish authorities put her on a ship
back to England and put her on shore and left. Authorities then charged her despite the fact
that she was refused entry.
Decision: Guilty
Reasons: She could not have chosen otherwise bc wasnt in physical control of actions.
Precedent: Viewed as a mistake bc improperly ignored the voluntariness component of the
Actus Reus.
R. v. Kilbride (1962 New Zealand S.C.)
Ratio: Voluntary physical act is an element of actus reus bare mental component of physical
act.
Facts: Person charged with failure to display warrant (license) for vehicle. Had done so but it
blew off or someone removed it from car.
Decision: Not guilty.
Reasons: Not the guilty act of the accused therefore no AR. There is no free and conscious
exercise of will.
R. v. Ruzic (2001 SCC)
Ratio: LeBel -- Punishing a person whose actions are involuntary in the physical sense is
unjust because it conflicts with the assumption in criminal law that individuals are autonomous
and freely choosing agents. (para. 46). Duress (if youre threatened to do something) means
youre morally involuntary even though you are physically voluntary.

42

OMISSIONS
Status:
have fallen into disrepute and are not a meaningful part of our contemporary criminal
law (ie. offence in which the person is punished for being something features of the
person , some argue pseudo-status offences ie. bawdy house keeping (others argue
you have to do acts within this) ie. terrorism put acts in so ppl can have
control/agency
Omissions:
liability argued on failure to do something; going to have to find legal obligation, bc
doesnt like punishing for things dont do - General principle a person will not be
criminally liable for failing to act unless he or she is under a legal duty to act - no legal
duty to be a good Samaritan traditional common law perspective is that there is no
duty to assist one excpetion law of Quebec in Q Charter, s.2 imposes duty to
assist person in imperil if you dont put yourself at risk affirmative duty
Statutory Law
R v Browne (1997 Ont CA)
Ratio: Must be an undertaking before there is a legal duty within s. 217. No pre-existing
relationship that creates the legal duty.
Facts: Two drug dealers, Browne and Greiner. G swallowed bag with crack cocaine and bag
ruptured. Brown said he would take her to hospital by taxi. She died. He is going by taxi to
be distanced from it in some form. Trial -- in failing to do this he showed wanton and reckless
endangerment of life.
Charge: s. 219 Criminal negligence causing death.
Issues: Was there an undertaking? Did Browne have a legal duty arising from an
undertaking within the meaning of s. 217 to take her to hospital. If so, was he criminally
negligent causing death for omitting to do what was his duty to do.
Decision: Not guilty. No undertaking and therefore no legal duty.
Reasons: Abella -- An undertaking has to be something upon which reliance could be placed
and this was not considered an undertaking too casual, too quick. Saying take you to
hospital is not a firm enough undertaking.
Notes: Prof disagrees he undertook and discharged a duty. Could say there is an
undertaking to save her life by taking her immediately to hospital and he failed in doing that.
In so doing, he caused the death of his companion.
Common Law - not establishing offense, just duty
R v Coyne (1958 NBCA)
Ratio: A duty imposed by law used for criminal negligence can be found in common law or
statute.
Decision: Legal duty to take reasonable care when handling dangerous objects like a rifle
exists at common law and therefore can snap into the criminal negligence.
Popin (1981 Ont. CA)
Ratio: A duty can arise at statute or common law.
Facts: Child abuse case
Issues: What does duty imposed by law contained in definition of criminal negligence
mean?
Decision: Guilty of criminal negligence
Reasons: There is a CL duty for a parent to take reasonable steps to prevent child from being

43

exposed to illegal violence.


Notes: Could also have used s. 251
Nixon (BCCA)
Decision: Common law duty to provide care and protection to inmates under the care of
police. Officer in charge held liable for assaults of other police says can draw on common law
Notes: Could also have used s. 217.1, 215(1)(c)(i)
R v. Thornton (1991 Ont CA)
Ratio: Legal duty in s. 180(2) can be imposed by statute or common law.
Facts: Donated blood to Canadian Red Cross knowing he was HIV Positive.
Charge: s. 180(2) Common Nuisance
Issues: Can the legal duty in s. 180 arise from a common law duty?
Decision: Guilty under s. 216 + 180
Reasons: SCC upheld conviction under s. 216 (not CL) for failure to discharge legal duty of
using reasonable care in undertaking medical treatment did not ground liability in common
law duty of Donaghue and Stevenson obligation to care for neighbour
Circumstances: (ie. driving is not a crime, it is the circumstance in which act takes place that turns it
into a crime ie. impairment not always part of AR, but need to think about it, act becomes criminal bc
of circumstance, circumstance drives offense sometimes will have prescribed circumstance that is just
as important assault is example)

CONSEQUENCES AND CAUSATION


If part of the actus reus, must be proven beyond a reasonable doubt - ultimately
becomes an issues of causation - can we link accused to outcome/consequence; did
accused cause consequences; criminal code has no general prohibitions on causation
Factual Cause:
Was there some causal link between the accuseds actions and the prescribed
consequence - is there a factual link; usually not issue
Factual causation is not enough... Legal causation asks whether
the factual cause is significant enough to justify criminal liability.
legal causation... is a narrowing concept which funnels a wider range of factual
causes into those which are sufficiently connected to a harm to warrant legal
responsibility. (Maybin, para. 16)
R v Winning 1973 Ont CA
Ratio: Must be a factual link bw accuseds actions and the prohibited consequence.
Facts: Accused charged with obtaining credit by false pretense. Filled out name and address
accurately and defrauded on everything else. Turns out that Eatons only ever looked at name
and address and then gave credit.
Decision: Fraud did not result in false credit bc there was no factual cause.
Reasons: No factual causation line bw actions of accused and prohibited outcome.
Manslaughter
Smithers v The Queen 1978 SCC what is sufficient to say made out legal?
Ratio: Test for legal causation established. Test for legal causation is whether or not the

44

accuseds actions were a non-trivial or not insignificant cause of the prescribed


result. Thin skull rule applies in criminal matters: take victim as you find him.
Facts: Smithers kicked victim in stomach and died within five minutes. Autopsy revealed he
died from malfunctioning epiglottis choking on vomit. S
Decision: Guilty of manslaughter
Reasons: Kick was at least a contributing cause of death outside the de minimus range.
R v Cribbin Ont CA 1994 low Smithers standard is constitutionally sound
Ratio: The de minimus causation test and the thin skull principle approved in Smithers and
Creighton are consistent with the principles of fundamental justice that forbid the punishment
of the morally innocent.
Facts: Victim was beaten, left at side of road and drowned in his blood.
Issues: Is the causation test for manslaughter constitutional? Too sweeping?
Decision: Conviction upheld. Test is constitutional
Reasons: Person can be convicted of manslaughter on basis that acts were non-trivial role in
death, even though death not reasonably foreseeable. Here, the accuseds assault contributed
to the victims death even though victim had been subject to additional assaults by another
person and had died bc he drowned in his blood.
The Special Case: s. 231(5) Harbottle Causation
R v Harbottle 1993 SCC heightened level of causation from Smithers physical
involvement enough to satisfy
Ratio: In first degree murder cases under s. 235(1) and other sections where language of
caused by that person is used in the provision, the test for legal causation is: the
actions of the accused must form an essential, substantial, and integral part of the
killing of the victim. The higher standard will usually involve actual physical involvement in
the killing.
Facts: Harbottle and friend forceably confined a women, sexually assaulted her and killed her.
Harbottle held her legs down while other strangled her.
Charge: s. 231(5) Murder is 1st degree when death caused while committing sexual assault,
kidnapping, hijacking, aggravated sexual assault, etc.
Issues: Can Harbottle be found guilty of first degree murder pursuant to s. 231(5)
Decision: Guilty.
Reasons: Cory For crimes 231(5) and with language of caused by that person, for
specifically first degree murder, the test for causation is higher: Actions of the form an
essential, substantial, and integral part of the killing of the victim. Here it was impossible to
distinguish bw strangler and person who held down legs.
Irrespective of whether a murder is planned and deliberate on the part of any person, murder
is first degree murder in respect of a person when the death is caused by that person while
committing or attempting to commit...
R v Nette 2001 SCC
Ratio: Smithers standard of causation is the one standard in criminal with one narrow
exception of Harbottle for s. 231(5) and sections with the associated language for first degree
murder.
Facts: Nette robbed 95yr woman. After he left, she fell off bed and suffocated.
Issues: Does the Smithers causation apply? Should Harbottle apply to second degree murder
as well?
Decision: Conviction of second degree murder upheld. Smithers test applies.
Reasons: Unanimous Smithers test is one test of causation for criminal law. Court split on
how to articulate the Smithers standard. Majority (Arbour) preferable to phrase the standard

45

of causation in positive terms using a phrase such as significant contributing cause


The causation standard expressed in Smithers is still valid and applicable to all forms of
homicide. In addition, in the case of first degree murder under s. 231(5) of the Code, Harbottle
requires additional instructions, to which I will return. The only potential shortcoming with the
Smithers test is not in its substance, but in its articulation. Even though it causes little
difficulty for lawyers and judges, the use of Latin expressions and the formulation of the test in
the negative are not particularly useful means of conveying an abstract idea to a jury. In order
to explain the standard as clearly as possible to the jury, it may be preferable to phrase the
standard of causation in positive terms using a phrase such as significant contributing cause
rather than using expressions phrased in the negative such as not a trivial cause or not
insignificant. Latin terms such as de minimis are rarely helpful. (Arbour J. for the majority)
Dissent: Leave articulation of Smithers as is. Significant is different than not insignificant.
Words have a meaning that should be given to them and different words often convey very
different standards to the jury. In my view, describing a contributing cause as having a
significant impact attaches a greater degree of influence or importance to it than do the
words not insignificant.
(LHeureux-Dub J., in minority, para. 10) urge to use significant, but insignificant not wrong
Notes: Appears that the majority is trying to increase the test by changing the wording.
However, they explicitly say they are not changing the standard. Its just preferable to use a
certain type of language split 5-4 LHereux Dube says raising standard significant is
higher standard than not insignificant
Intervening Causes chain of causation severed- rare to be let off on chain of causation
R v Pagett 1983 UK
Ratio: Chain of causation not severed if accuseds act precipitated the direct cause.
Facts: Accused was coming out of scene of crime, a standoff with police. Used pregnant gf as
defence. Police in self defence shot the gf and she died.
Decision: Guilty of manslaughter. The chain of causation is not severed
Reasons: Intervening act that directly caused the death was precipitated by accused action.
R v Blaue 1975 UK
Ratio: Chain of causation not severed if accuseds act precipitated the direct cause put in
motion change of acts that led to her death.
Facts: Accused stabbed woman, punctured her lung. Refused blood transfusion bc Jehovahs
Witness. Died.
Issues: Did accused cause the death?
Decision: Guilty of manslaughter. The chain of causation is not severed.
Reasons: Intervening act that directly caused the death was precipitated by accuseds action.
R v Maybin (2012 SCC 24)
Ratio: Governing question is always: were the dangerous unlawful acts of accused a
significant contributing cause of the death? Two approaches to analyzing intervening causes:
1) Reasonable foreseeability: Was the general nature of the intervening act reasonably
foreseeable? 2) Independence: Was the intervening act sufficiently independent and
overwhelming to sever the impact of the accuseds actions?
Facts: Maybin beat victim who falls unconscious onto pool table. Bouncer then punches him in
back of head while unconscious. Cause of death was cranial damage.
Decision: Guilty. Maybin was a significant, contributing cause of the death.
Reasons: Karakatsanis 1) General nature of intervening act was reasonably foreseeable to
Maybin. 2) The intervening act was not sufficiently independent it was part of a chain of

46

events. Find causation on both foreseeability and independence.


Depending on the circumstances, assessments of foreseeability or independence may be
more or less helpful in determining whether an accuseds unlawful acts were still a significant
contributing cause at the time of death. Any assessment of legal causation should maintain
focus on whether the accused should be held legally responsible for the consequences of his
actions, or whether holding the accused responsible for the death would amount to punishing
a moral innocent. (para. 29)
Is it fair to attribute the resulting act to the initial actor? (para. 30)
both are helpful think about both- decide in aggregate together (says ultimate standards is
always were dangerous acts significant contributing cause of death (nette language)

MENS REA
Remember the maxim: actus non facit reum, nisi mens sit rea - the act is not guilty
unless the mind is also guilty
Best thought of as the required state of mind

MENS REA

Each different crime has a specific fault element which must be related to the actus reus of
that crime.
Principle of Symmetry: the prescribed mens rea must attach to every element of the
actus reus.
DETERMINING THE REQUISITE MENS REA
Statutory Provision

Is there EXPRESS language of mens rea? (e.g. intent, wilfully, etc)



Yes
No

Language governs
Is the offence a true crime or regulatory

offence? (Prue and Baril)


1. Nature and seriousness of the
offence
o Purpose of object of the
legislation?
o High stigma (mala in se deviant and bad)? Low
stigma (mala prohibita -didnt live up to a
standard of conduct)
2. Seriousness of the penalty

Is the prescribed mens rea unconstitutional?
True Crime
Regulatory Offence

Mens rea is
The mens rea is

47

Yes if.

presumptively any
subjective form of
mens rea (Buzzanga
and Desrocher)


Stigma
offence with
objective
mens rea
(Vaillancourt)

True crime
with strict or
absolute
liability
(Hundal)

Risk of loss of
liberty with
absolute
liability (BC
Motor
Vehicles)

FORMS OF MENS REA


Subjective
INTENT (indicated by
Forms of
words with intent,
Mens Rea
wilful, for the purpose
(accused subj
of, means)
had guilty mind
in relation to
consequences)

presumptively strict
liability (Sault Ste
Marie)

Intent is really about


deciding to engage in a
course of action, whether
you desire to or not.
KNOWLEDGE (knowing)
RECKLESSNESS

WILFUL BLINDNESS

Direct Intent To do an act with the goal,


purpose, or desire of bringing about the
prescribed harm.
Indirect (Oblique) Intent The accused does
an act with the goal, purpose, or desire of bringing
about something other than the prescribed harm
but also knows (subjectively foresees) that the
prescribed harm is certain or virtually certain to
occur (Hibbert)
Has subjective knowledge of some fact or state of
affairs
Involves knowledge of a danger or risk and
persistence in a course of conduct which creates
the risk that the prohibited result will occur
(Sansregret)
Arises where a person who has become aware of
the need for some inquiry declines to make that
inquiry bc he does not wish to know the truth.
(Sansregret)
Where willful blindness is shown, that is the same
as knowing. When a crime specifies knowledge as
a requirement, willful blindness will also do
(Briscoe)

If the statute is silent, the mens rea for a true crime is satisfied by any of the above
(Buzzanga and Desrocher)
If the crime is a stigma offence, the mens rea cannot be below this line (Vaillancourt;
Martineau; Finta)

Objective
Forms of
Mens Rea

(reasonable
person in
accuseds
position would
have had the
required guilty
mind or would
have acted
differently)

CRIMINAL NEGLIGENCE

PENAL NEGLIGENCE

A marked and substantial departure from the


conduct of a reasonable person which shows
wanton and reckless disregard for the lives or
safety of others. (Tutton and Waite)
A marked departure from the conduct of a
reasonable person (Hundal) See also Beatty, Roy

True crimes cannot have a mens reas below this line (Hundal)

STRICT LIABILITY
If the statute is silent, this
is the default mens rea for
a regulatory offence. (Sault
Ste Marie)
ABSOLUTE LIABILITY
Not permitted when the

Once the Crown proves actus reus beyond a


reasonable doubt, the accused can advance a due
diligence defence.
Once the Crown proves actus reus beyond a
reasonable doubt, liability attaches. No defence

48

punishment involves a
based on an absence of mens rea.
potential loss of liberty (BC
Motor Vehicles)
Stigma Offences
Constitutional requirement to have a subjective MR to justify the
stigma associated with the conviction and sentence of the offence
(Vaillancourt)
Stigma offences must have symmetry bw all elements of AR and MR
the whole way through (Vaillancourt)
Rationale: it is a PFJ that there must be proportionality bw the moral
blameworthiness of the person and the seriousness of the conduct
(Vaillancourt) therefore subj foresight is required.
Includes murder (Vaillancourt, Martineau), attempted murder, theft
(Vaillancourt), crimes against humanity (Finta)
Manslaughter is NOT a stigma offence (Creighton)
Regulatory
Lower form of MR. Three phases of MR developed:
offences
Phase 1: absolute liability
No mental fault requirement. Upon proof by the Crown beyond a
reasonable doubt that the actus reus is made out, the accused is found
guilty (Pierce Fisheries).
Phase 2: strict liability
Middle standard bw subj MR and absolute liability.
This is the presumption if the statute is silent (Sault Ste Marie)
Upon the Crown proving the actus reus beyond a reasonable doubt, the
burden shifts to the accused to prove due diligence on a balance of
probabilities (Sault Ste Marie)
Phase 3: constitutional constraints
You can not be exposed to a loss of liberty without mens rea. Absolute
liability and a potential for a loss of liberty can no longer be
combined. (BC Motor)
Reverse burden on accused to show due diligence is constitutional
(Wholesale Travel)
Is it a true crime
1. Nature and seriousness of the offence
or a regulatory
o Purpose of object of the legislation?
offence? (Prue
o High stigma?
and Baril)
2. Seriousness of the penalty

INTENT
Common Law Presumption of Subjective Mens Rea - As for the role of the
presumption of subjective fault in assessing competing interpretations, it sets out an
important value underlying our criminal law. It has been aptly termed one of the
presumptive principles of criminal justice: R. v. Beatty, 2008 SCC 5, [2008] 1 S.C.R.
49, per Charron J., at paras. 22-23. While the presumption must and often does
give way to clear expressions of a different legislative intent, it nonetheless
incorporates an important value in our criminal law, that the morally innocent should
not be punished. - Justice Cromwell in R v ADH, 2013 SCC 28, para 27.

R v Buzzanga and Durocher Ont CA 1979


Ratio: Mens rea is presumptively subjective mens rea - The greater the likelihood of the
relevant consequences ensuing from the accuseds act, the easier it is to draw the inference
that he intended those consequences. The purpose of this process, however, is to determine

49

what the individual intended, not to fix him with the intention that a reasonable person might
be assumed to have in the circumstances
Meaning of -Wilfully - means intentionally causing an outcome or recklessly, but section
doesnt include recklessness
Facts: Ds put out a satirical document that appeared to be promoting hatred of French
Canadians. They intended the document to support building a French high school. The trial
judge treated testimony that they wished to create a controversy, furor, and uproar as
admission that they had a guilty mind.
Charge: Wilfully promoting hatred s. 319(2) (281.2(2))
Issues: What is the meaning of wilfully? Did the trial judge misdirect himself as to the
meaning of willfully? What does the word willfully in s.319(2) mean?
Decision: New trial ordered b/c intent was not to promote hatred
Reasons: An intention to create controversy, furor and uproar is not the same thing as an
intention to promote hatred and it was an error to equate them.
As a general rule, a person who foresees that a consequence is certain or substantially
certain to result from an act which he does in order to achieve some other purpose, intends
that consequence [at law]. The actors foresight of the certainty or moral certainty of the
consequences resulting from his conduct compels a conclusion that if he, none the less, acted
so as to produce it, then he decided to bring it about (albeit regretfully), in order to achieve his
ultimate purpose.
R v Tennant and Naccarato Ont CA 1975
Ratio: Distinguishes bw subjective and objective mens rea where liability is imposed on a
subjective basis, what a reasonable man ought to have anticipated is merely evidence from
which a conclusion may be drawn that the accused anticipated the same consequences. On
the other hand, where the test is objective, what a reasonable man should have anticipated
constitutes the basis of liability.
R v Steane 1947 UK - the true meaning of intention and the reasons of its confusion with
motive
Ratio:
Facts: Steane was a radio announcer during the war and was forced by Nazis into reading the
German news and assisting with film production. Charged with doing acts to assist the
enemy. Steane challenged on the ground that he only did it for fear for his wife and children.
Issues: Can he have a guilty mind if he never intended to assist the enemy?
Decision: Conviction quashed - motive has allowed acquittal of the accused since they
highlighted the moral gap between what Steane did (help the enemy) and his purpose (save
his family).
Knowledge that propaganda broadcasts would assist the enemy was defeated by his innocent
intention to protect himself and his family. It seems that where duress is involved, knowledge
is not sufficient intent - requires desire (direct intention).
This is not a consistent interpretation of M/R. Usually knowledge (indirect intention) that
actions will cause prohibited results is sufficient intent. Confused intent with motive, as well
as with duress.
Reasons: Accuseds intent was negative bc he only did it under force of threat and to save
wife and children from concentration camp.
Hibbert v The Queen 1995 SCC - Duress does not raise M/R requirement to desire
clarifies Steane duress does not negate MR (which requires knowledge, not desire), but it can

50

be a defense
Ratio: Purpose in s. 21(1)(b) means intent for the purposes of mens rea. Duress can only
negate mens rea for some offences and will depend on wording of statute. Duress available as
CL defence though.
Facts: Accused testified that he was forced at gunpint to lure victim, his friend, down into
lobby where he was then shot.
Charge: Aiding and abetting murder
Issue: What is the applicability of the defence of duress in the context of aiding and abetting
the commission of an offence under s. 21(1)(b) of the Criminal Code?
Decision: Appeal allowed. Trial judges instructions were incorrect. New trial ordered.
Reasons: Lamer -- Trial judged erred in instructing jury that mens rea could be negated by
duress. Duress is capable of negating the mens rea for some offences (is the definition of the
offence as written by Parliament capable of supporting the inference that the presence of
coercion can have a bearing on the existence of mens rea?), but not for aiding the commission
of an offence under s. 21(1)(b). Nonetheless, duress can still function as an excuse-based
defence at common law even if Crown successfully proves elements of the offence. .

RECKLESSNESS

Recklessness is a lower form of intention. Concerned with the consciousness of risk and
then conduct despite that risk - must be subjectively aware of the risk

Reckless: person foresees that action MAY cause the result and takes deliberate and unjustifiable
risk anyway; the degree of likelihood that person foresaw is anything less than certainty (anywhere
between 0 and 100% chance).

Distinct from intent - about seeing a risk, not a virtual certainty; distinct from negligence

WILLFUL BLINDNESS

type of knowledge subjectively knew but didn't question; Willful blindness is about the
recklessness related to knowing the facts. Ask: did the accused deliberately choose not to
inquire bc he knew that looking would fix him with knowledge?- Suspicions aroused (re:
circumstances or that actions will cause consequences) but deliberately close eyes to risk,
does not investigate further

R v Sansregret 1985 SCC


Ratio: Willful blindness arises where a person who has become aware of the need for some
inquiry declines to make the inquiry because he does not wish to know the truth. Where willful
blindness is shown, that is the same as knowing. When a crime specifies knowledge as a
requirement, willful blindness will also do.
Facts: Accused charged with sexual assault after breaking into womens home. She admitted
to consenting to sex but did so to ensure her own safety. Accused had done this before and his
probation officer convinced victim not to press charges. Accused argued he didnt subjectively
know there was no consent.
Charge: Sexual assault requires as part of actus reus absence of consent. Accused must
subjectively know that there is no consent.
Decision: Guilty. He was wilfully blind and therefore had the knowledge.
Reasons: Given past history, he subjectively knew there was no consent and decided not to
inquire bc he knew the risks.

51

Recklessness is found in the attitude of one who, aware that there is danger that his conduct
could bring about the result prohibited by the criminal law, nevertheless persists, despite the
risk. (Sansregret pg. 447)
Negligence is tested by the objective standard of the reasonable man. A departure from this
accustomed sober behaviour by an act or omission which reveals less than reasonable care
will involve liability at civil law but forms no basis for the imposition of criminal penalties. In
accordance with well-established principles for the determination of criminal liability,
recklessness, to form part of the criminal mens rea must have an element of the subjective.
[Sansregret, pg. 447]
Willful blindness arises where a person who has become aware of the need for some inquiry
declines to make the inquiry because he does not wish to know the truth. (Sansregret, p. 448)
R v Briscoe 2009 SCC
Ratio: Willful blindness can substitute for actual knowledge whenever knowledge is a
component of the mens rea. The doctrine of willful blindness imputes knowledge to an
accused whose suspicion is to the point where he sees the need for further inquiries but
deliberately chooses not to make those inquiries.
Facts: Accused charged with first degree murder, kidnapping and assault. Charged as aider
and abetter (he drove, physically restrained person) but did not do killing. D suspected might
kill her but didnt ask bc he didnt want to know.
Decision: Guilty. Wilful blindness. Knew there was a risk there would be a murder but didnt
want to know.

CRIMINAL NEGLIGENCE
Criminal Negligence
Actus Reus: Section 219: Every one is criminally negligent who (a) in doing anything or (b)
in omitting to do anything that it is his duty to do, shows wanton and reckless disregard for
the lives or safety of others.
Mens Rea: Showing, through your actions, a marked and substantial departure form the
conduct of a reasonable person which shows wanton and reckless disregard for the lives
or safety of others. (Tutton and Waite)
R v Tutton and Tutton 1989 SCC
Ratio: Crim neg is an objective form of MR. Mens rea is a marked and substantial
departure from the conduct of a reasonable person which shows wanton and
reckless disregard for the lives or safety of others.
Facts: Due to religious views, refused to administer insulin to diabetic 5 yr old son. He died.
Charge: Manslaughter through criminal negligence
Issues: Is the test for crim neg subjective or objective?
Decision: Split 3/3
Reasons: Three reasoned that language of section said nothing about subjective
apprehension of risk. Three said there is a subjective aspect in wanton and reckless
disregard.
Precedent: In R v Waite 1989 SCC, judges also divided 3/3 on subjective/objective.
R v Hundal 1993 SCC
Ratio: There is no constitutional principle that requires subjective foresight for criminal
offences. The constitutional bar for true crimes is not below penal negligence. Mens rea for
dangerous driving is a modified objective test: marked departure form the standard of care

52

expected of a reasonable person in the circumstances of the accused


Facts: A dump truck carrying a heavy load went through a red light and killed passenger in
another car. Accused did not believe he could stop so went through pressing on horn.
Charge: Dangerous driving causing death.
Issues: Is there a subj element of fault required for dangerous driving?
Decision: Guilty. Appeal dismissed.
Reasons: Cory - His driving represented a gross departure from the standard of a reasonable
prudent driver
R v Creighton
Ratio: 1. Criminal negligence is an objective standard 2. Section 7 of the Charter does not
require that objective fault relate to all prohibited consequences of the offence (ie. not perfect
symmetry). Its acceptable that there is only objective foresight of bodily harm, rather than
relating to the death.
Facts: Accused injected cocaine into victim causing death.
Charges: 222(5)(a) manslaughter due to an unlawful act
Issues: Does the CL definition of unlawful act manslaughter violate s. 7 of the Charter? Is
manslaughter a crime that should have subjective mens rea?
Decision: Guilty. Appeal dismissed.
Reasons: McLachlin CL requirement for mens rea for manslaughter (objective
foreseeability of bodily harm as opposed to death) is constitutional. Must be a marked
departure from the standard of care of a reasonable person. Fact that manslaughter is named
differently from murder means it is less blameworthy and less stigma attached. Only in cases
of incapacity to appreciate the risk should personal characteristics be considered.
Dissent: Lamer Standard for manslaughter should be objective foreseeability of death and
objective standard should account for persons experiences and frailties.

PENAL NEGLIGENCE
R v Beatty 2008 SCC
Ratio: Mens rea for penal negligence is a marked departure from the conduct of a
reasonable person. Mens rea for dangerous driving is a modified objective test: marked
departure from the standard of care expected of a reasonable person in the circumstances of
the accused
Facts: Drifted into oncoming traffic after momentary lapse and killed three passengers in
other car.
Charge: Dangerous driving causing death (x3).
Issues: Is there a subj element of fault required for dangerous driving?
Decision: Not guilty. Appeal allowed
Reasons: Momentary lack of attention was not a marked departure.
Short of incapacity to appreciate the risk or incapacity to avoid creating it, personal attributes
such as age, experience, and education are not relevant. The standard against which the
conduct must be measured is always the same it is the conduct expected of the reasonably
prudent person in the circumstances. The reasonable person, however, must be put in the
circumstances the accused found himself in when the events occurred in order to assess the
reasonableness of the conduct. (Beatty, para 40)
wanton and reckless disregard required for criminal negligence will exist when, at minimum,
an accused gave no thought to an obvious and serious risk to the life or heath of another
person in circumstances where a reasonable person would have recognized that risk and
avoided it. [F(J), 2008]

53

Charron J. in Beatty: Unlike civil negligence, which is concerned with the apportionment of
loss, penal negligence is aimed at punishing blameworthy conduct. Fundamental principles of
criminal justice require that the law on penal negligence concern itself not only with conduct
that deviates from the norm, which establishes the actus reus of the offence, but with the
offenders mental state. A mere departure from the standard expected of a reasonably
prudent person will meet the standard for civil negligence, but will not suffice to ground
liability for penal negligence. The distinction between a mere departure and a marked
departure from the norm is a question of degree. It is only when the conduct meets the higher
threshold that the court may find, on the basis of that conduct alone, a blameworthy state of
mind. [paras. 6-7]
R v Roy 2012 SCC
Ratio: Confirmed Beatty standard for penal neg. For the purposes of the criminal law, must be
a MARKED (not mere) departure. ASK: 1) Would a reasonable person have foreseen the risk
and taken steps to avoid it if possible? 2) If so, was the failure to foresee the risk and take
steps to avoid it, a marked departure from the standard of care expected of a reasonable
person in the accuseds circumstances?
Facts: Accused pulled motor home into path of oncoming tractor trailer. Accuseds passenger
killed. Roy charged with dangerous driving causing death.
Decision: Not guilty.
Reasons: Cromwell Simple carelessness, to which even the most prudent drivers may
occasionally succumb, is generally not criminal. Mere carelessness = civil negligence. Marked
carelessness = penal neg. Here, there was a single and momentary error in judgment. The
manner of driving does not show that accuseds standard of care was a marked departure
from that expected of a reasonable driver in the same circumstance.
It is critically important to ensure that the fault requirement for dangerous driving has been
established. Failing to do so unduly extends the reach of the criminal law and wrongly brands
as criminals those who are not morally blameworthy. The distinction between a mere
departure, which may support civil liability, and the marked departure required for criminal
fault is a matter of degree. The trier of fact must identify how and in what way the departure
from the standard goes markedly beyond mere carelessness. (para. 30)

CHARTER CONSIDERATIONS AND REVIEW


What, if any limits, does s. 7 place on the mens rea for offences?
Stigma Offences
(crimes with
significant
"stigma"
attached, such as
culpable homicide
and constructive
murder, require
proof of the mens
rea element of
subjective
foresight of
death)

Constitutional requirement to have a subjective MR to justify the


stigma associated with the conviction and sentence of the offence
(Vaillancourt)
Stigma offences must have symmetry bw all elements of AR and MR
the whole way through (Vaillancourt)
Rationale: it is a PFJ that there must be proportionality bw the moral
blameworthiness of the person and the seriousness of the conduct
(Vaillancourt) therefore subj foresight is required.
Includes murder (Vaillancourt, Martineau), attempted murder, theft
(Vaillancourt), crimes against humanity (Finta)
Manslaughter is NOT a stigma offence (Creighton)

R v Vaillancourt 1987 SCC


Ratio: It is a PFJ that there must be proportionality bw the moral blameworthiness of the crime
and the seriousness of the offence. For stigma offences, the constitution demands

54

symmetrical, subjective mens rea (e.g. murder, theft)


Facts: Armed robbery in a pool hall. Accomplice used a gun that accused thought was
unloaded. Didnt foresee death and in fact thought that no death would occur.
Charge: Felony/constructive murder (s. 213(d) now repealed) if a person dies during the
commission of another offence, guilty of murder whether or not person intends to cause
death and whether or not death is foreseeable.
Issues: Does s. 213(d) of CC violated s. 7 of the Charter?
Decision: s. 213(d) unconstitutional. Violates the Charter.
Reasons: Lamer -- Subj mens rea will not constitutionally suffice. Court creates a concept
called a stigma offence (judicially created category), which are based on a central
proposition that court inscribed in PFJ that there must be proportionality bw the moral
blameworthiness of the person and the seriousness of the conduct. Before a homicide can
justify the stigma associated with the conviction and sentence of murder, it must have a
subjective MR.
there must be come special element with respect to the death before a culpable homicide
can be treated as murder. That special mental element gives rise to the moral
blameworthiness which justifies the stigma and sentence attached to a murder conviction.
(page 706)
R v Martineau 1990 SCC
Ratio: Murder is a stigma offence. PFJ require that stigma offences need a subjective mens rea
that reflect the seriousness of the crime. Any form of murder that doesnt require subjective
mens rea all the way through is unconstitutional.
Facts: Felony/constructive murder case. Accused in a robbery when accomplice killed two ppl.
He didnt intend to kill or foresee the death and thought he was participating only in a B&E.
Issues: Does felony murder under s. 213 violate ss. 7 and 11(d) of the Charter?
Decision: Yes. S. 213 violates Charter and not justified under s. 1
Reasons: Fundamental justice demands that moral blameworthiness is proportional to the
stigma and punishment of the offence. This degree of moral blameworthiness only arises
where you have a subjective mens rea. Here stigma and punishment attaching to murder
should be reserved for those who choose to intentionally cause death or who choose to inflict
bodily harm that they know is likely to cause death. To label and punish a person as a
murderer who did not intend or foresee death unnecessarily stigmatizes and punishes those
whose moral blameworthiness is not that of a murderer.
Dissent: LHeureux-Dube 1. Misplaced compassion. These people are not morally innocent
in the first place. 2. There is enough culpability in the act and that is enough to deal with the
stigma. 3. If deterrence is ever going to have a role to play, it will be in cases like robberies
where if a person dies you will be labelled a murderer.
R v Finta 1994 SCR
Ratio: Crimes against humanity are stigma offences and therefore must have a subjective
mens rea in order to ensure that the moral blameworthiness is proportional to the stigma of
the conviction and sentence.
Facts: Finta was a senior officer in concentration camp in Hungary. Crown argued that only
needed to show he had mens rea for underlying acts (ie. unlawfuly confinement, robbery,
kidnapping and manslaughter) not for war crimes or crimes against humanity.
Issues: Does accused have to be aware that his conduct would bring his actions into the
realm of crimes against humanity? In particularly, that it would shock the conscience of all
right thinking people (test)?
Decision: Acquitted
Reasons: Cory - Majority applied Martineau and held that crimes against humanity are stigma
offences and therefore must show subjective mens rea. If you are going to convict someone of

55

crimes against humanity, must show they had a subjective relationship to crimes against
humanity and not just the underlying act.
Precedent: O
there are certain crimes where, because of the special nature of the
unavoidable penalties or of the stigma attached to a conviction, the principles of fundamental
justice require a mental blameworthiness or a mens rea reflecting the particular nature of that
crime. It follows that the question which must be answered is not simply whether the accused
is morally innocent, but rather, whether the conduct is sufficiently blameworthy to merit the
punishment and social stigma that will ensue upon conviction for that particular offence.
R v De Sousa other result-driven offences
Ratio: whether s.269 of the Criminal Code violated the s.7 of the Charter as it potentially
allowed for prison sentences for "Absolute Liability" offences - held that s.269 did not violate
s.7
due to the lack of stigma or any sort of significant prison sentence attached to the offence it
did not warrant a higher "subjective fault" requirement (R. v. Martineau).
The Court dismissed the argument that the offence would punish the morally innocent by not
requiring proof of intention to bring about the consequences. Instead the offence aims to
prevent objectively dangerous acts (this justification was elaborated on in R. v. Creighton).
Kent Roach, Mind the Gap (pg. 488)

ABSOLUTE AND STRICT RESPONSIBILITY


Regulatory/Public
Welfare Offences
not true crimes,
but can be
convicted of (part
of penal law)

Is it a true crime or
a regulatory
offence?
Enacted in CCC
true crime; when
outside like CDA,

Lower form of MR fall below Hundal bar. Three phases of MR


development:
Phase 1: absolute liability
No mental fault requirement or demand for fault. Upon proof by the
Crown beyond a reasonable doubt that the actus reus is made out,
the accused is found guilty (Pierce Fisheries).
Phase 2: strict liability
Middle standard bw subj MR and absolute liability.
This is the presumption if the statute is silent (Sault Ste Marie)
Upon the Crown proving the actus reus beyond a reasonable doubt,
the burden shifts to the accused to prove due diligence on a balance
of probabilities (Sault Ste Marie)
Phase 3: constitutional constraints
You can not be exposed to a loss of liberty without mens rea.
Absolute liability and a potential for a loss of liberty can no
longer be combined. (BC Motor)
Reverse burden on accused to show due diligence is constitutional
(Wholesale Travel)
1. Nature and seriousness of the offence (Prue and Baril; Sault Ste.
Marie);
o nature of conduct prohibited - mala in prohibita (risk

which weve prohibited) v. mala in se (know its wrong bc


bad in itself)? difficult to do this way b/c not clear
o

(Wholesale Travel)
Purpose of object of the legislation? (regulatory prevent future

56

Firearms Act need


to look at

harm by creating standards of conduct; or inherently wrong)


High stigma? (criminal stigma is high bc seen as deviant; reg
stigma is low)
2. Seriousness of the penalty
o

PHASE 1: Absolute Liability Upon proof by the Crown beyond a reasonable doubt that the
actus reus is made out, the accused is found guilty (subject to any applicable free-standing
defences)! no MR!
Beaver v The Queen 1957 SCC
Ratio: Possession offences require the Crown to prove that the accused had subjective
knowledge of the nature of the substance cant have absolute liability for true crime
need to prove hard
Facts: Beaver charged with possession of heroin. He thought he had powdered milk and in
fact he had heroin. Crown argued silent on mens rea, so he doesnt need to know (ie. absolute
liability, no mens rea).
Decision: Not guilty of possession. This is a true crime and if statute is silent, we presume
subjective.
Reasons: A person cannot be said to possess that substance unless they knew the nature of
the substance. Here, since it was an honest belief, Beaver had no MR.
R v Pierce Fisheries Ltd 1971 SCC
Ratio: In past, common law imposed strict liability for regulatory offences.
Facts: Accused charged with having undersized lobsters in its possession contrary to the
Fisheries Act - regulatory
Decision: Guilty. Absolute liability.
Reasons: It doesnt matter if company intended it, if they had taken care not to fish for
undersized lobster or didnt even know they had them bc the ACT was made out, which is all
that is required for absolute liability offences.
PHASE 2: Strict Liability - Upon the Crown proving the actus reus beyond a reasonable
doubt, the burden shifts to the accused to prove due diligence on a balance of probabilities.
R v City of Sault Ste. Marie 1978 SCC
Ratio: Courts will no longer interpret regulatory offences as absolute liability offences unless
statute explicitly states that. Common law presumption is now strict liability. Strict Liability
Upon the Crown proving the actus reus beyond a reasonable doubt, the burden shifts to the
accused to prove due diligence on a balance of probabilities. Mens rea is if you take care and
can show you did. Courts presume strict unless legislate says dont want any wont
presume this
Facts: City of Sault Ste Marie charged with polluting a stream contrary to Water Resources
Act.
Decision: Offence of causing pollution is a strict liability offence.
Precedent: Why change the rule? 1) purpose of these offences is to promote careful conduct
in commercial life and mens rea should therefore be about taking reasonable care. 2)
background principle that deprivation of liberty should not be inflicted on someone without a
degree of fault. Why did they reverse the burden? The accused is in a better position to show
it. They can tell you how they took care. Prior to Charter, in this second phase
PHASE 3: Charter Considerations
Reference re Section 94(2) of the BC Motor Vehicle Act 1985 SCC

57

Ratio: 1) PFJ (s.7) are substantive review of law. 2) Section 7 is the parent for ss 8-13. They
are really just specifications of the kinds of things that are required for a just system. 3) It
constitutionalizes Sault Ste. Marie. You will not be exposed to a loss of liberty without mens
rea. Absolute liability and a potential for a loss of liberty can no longer be combined
if risk through imprisonment or probation (otherwise can have absolute)
Facts: BC govt had an offence for driving with suspended license whether or not they know
its suspended.
Issues: Is it constitutional to have absolute liability for driving with a suspended license?
Decision: No. Any regulatory offence that had absolute liability as a consequence and
imprisonment as a punishment is unconstitutional. Offends PFJ by allowing conviction of
morally innocent.
Reasons: A law that has the potential to convict a person who had not really done anything
wrong offends the principles of fundamental justice and, if imprisonment is available as a
penalty, such a law then violates a persons right to liberty under s. 7 (includes substantive
review if law is substantively fair)
R v Wholesale Travel 1991 SCC
Ratio: The reverse burden to show due diligence that is associated with strict liability is
constitutionally acceptable.
Facts: Company charged with false or misleading advertising contrary to Competition Act for
wrongly advertising that its travel packages were wholesale. Accused challenged s. 36 of
Competition Act which created a statutory due diligence defence violated s 11(d) of the
Charter.
Issues: Upon proof of the act, you are presumed guilty (ie. absolute liability). Was the reverse
burden a breach of the presumption of innocence?
Decision: Split decision. still regulatory offense even though up to 5 yrs
Reasons: Lamer violates the presumption of innocence and cannot be justified under s. 1.
Ioaccobucci The reverse burden does infringe s. 11(d) but is justified under s.1. For a
regulatory offences, the reversal of burden is justified bc the purpose is to try to get people to
take care. Cory, LHeureux-Dube no breach of presumption of innocence under s. 11(d).

DEFENCES
Forms of
Defences

1. Negative Defences
No mens rea OR no actus reus

Mistake of fact (no MR), intoxication, mental disorder, automatism


2. Affimative Defences
AR and MR, but person should not be punished.
o Excuses You did the wrong thing but we understand and wont
punish you. A concession to human frailty
Duress, necessity, provocation
o Justifications shouldnt be punished bc in some way you did
the right thing
Self defence
3. Procedural Stops judicial proceedings stay
Called defences but dont have anything to do with the culpabilitiy of the
offender. Have more to do with governments process.
E.g. entrapment, abuse of process, officially induced error
Exemptions to criminal liability
Children under 12, foreign dignitaries, mental disorder
Basic test Air of reality Must show that on the facts, its a live issue. Applies to any
for when
defence.
a defence ASK: Is there some evidence on the basis of which a properly instructed jury

58

is live
Burden of
proof

Justificatio
n vs.
Excuse

action reasonably could acquit? (Osolin; Cinous; Mayaurin) normal rule - once
has this, Crown must prove BRD
Standard BOP
For any defence for which there is an air of reality, the Crown must disprove it
beyond a reasonable doubt.
Applies to mistake of fact, normal intoxication, provocation, duress, self
defence, necessity
Reverse BOP
Burden is on the accused to prove the defence on a balance of probabilities
Applies to mental disorder, extreme intoxication, automatism
What difference does it make if defence is classified as excuse or
justification?
Largely its theoretical and conceptual but there are some practical
implications.
1. It could have effect on how you create criteria you enforce for a defence.
o If its an excuse, will insist more on pressure, immediacy, urgency.
o If justifications are about getting it right, less worried about immediacy.
2. Cannot be sued in civil law for doing the right thing (ie. justification).
3. Cannot be a party to a justified offence, but you could be a party to an excused
offence. (Eg. If you hand someone a gun to shoot their aggressor in justified
self defence, cant be held liable. But if done so and they go commit a robbery,
argue provocation could be held liable.)

SEXUAL ASSAULT, MISTAKE OF FACT AND MISTAKE BELIEF


IN CONSENT
MISTAKE OF FACT (s. 273)
Ignorantia facti excusat, ignorantia juris non excusat.
Ignorance of facts excuses, ignorance of the law is no excuse.
Is it enough that the belief in some fact is honestly held or must it also be reasonable? if
unreasonable, but honestly held reasonableness of belief tracks
Summary
Negative defence (either no AR or MR)
No MR
Results in acquittal if proven
Burden of
BOP on Crown to disprove defence beyond a reasonable doubt
Proof
Mistake of
If the required mens rea is subjective, the mistaken belief needs to be
Fact
honest but NOT reasonable. (infer subjective mistake from objective facts)
If the required mens rea is objective, the mistaken belief must be
reasonable AND honest.
Elements of Consent
Sexual
Consent is part of both the actus rea (a factual matter) and mens reas (did
Assault
the accused have mental element knowing that there was no consent)
Implied consent does not exist in context of sexual assault. (Ewanchuk)
Actus reus
AR of assault (265):
1. Physically voluntary
2. Touching
Without consent (applies to all assault offences)

59

3. Whether the complainant, in her mind, consented to the touching


(Ewanchuk)
4. Committed in circumstances of a sexual nature (Chase) (circumstance)
TEST: Would a reasonable person conclude the touching was of a sexual
nature? interferes with sexual autonomy of person (Chase)

Consent in AR is factual issue


a. Determined from the perspective of the complainant, and
particularly through a consideration of whether the complainant, in
her mind, wanted the sexual touching to take place. (Ewanchuk)
If there is reasonable doubt as to complainants consent, did the
complainant consent because of force, fear, threats, fraud or the exercise
of authority (265(3)) (Ewanchuk)

Mens Rea
MR is subjective
1. An intention to apply direct or indirect force to another person; and
2. Knowledge of, or recklessness, or wilful blindness (Sansregret) as to lack of
consent.
3. Consent for MR is about the mind of the accused.
Did the accused have the mental element in relation to the fact of nonconsent (which would be established in AR)
a. Did the complainant affirmatively communicate by words or conduct
an agreement to engage in sexual activity?
b. Silence is not consent (Ewanchuk)
Did the accused honestly believe that the complainant communicated
consent through her words and/or actions?
c. If yes, the accused could argue that the MR did not attach to the AR
and there was a mistake of fact.
1. Is the MR for the offence subjective or objective?
Subjective general rule: If the required mens rea is subjective, the
mistaken belief needs to be honest but NOT reasonable.
a. Is it a sexual assault offence under ss. 271, 272 or 273?
i. YES s. 273.2(b) changes the general test for mistake of
fact. For mistaken belief in consent related to sexual assault,
the mistaken belief requires and honest AND reasonable
belief. Here reasonable means that reasonable steps were
taken to determine if consent was obtained.
1. Presumption is that someone is not consenting unless
you have taken reasonable steps to obtain consent
(Ewanchuk)
2. Was consent vitiated by any of the factors from
273.1(2)
a. Consent given by someone else
b. Complainant is incapable of consenting
c. Accused induced position of trust, power or
authority to obtain consent
d. Complainant expressed, by words or conduct, a
lack of agreement to engage in the activity; or
e. Complainant, having consented to engage in
sexual activity, expresses, by words or conduct,
a lack of agreement to continue to engage in the

60


273 of CCC
(postPappajohn;
1992
amendment
s)

activity.
3. Defence of mistake of fact not available where
mistaken belief in consent arose from self-induced
intoxication (273.2(a)(i)); or recklessness or wilful
blindness (273.2(a)(ii))
a. If belief was due to recklessness, WB or one of
those factors, mistake of fact is no defence and
MR made out
4. Accused must take reasonable steps in the
circumstances known to accused at the time to
determine if there was consent (273.2(b))
a. If accused did NOT take reasonable steps,
mistake of fact is no defence and MR made out
ii. No Mistaken belief must be honest but does not have to be
reasonable
Objective If the required mens rea is objective, the mistaken belief
must be reasonable AND honest.

273.1(1) Meaning of "consent"


Subject to subsection (2) and subsection 265(3), "consent" means, for the
purposes of sections 271, 272 and 273, the voluntary agreement of the
complainant to engage in the sexual activity in question.
273.1(2) Where no consent obtained
No consent is obtained, for the purposes of sections 271, 272 and 273,
where
a. the agreement is expressed by the words or conduct of a person other
than the complainant;
b. the complainant is incapable of consenting to the activity;
c. the accused induces the complainant to engage in the activity by abusing
a position of trust, power or authority;
d. the complainant expresses, by words or conduct, a lack of agreement to
engage in the activity; or
e. the complainant, having consented to engage in sexual activity,
expresses, by words or conduct, a lack of agreement to continue to
engage in the activity.
There is no point where consent cannot be withdrawn
265(3) Consent
For the purposes of this section, no consent is obtained where the
complainant submits or does not resist by reason of
a. the application of force to the complainant or to a person other than the
complainant;
b. threats or fear of the application of force to the complainant or to a person
other than the complainant;
c. fraud; or
d. the exercise of authority.
273.2 Where belief in consent not a defence need to always not just if
you subjectively think different from blindness; parliament trying to instil
practice
It is not a defence to a charge under section 271, 272 or 273 that the accused
believed that the complainant consented to the activity that forms the

61

Progression
of Law of
Sexual
Assault

subject- matter of the charge, where


a. the accused's belief arose from the accused's
i. self-induced intoxication, or
ii. recklessness or wilful blindness; or
b. the accused did not take reasonable steps, in the circumstances
known to the accused at the time, to ascertain that the complainant was
consenting.
OLD LAW:
Mens rea for sexual assault is subjective, so belief in consent needs to be
honestly held but NOT reasonable (Pappajohn). But, you cannot be wilfully
blind (Sansregret)
To put forth a mistaken belief in consent, there must be some evidence or
air of reality beyond the mere assertion of the mistaken belief
(Pappajohn)
CURRENT LAW (Post-Legislation; Post-Ewanchuk)
Defence of mistake of fact not available where belief arose from selfinduced intoxication, recklessness or wilful blindness (273.2(a) and
Sansregret for wilful blindness) OR
if person did not take reasonable steps in the circumstances to
determine if there was consent. (273.2(b)))
Presumption now is that someone is not consenting unless you have
taken reasonable steps to determine if they are. Silence is not consent
(Ewanchuk)

Seaboyer
perhaps more than any other crime, the fear and constant reality of sexual assault affects
how women conduct their lives and how they define their relationship with the larger society.
R v Chase 1987 SCC
Ratio: Definition of sexual assault. Sexual assault is an assault within any one of the
definitions in the CC which is committed in a sexual nature, such that sexual integrity of the
victim is violated. Objective test used to determine the actus reus would a reasonable person
assume the touching is of a sexual nature?
Actus:

Physically voluntary;

Unconsented to;

Touching;

Of a sexual nature.
Mens Rea

Intent to touch;

Knowledge (or willful blindness) as to a lack of consent.


Facts: Chase grabbed his 15 yr old neighbour around shoulders and grabbed her breasts
saying cmon, i know you want it and that he would tell ppl she had raped him. Charged with
sexual assault.
Decision: Guilty
Reasons: Conduct constituted an assault of a sexual nature.
JA and Advance consent
Ratio: at law, one cannot give advance consent for sexual touching that will take place while
one is unconscious
Facts: Consented to sexual acts while passing out; she woke up, regained consciousness,

62

engaged in sex again issue was whether you can give advance consent for something when
later unconscious; no bodily harm, so Jobidaw not issue
Decision: McLaughlin (all of the female judges on the Supreme Court agreed with the majority
reasons) law will never allow someone to provide advanced consent
Reasons: Court makes it clear must be conscious- parliament requires ongoing conscious
consent to ensure not subject of sexual exploitation; need to be capable of asking to stop and
revoke; if consent is voluntary agreement, consent must be specifically consented to each and
every act negates argument of broad advance consent
the Code makes it clear that an individual must be conscious throughout the sexual activity in
order to provide the requisite consent. Parliament requires ongoing, conscious consent to
ensure that women and men are not the victims of sexual exploitation, and to ensure that
individuals engaging in sexual activity are capable of asking their partners to stop at any
point. (3)
the consent of the complainant must be specifically directed to each and every sexual act,
negating the argument that broad advance consent is what Parliament had in mind. (34)
S. 273.1(2)(b) consent not obtained if the complainant is incapable of consenting to the
activity.
S. 273.1(2)(e) Parliament wanted people to be capable of revoking their consent at any
time during the sexual activity. (40)
The dissent disagrees, saying that such consent should not be precluded by law.
The provisions of the Criminal Code regarding consent to sexual contact and the case law
relied upon the Crown were intended to protect women against abuse by others. Their
mission was not to protect women against themselves by limiting their freedom to determine
autonomously when and with whom they will engage in the sexual relations of their choice.
Put differently, they aim to safeguard and enhance the sexual autonomy of women, and not to
make choices for them.
Points to certain absurdities that he says would result from the Majority position:
Adopting the Crowns position would also require us to find that cohabiting partners across
Canada, including spouses, commit a sexual assault when either one of them, even with their
express prior consent, kisses or caresses the other while the latter is asleep. (74)
Mc Laughlin says not going to use de minimus and not going to allow in marriage; 1985 got
rid of cant rape spouse; parliament should clean up
Currier, Consent and Fraud s. 265 (3)
Ratio: not any misrepresentation has to be misrepresentation that changes nature and
quality of the act
Fraud sufficient to vitiate consent, in the context of STD transmission) has two branches
A dishonest act (either falsehoods or failure to disclose status)
A deprivation (denying the complainant knowledge which would have caused her to refuse
sexual relations that exposed her to a significant risk of serious bodily harm)
Facts: What kind of fraud vitiates consent?
Decision:
Reasons:
Mabior - about clarifying and interpreting this test
Ratio: good test, but need more guidance - here there is a realistic possibility of transmission
of HIV, a significant risk of serious bodily harm is established, and the deprivation element of
the Cuerrier test is met. (84)

63

When is a realistic possibility of transmission negated?


on the evidence before us, the combined effect of condom use and low viral load precludes a
realistic possibility of transmission of HIV. (103)
Facts: HIV + individual managed illness; not telling sex partners; charged with 9 accounts of
aggravated sexual assault (exposed to risk to life so aggravated); no one contracted, but said
wouldnt consent if knew was positive; challenging Currier test significant risk too vague
Decision:
Reasons:
A standard of any risk, however small, would arguably set the threshold for criminal conduct
too low. On the other hand, to limit s. 265(3)(c) to cases where the risk is high might
condone irresponsible, reprehensible conduct. (87)
The law must strike a balance between this interest and the need to confine the criminal law
to conduct associated with serious wrongs and serious harms. Drawing the line between
criminal and non-criminal misconduct at a realistic possibility of transmission arguably strikes
an appropriate balance between the complainants interest in autonomy and equality and the
need to prevent the over-extension of criminal sanctions. (89)
R v Hutchinson
Ratio:
Facts: Poking holes in condom for women who has not consented to unprotected sex
Decision: Guilty fraud vitiating consent
Reasons:
Pappajohn v The Queen 1980 SCC - showing state of law before 1992 reform; accepted
subjective
Ratio: Mens rea for sexual assault is subjective so belief in consent needs to be honestly held
but NOT reasonable. An honest, but mistaken belief, of consent should lead to an acquittal. To
put forth a mistaken belief in consent, there must be some evidence or air of reality beyond
the mere assertion of the mistaken belief.
Facts: Accused charged with sexual assault after victim came running from house with tie
around neck and hands tied. Victim said she didnt consent at any time. Accused said that she
did consent and it was only when she was bound that there was no longer consent. Charged
with rape.
Issues: Is this defence available? Does mistake of fact have to be a reasonable mistake?
Decision: Appeal dismissed. Trial judge was correct in holding there was not sufficient
evidence to justify putting defence of mistake of fact forward to jury.
Reasons: McIntyre -- Defence of mistake of fact is available in Canada. However, to put forth
a mistaken belief in consent, there must be some evidence or air of reality beyond the mere
assertion of the mistaken belief. Dickson Mens rea for sexual assault is subjective so a belief
in consent needs to be honestly held but not reasonable.
Precedent: Critiqued deeply after it came out. Christine Boyle that kind of room for a
subjective, unreasonable belief gives a lot of room for regressive assumptions and myths to
happen. Eg. No means yes. Passive woman is consenting. Tony Pickerd doesnt make sense
as social policy approach would it be so wrong to impose a social responsibility to act
reasonably in sexual relations with another. Its protective of women and makes sense.
Reasonable steps should be required (which eventually does happen).
Note: At this time, no female judges!
R v Sansregret 1985 SCC
Ratio: Mistaken belief doesnt have to be reasonable, but you cant be wilfully blind. If youre

64

wilfully blind, you dont have an honest mistake of belief.


Facts: Accused charged with sexual assault after breaking into womens home. She admitted
to consenting to sex but did so to ensure her own safety. Accused had done this before and his
probation officer convinced victim not to press charges. Accused argued he didnt subjectively
know there was no consent.
Charge: Sexual assault requires as part of actus reus absence of consent. Accused must
subjectively know that there is no consent.
Decision: Guilty. He was wilfully blind and therefore had the knowledge.
Reasons: Given past history, he subjectively knew there was no consent and decided not to
inquire bc he knew the risks. One cannot honestly hold a mistaken belief if you are wilfully
blind as to the facts. He was subjectively aware of the likliehood that she was not consenting
but chose to not inquire.
Darrach
Ratio: The presumption of subjective mens rea requires if stigma offense; after 1992
amendment constitutionality issue; mistake of fact now has objective component; even if is
stigma, still has enough subjective component to be constitutional
Facts:
Issues:
Decision:
Reasons:
R v Ewanchuk 1999 SCC - lays out components of modern law of sexual assault test
Ratio:
1. Implied consent does not exist in context of sexual assault.
2. The actus reus should be determined from the perspective of the

complainant, and particularly through a consideration of whether the


complainant, in her mind, wanted the sexual touching to take place. Consent for
AR complainant, in her mind, consented to sexual touching.
3. Consent for MR complainant affirmatively communicated by words or conduct an
agreement to engage in sexual activity. Silence is not consent.
Facts: 17 yr old complainant was victim of sexual touching and said no four times. After each
time, he stopped. Trial judge found her to be credible. Accused argued that there was an
implied consent bc she went along with him, portrayed calm and didnt leave the trailer.
Issues: Is there anything called implied consent?
Decision: Guilty of sexual assault. She clearly did not consent and furthermore he did not
take reasonable steps to determine if consent was obtained after each time she said no.
Reasons: There is no such thing as implied consent. (see ratio)
LHeureux-Dube takes up McClungs statements. Reinforces the myth that youre less likely
to consent based on how youre dressed. Feeds background assumption that this kind of
conduct is natural or normal for women to deal with .
It is open to the accused to claim that the complainants words and actions, before and
during the incident, raise a reasonable doubt against her assertion that she, in her mind, did
not want the sexual touching to take place. If, however, as occurred in this case, the trial
judge believes the complainant that she subjectively did not consent, the Crown has
discharged its obligation to prove the absence of consent. (para 27)
Justice Major on Mistaken Belief in Consent:
Silence is not consent
Did the accused honestly believe that the complainant had communicated, in words or
conduct, consent? Any other belief, no matter how honestly held, is not a defence. (para. 64)

65

The belief cannot be the product of willful blindness


Once a person says no, one must seek affirmative consent to touch again
One must take reasonable steps to ascertain consent
McClung J. in Ewanchuk at the Alberta Court of Appeal
it must be pointed out that the complainant did not present herself to Ewanchuk or enter his
trailer in a bonnet and crinolines. reinforces myth that people worthy of belief or not and
dressing slutty
The advances were far less criminal than hormonal. this normalizes woman should
expect to be subjected to this; men just do this
In a less litigious age going too far in the boyfriends car was better dealt with on site a
well-chosen expletive, a slap in the face or, if necessary, a well directed knee. - normalizing;
cant be a victim of unless you fight back
INVOLUNTARINESS

Involuntariness involves accused NOT being in CONTROL of actions

CONSCIOUS
Accused was CONSCIOUS but NOT in control of actions

NOT CONSCIOUS
Accused was NOT CONSCIOUS (or impaired) and t/f NOT in control of actions

WHY?

This is a defence of simple involuntariness that negate the actus reus. Accused was conscious but action
cannot be fairly attributed to him.
Extreme Self-Induced Intoxication
Disease of Mind
Some cause other than self-induced intoxication or mental
disorder
Examples: physical twitch, spasm, accidental mvmt, physical compulsion by another (Larsonneur)
Daviault intoxication, intoxication akin to automatism
Considered mental disorder and will proceed under NCRMD provisions (s. 16)
Automatism proper (non-insane automatism)
Examples: physical blows (Bleta); sleepwalking (Parks), but unlikely after Stone/Luedecke; involuntary
intoxication (King); in very limited circumstances, psych blow (Rabey/ Stone)
Burden of proof: Crown must prove basic physical voluntariness beyond a reasonable doubt

Burden of proof: on accused on a balance of probabilities (REVERSE)


Burden of proof: Party raising the issue must satisfy s. 16 on a balance of probabilities

Burden of proof: Accused must prove automatism on a balance of probabilities (REVERSE) (After Stone
presumption is that accused is NCRMD, not a/t)

If Crown cannot, result is acquittal


S 33.1 now governs this and is a defence ONLY for general intent offences NOT involving interference
with bodily integrity.
If accused successful, result

is acquittal
If s. 16 is satisfied, the verdict will be NCRMD and the accused will be dealt with through that system (ie.
review board)

66

If accused can show automatism proper, the result is acquittal.

INTOXICATION
INTOXICATION (s. 33.1)
Summary
Negative defence (either no AR or MR)
Normal intoxication (No MR); Extreme Intoxication (No AR or MR) (like
automatism)
Results in acquittal if successful
Competing
Unjust to punish the morally innocent (no MR).
views on
However, dont want to concede to the person who gets drunk and
intoxication
commits a crime.
1. Is the
Argue for one or the other based on the circumstances involved.
offence
Classify it as either specific or general intent; then turn it over to the facts
specific
of the case.
intent or
general
General intent offence
intent?
MR is intending to apply physical force to your body. Nothing intended

2. Does this
involve Normal
or Extreme
Intoxication?
Rules of
Intoxication
Defence

beyond the physical act itself. Its a very simple mens rea. E.g. just
making body move
Assault, assault/bodily harm, assult/peace officer, B&E, crim
harassment, incest, indecent assault, manslaughter, mischief, sexual
assault, unlawful confinement, wilful obstruction/peace officer
Specific intent offences
MR involves some extra purpose that is a more complicated.
Look for: Language that requires planning or foresight, a specific type
of knowledge
Aiding and abetting, arson, assault/intent to resist arrest, assaulting
police officer, attempt at any offence, attempted murder, B&E,
murder, possession stolen prop, rec stolen prop, robbery, theft, sexual
exploitation, wilfully causing fire

Purpose of the specific/general distinction is not about logic but


more about policy driven distinctions.
1) Normal Intoxication
Intoxication affects the capacity to form MR.
2) Extreme Intoxication (Daviault)
Intoxication akin to automatism. No capacity to form AR or MR. Brain isnt
in control and no longer acting voluntarily.
Is it a specific intent offence?
1. For specific intent offences, normal intoxication is a defence. The
jury must acquit if there is a reasonable doubt as to whether the accused
formed the requisite MR. (George)
o Burden of proof is on the Crown beyond a reasonable doubt.
o D must establish an air of reality
Is it a general intent offence?
2. For general intent offences, the form of MR required is so simple and
basic that normal intoxication cannot raise a reasonable doubt as to MR.
However, extreme intoxication (akin to automatism) is a defence.

67

The burden of proof is on the accused on a balance of probabilities.


Expert evidence normally required. (Daviault)

Does it involved interference with bodily integrity of another


person?
Yes Statutory bar based on s. 33.1
3.

S. 33.1 of the
CCC

S. 33.1 - for general intent offences involving interference with


bodily integrity of another, even extreme intoxication is not an
available defence.
o For general intent offences involving interference with bodily
integrity of another, intoxication is NO DEFENCE at all.
(1) It is not a defence to an offence referred to in subsection (3) that the
accused, by reason of self-induced intoxication, lacked the general intent or
the voluntariness required to commit the offence, where the accused
departed markedly from the standard of care as described in subsection (2).
(2) For the purposes of this section, a person departs markedly from the
standard of reasonable care generally recognized in Canadian society and is
thereby criminally at fault where the person, while in a state of self-induced
intoxication that renders the person unaware of, or incapable of consciously
controlling, their behaviour, voluntarily or involuntarily interferes or
threatens to interfere with the bodily integrity of another person.
(3) This section applies in respect of an offence under this Act or any other
Act of Parliament that includes as an element an assault or any other
interference or threat of interference by a person with the bodily integrity of
another person.

DPP v Beard 1920 HL


Ratio: Intoxication can be used as a defence where drunkenness renders the accused
incapable of forming the mens rea (to foresee or intend) Rationale: if you dont have the
requisite MR, its unjust to punish you.
Facts: Beard drunk while he killed a woman in the course of rape.
Issues: Can intoxication be used as a defence.
Decision: Guilty
Reasons: Drunkenness was no defence in this case bc Beard was not so drunk at the time
that he was incapable of forming the intent to commit the crime.
Precedent: Until 19th century, intoxication was not a defence viewed as an aggravating
factor. Getting drunk was blameworthy in itself. This view relaxed over time and with Beard
new principle developed
EXPLAINING RULE 1 NORMAL INTOXICATION A DEFENCE TO SPECIFIC INTENT
OFFENCES.
R v George 1960 SCC
Ratio: Normal intoxication is never a defence for general intent offences. For specific intent
offences, normal intoxication is a defence and can raise a reasonable doubt as to MR.
Facts: George was drunk, committed robbery and argued defence of intoxication. Trial judge
acquitted him and also decided not to convict him of the lesser offence of assault. Crown
appealed and argued for a distinction bw crimes where there is simple MR (assault) and those
with added MR (robbery).
Charge: Robbery (AR: assault MR: intent to steal)
Issues:
Decision: Acquitted of robbery; convicted of assault.

68

Reasons: In general intent offences, there is a simple form of MR intent to move body. In
specific, it involves ulterior motive to an extra end or goal. The mental element for general
intent crimes is so minimal that normal intoxication can never be a defence. For specific intent
offences, MR is complicated enough that intoxication could interfere with MR. Here, robbery =
specific and assault = general therefore intoxication is no defence.
a distinction is to be drawn between intention as applied to acts done to achieve
an immediate end on the one hand and acts done with the specific an ulterior motive
and intention of furthering or achieving an illegal object on the other hand. (bottom of
page 850-51 in RBHS)
The former acts may be the purely physical products of momentary passion whereas
the latter involve the mental process of formulating a specific intent. (pp. 850-51 in
RBHS)
EXPLAINING RULE 2 DAVIAULT INTOXICATION IS A DEFENCE TO GENERAL INTENT
OFFENCES
R v Bernard 1988 SCC
Ratio: General and specific intent offence distinction does exist.
Facts: Sexual assault causing bodily harm.
Issues: Does the general and specific intent offence distinction exist? Is there a bar on using
extreme intoxication as a defence?
Decision: New trial ordered.
Reasons: Split 2/2. 1) The MR for getting that drunk and being reckless should be substituted
for MR of the offence. Therefore, shouldnt allow defence of extreme intoxication. 2) Wilson &
LHeureux-Dube Substituion is not right bc it matches the guilty the mind for getting drunk
with the guilty act for another. Drunkenness akin to automatism should be a defence for
general intent offences.
Precedent: Situation after this unclear bc 2 and 2. Law clarified in Daviault.
R v Daviault 1994 SCC
Ratio: Extreme intoxication is a defence to general intent offence. If the accused can prove on
a balance of probabilities that they were in such an extreme degree of intoxication that they
were in a state akin to automatism, then it can be considered as a defence (reverse BOP).
They must be incapable of having even the minimal intent required for a general intent
offence. Expert evidence will normally be required.
Facts: Accused charged with sexual assault of elderly women. Consumed 8 bottles of beer
and 40 of brandy. Blood alcohol was 0.4-0.6. Medical experts testified that at that level of
drunkenness he should be dead. If a person didnt die of this, it would lead someone to be in a
dissociative state.
Issues: Is extreme intoxication available as a defence?
Decision: Extreme intoxication is a defence for general intent offences.
Reasons: Substitution analysis from Bernard rejected. Burden of proof on the accused to
establish this intoxication akin to automatism on a balance of probabilities. Expert evidence
will normally be required.
Dissent: They would have allowed the substitute MR bc getting that drunk is so reckless.
Precedent: Defence is out there but rarely used.
EXPLAINING RULE 3 DAVIAULT INTOXICATION NOT A DEFENCE TO CERTAIN
GENERAL INTENT OFFENCES
Parliament was concerned with the effects of Daviault intoxication, esp on women and
children, and people being able to use extreme drunkenness as a defence, so they enacted
new legislation stating that for general intent offences, extreme intoxication is not a

69

defence for acts involving bodily interference of another person.

MENTAL DISORDER
At the end of a trial, three things take place:

There is a finding of guilt a verdict is rendered

The verdict is entered a conviction is entered

A sentence is passed or consequences are imposed.

MENTAL DISORDER (s. 16)


Negative defence (No AR or No MR)
Mental d/o negates the MR
Excuse
Not Criminally
Prior to 1991, defence was not guilty by reason of insanity and would
Responsible by
result in an acquittal. Held indefinitely until Swain ruled that violation of s.
reason of
7.
Mental
Parliament created NCRMD and new regime with goals of public safety
Disorder
and treatment.
(NCRMD)
If found
Disposition hearing by review board. Accused will be (s. 672.54):
NCRMD, a
o Released (discharged) without conditions;
disposition
o Released (discharged) subject to appropriate conditions; or
hearing is
o Detained in custody in a hospital subject to appropriate conditions.
held before the The disposition will turn on factors listed in s. 672.54:
court or a
o Need to protect the public, Mental condition of the accused,
specially
Reintegration of the accused, Other needs of the accused.
constituted
If person found NCRMD, can be held in an institution indefinitely
review board.
Has the effect that defence wont necessarily raise the defence bc person
(s. 672.54
could spend less time incarcerated if they plead guilty and do time.
1. What is a
Mental Disorder means a disease of the mind (s. 2 of the Criminal
disease of
Code)
mind?
Any illness, disorder, or abnormal condition which impairs the human
mind and its functioning Excluded, however, are: self-induced states
caused by alcohol or drugs, as well as transitory mental states such as
hysteria or concussion. (Cooper)
Should be a broad meaning (Cooper)
Disease of mind is a legal concept, although it includes a medical
component. Judge determines whether the condition constitutes a disease
of the mind. (Simpson)
Who can raise
Accused can whenever they wish up until the point that a conviction is
defence and
entered (ie. after verdict given but before conviction).
when?
Crown cannot raise MD until accused has put his mental health at issue
in the trial OR in the space bw rendering of verdict and entering of
conviction (Swain) Rationale:
Accused has the right to be found guilty or not guilty of the charge
regardless of if they have a mental illness.
If MD brought up, early might increase assumption of guilt.
Burden of Proof 16(3) The burden of proof that an accused was suffering from a mental

70

TEST of
NCRMD (s. 16
of CCC)

disorder so as to be exempt from criminal responsibility is on the party that


raises the issue.
Reverse burden. If the Crown wants to raise mental disorder, then burden on
them. Its on a balance of probabilities
16.(2) Presumption is no mental disorder
16 (3) REVERSE BURDEN: Burden of proof on party raising the issues on a
balance of probabilities.
16 (1) Did the disorder render the accused incapable of appreciating the
nature and quality of the act or omission OR of knowing that it was
wrong? (s. 16(1))
A) To appreciate the nature and quality of the act means
1) knowledge of the act AND
2) capacity to measure and foresee the consequences. (Cooper)
Does not mean:
o Having appropriate emotional feelings or remorse. (Kjeldsen)
Psychopaths are then guilty and should be convicted and
punished.
o Knowing the penal consequences (Abbey)
o Irresistible impulses

s. 16 of CCC

B) To determine whether accused was capable of knowing the act was


wrong
ASK: Whether, owing to a disease of the mind, an accused was incapable
of rationally evaluating whether the act was wrong in either a legal
sense OR as a matter of societys moral views (Chaulk and Morrissette;
Oommen; Landry).
16(1) No person is criminally responsible for an act committed or an
omission made while suffering from a mental disorder that rendered the
person incapable of appreciating the nature and quality of the act or
omission OR of knowing that it was wrong.
(2) Every person is presumed not to suffer from a mental disorder so
as to be exempt from criminal responsibility by virtue of subsection (1), until
the contrary is proved on the balance of probabilities. (this is the
presumption of sanity)
(3) The burden of proof that an accused was suffering from a mental
disorder so as to be exempt from criminal responsibility is on the party that
raises the issue. (reverse burden. If the Crown wants to raise mental
disorder, then burden on them. Its on a balance of probabilities)

1. WHAT IS A DISEASE OF THE MIND?


R v Simpson 1977 Ont CA
Ratio: Disease of mind is a legal concept, although it includes a medical component. Judge
determines whether the condition constitutes a disease of the mind.
Facts: Accused appealed conviction of attempted murder on grounds of insanity.
Issues: Does accuseds personality disorder constitute a disease of the mind within the
meaning of s. 16?
Decision: Personality disorders are included in the definition of disease of mind in s. 16 of
CCC.

71

Reasons: Split court. Criminal law is making claims about MD for own purposes.
Cooper v The Queen 1980 SCC
Ratio: Definition of disease of mind is: any illness, disorder, or abnormal condition which
impairs the human mind and its functioning Excluded, however, are: self-induced states
caused by alcohol or drugs, as well as transitory mental states such as hysteria or
concussion. To appreciate means knowing what youre doing and also appreciating its
consequences.
Facts: Accused choked victim to death and claimed defence of insanity.
Reasons: Dickson -- Disease of mind is to be given a broad meaning (see above). Must ask if
accused, by reason of disease of the mind, was deprived not only of the mental capacity to
know what one is doing but also the mental capacity to foresee and measure the
consequences of the act. The word appreciate imports an additional requirement to mere
knowledge of the physical quality of the act. The requirement, unique to Canada, is that of
perception, and ability, to perceive the consequences, impact, and results of a physical act.
2. WHAT DOES IT MEAN TO APPRECIATE THE NATURE AND QUALITY OF ONES ACTS
OR OMISSIONS?
R v Abbey 1982 SCC
Ratio: Appreciate means appreciating what you are doing but also the consequences.
Appreciating the consequences does NOT include knowing the penal consequences (ie. that
you might be punished).
NOTE: Kjeldsen appropriate feelings not included
Facts: Accused charged with importing cocaine for purpose of trafficking. Suffered from
hypomania. He appreciated what he was doing, but had a delusion that if caught he couldnt
be punished. Crown appealed
Issues: Does appreciate mean knowing the penal consequences?
Decision: NO appreciation of penal consequences not included
Reasons: A delusion that renders accused incapable of appreciating the penal consequences
does not go to the MR of the offence and does not render him incapable of appreciating the
nature and quality of the act and therefore does not bring the insanity defence into effect.
3. WHAT DOES IT MEANT TO KNOW THAT AN ACT OR OMISSION IS WRONG?
Wrong could be:

Illegal and/or

Immoral and if immoral

Do you view it as wrong?


Do you appreciate that society views it as wrong?

R v Chaulk and Morrisette 1993 SCC


Ratio: Wrong is concerned with whether the individual knew the act was immoral by societal
standards. Deviant moral code must be connected to the disease of the mind.
Facts: Two accused B&E and then killed someone. Knew it was legally wrong but paranoid
psychosis made them think they had moral authority to do it bc they ruled the world and it
was necessary. Reverse burden of proof was challenged. Accused argued reverse burden
violates s. 7.
Decision: Reverse burden does violate s. 7 but is justified under s. 1.

72

Reasons: Lamer - It would be an impossible burden for the Crown. For mental disorder,
wrong concerned with whether the individual knew the act was immoral. If we are going to
say you are morally blameworthy, then need to know if people can appreciate if they know its
morally wrong bc of disease.
Dissent: Wilson Breach and not justified.
R v Oommen 1994 SCC
Ratio: The crux of the inquiry is whether the accused lacks the capacity to rationally decide
whether the act is right or wrong from a societal standpoint, and hence to make a rational
choice about whether to do it or not. If lacks that capacity, then defence of insanity applies.
Facts: Accused was in a paranoid psychosis. He knew the difference bw right and wrong but
came to believe that his roommate was part of conspiracy to kill him. He murdered her after
interpreting the buzzer as her signal to kill him.
Decision: Not guilty.
Reasons: Must possess at least capacity to know right and wrong but must also be able to
apply that knowledge in a rational way to your circumstances. Right and wrong is from
societys standpoint. Here, mental disorder deprived him of capacity to know his act was
wrong by standard of ordinary person.
R v Landry 1980 Que CA
Ratio: Knowing means that accused needs capacity to know that something s morally wrong
in the circumstances
Facts: Paranoid schizophrenia who thinks he is God kills his friend who he believes to be
Satan.
Decision: Not guilty
Reasons: Accused, by his God delusion, was rendered incapable of knowing the act was
morally wrong in the circumstances.

Determining whether an accused was incapableof knowing that it was wrong means asking
whether, owing to a disease of the mind, an accused was incapable of rationally evaluating whether
the act was wrong in either a legal sense or as a matter of societys moral views.

AUTOMATISM
AUTOMATISM
What is the
type of
defence?
Burden of Proof

Two forms of
automatism
recognized at
law:

Negative defence (No AR or no MR)


Negates the AR
Results in acquittal
Reverse burden: Accused must establish the defence on a balance of
probabilities (Stone)
Possible evidence: involuntariness, psychological ev, severity of trigger,
bystanders, states of a/t at other times, motive for crime exists, other
explanation.
1. Rationale: accused can easily fake defence; create consistency with
Daviault.
2. Mental Disorder automatism
Involuntary action that results from a disease of the mind
Results in review board, NCRMD, not acquitted
3. Non-Mental Disorder automatism
A state of impaired consciousness, rather than unconsciousness, in

73

Will it be
NCRMD or
automatism
proper?

Consequences

which an individual, though capable of action, has no voluntary control


over that action (recognized in Rabey; Stone)
Results in acquittal
Can arise from:
o Physical blow
o Physical condition (e.g. stroke, diabetes)
o Psychological blow (Rabey)
Contextual objective test: Trigger of involuntary conduct
must be an ordinary event that could send a normal person in
similar circumstances into an automatic state. (Rabey)
Two-stage test (Stone):
1. The judge must assess whether a proper foundation for a defence of
automatism has been established.
a. Is there evidence on the record that person an air of reality to the
defence of automatism?
2. If this evidentiary foundation has been laid, the judge must determine
whether the condition is one of mental disorder or non-mental
disorder automatism.
a. Consider for 2:
4 Principles (Stone)
1. If the impaired consciousness resulting in involuntary conduct arises from
a disease of the mind, then the proper defence is mental disorder,
not automatism.
2. Judges should start from the presumption that the condition the
accused claims to have suffered is a disease of the mind, and then
decide whether the evidence takes the condition out of the disease of the
mind category.
3. A holistic approach should be taken by the trial judge in deciding whether
it is a disease of the mind. This holistic approach is informed by:
The internal cause factor
o If internal, then NCRMD
The continuing danger factor
o If continuing danger, then NCRMD
When not clearly internal cause and no continuing danger, judge can
consider policy factors in deciding bw automatism and NCRMD. (e.g.
defence is easily faked, public upset at an outright acquittal).
All three are considered and then a determination is made.
4. Instances of psychological blow automatism will be very rare, requiring
an extremely shocking trigger that might cause a normal person to
enter into a state of automatism. (Stone, Rabey)
Contextual objective test: would a similarly situated ordinary person
react?
Policy factors small chance of jury finding automatism proper bc it
seems unfair to just give someone an acquittal and doesnt sit right
with ppl. Rather have NCRMD where there is a disposition hearing

Rabey v The Queen 1980 SCC defense recognized


Ratio: Automatism is recognized as an unconscious, involuntary act where the mind does not
go with what is being done. Objective test for psychological blow automatism: Trigger of
involuntary conduct must be an ordinary event that could send a normal person in similar
circumstances into an automatic state. Subjective test used for physical blow and reaction to
drugs. NORMAL CRIM BURDEN APPLIES

74

Facts: Accused assaulted woman with a rock after being informed they were just friends.
Accused argued he was in a state of automatism caused by the emotional blow.
Issues: Should there be a defence of insane or non-insane automatism?
Decision: Defence of non-insane automatism is not available to accused. Caused by disease
of mind therefore NCRMD
Reasons: Court split 4/3 on whether it was a disease of mind or automatism proper. Majority:
Action caused by a disease of the mind and therefore NCRMD. Where psychological blows are
the purported cause, a dissociative state cannot be from an ordinary stress and
disappointment of life. Automatism proper could only arise from a psych blow if it was an
extraordinary event that would cause a normal, average person to go into dissociative
state (e.g. seeing a loved one killed). Here, must be caused by factors internal to accused and
would have to be bc of a mental disorder.
Dissent: Dickson Medical evidence did not suggest a mental disorder. Should not be an
objective test. Must look at mind of accused. Best way to tell which it is is by looking at if
there is a continuing danger.
R v Parks 1992 SCC
Ratio: Defence of automatism proper is embraced. Can use either internal/external theory or
continuing danger theory to conclude that the proper defence is NCRMD and not automatism.
Normal criminal burdens still seem to apply.
Facts: Parks attacked parents-in-law while sleepwalking. Killed mother and seriously injured
father in law. Argued automatism proper.
Issues: Is sleepwalking included within the defence of automatism? Is it a disease of the
mind?
Decision: Acquitted. Automatism proper.
Reasons: Sleepwalking here was not a disease of the mind, but was automatism proper.
R v Stone 1999 SCC
Ratio: 1) Definition: Automatism is a state of impaired consciousness, rather than
unconsciousness, in which an individual, though capable of action, has no voluntary control
over that action. 2) The key question is voluntariness 3) Burden of proof: The accused must
convince the trier of fact of automatism on a balance of probabilities and, although this limits
the accuseds rights under s. 11(d), it is justified.
Will it be NCRMD or automatism proper?
Two-stage test:
1. The judge must assess whether a proper foundation for a defence of automatism has been
established.
2. If this evidentiary foundation has been laid, the judge must determine whether the
condition is one of mental disorder or non-mental disorder automatism.
When is it mental disorder and when is it automatism? 4 Principles
1. If the impaired consciousness resulting in involuntary conduct arises from a disease of the
mind, then the proper defence is mental disorder, not automatism.
2. Judges should start from the presumption that the condition the accused claims to have
suffered is a disease of the mind, and then decide whether the evidence takes the condition
out of the disease of the mind category. (Prof: this is a radical contraction of the defence)
3. A new, holistic approach should be taken by the trial judge in deciding whether it is a
disease of the mind. This holistic approach is informed by:
The internal cause factor
o If internal, then NCRMD

75

The continuing danger factor


o If cont danger, then NCRMD
And, when it is not clearly an internal cause nor is there a continuing danger, policy
concerns underlying this Automatism/NCRMD split.
o A judge can decide to call it a DOM if there are policy concerns to do so (e.g.
defence is easily faked, public upset at an outright acquittal). Although it might
look like a/t proper, reasons will lead to NCRMD
All three are considered
4. Instances of psychological blow automatism will be very rare, requiring an extremely
shocking trigger that might cause a normal person to enter into a state of automatism.
This is a contextual objective test.
Facts: Accused stabbed wife 47 times after she made insulting comments about his sexual
ability, his children and ex-wife. Argued defences of non-mental disorder automatism and
mental disorder automatism. Judge only left the defence of mental d/o automatism to jury.
Issues: How do you decided bw automatism proper and NCRMD? Which is the appropriate
defence to put forward to jury? Was leaving only mental disorder defence to jury justified?
Decision: Trial judge did not err in refusing to instruct jury on non-mental d/o automatism.
Stone got pushed into NCRMD (even tho evidence clearly showed that was no disease of the
mind).
Reasons: Bastarache -- Starting proposition for trial judges is that involuntary and automatic
behaviour was caused by a disease of the mind. They should only instruct the jury about noninsane automatism if there is evidence that establishes an air of reality for the defence.
Objective test for the trigger from Rabey does not violate ss. 7 and 11(d) of the Charter. In
determining whether its insane or non-insane, trial judge should consider policy factors
including public safety. Burden of Proof on accused on a balance of prob.
Precedent: Suggests a strong preference for finding NCRMD where accused establishes that
he was in a dissociative state and acted involuntarily. This is a radical constriction of
automatism proper. Claims now being funnelled through NCRMD.
Post-Stone Cases
R. v. Campbell (2000 Ont SCJ)
Sleepwalking is a DOM and went through NCRMD. Different from Parks
R. v. Jiang (2007 BC CA)
Fell asleep while driving from chronic insomnia. Court found non-insane a/t. Not allowed to
drive and danger removed.
R. v. Luedecke (2008)
Sexual assault will sleepwalking. Treated as a mental d/o
R. v. Graveline (SCC, 2006)
Successful automatism claim. Accused abused by husband over 32 years. On night in
question, abusive interactions and later she shot him while he was in bed. She claimed a/t and
didnt remember. Incident wasnt extremely shocking. Evidence showed she suffered from
battered woman syndrome. SCC said it was ok to leave non-insane a/t to jury. Shows that
courts will still exercise their discretion on whether or not to put the defence forward.

PROVOCATION
PROVOCATION (s. 232)
What type of
Affirmative defence
defence?
AR and MR are there but shouldnt be punished.
Only a defence to murder. It does not apply to any other crime.
Partial defence. If accepted, it reduces charge from murder to
manslaughter

76

Burden of Proof
s. 232 of CCC

Crown prove elements of offence beyond a reasonable doubt once an air of


reality is established.
232. (1) Culpable homicide that otherwise would be murder may be reduced
to manslaughter if the person who committed it did so in the heat of passion
caused by sudden provocation.
(2) A wrongful act or an insult that is of such a nature as to be sufficient to
deprive an ordinary person of the power of self control is provocation for the
purposes of this section if the accused acted on it on the sudden and before
there was time for his passion to cool.
Certain situations or types of acts excluded by statute:

Three
Elements of
Provocation

Relevance of
Culture
Rationale

Critiques

232(3) no one shall be deemed to have given provocation to another by


doing anything that he had a legal right to do, or by doing anything that the
accused incited him to do in order to provide the accused with an excuse for
causing death or bodily harm to any human being.
Legal right really means something sanctioned by law, such as a
sheriff proceeding to execute a legal warrant, or a person acting in
justified self-defence. (Tran, para. 27.)
It does not not have to be an illegal act to be a wrongful act or insult.
For a successful defence of provocation to a charge of murder, three things
have to be proven:
1. The accused was subject to a wrongful act or insult sufficient to
deprive an ordinary person of the power of self-control
Modified objective test.
o Subjective feature: Age (Hill), gender (Hill), past relationship,
racial slurs (Thibert)
o Objective: ordinary person
Subjective features inform the gravity of the insult, not the level of
self-control we expect of the ordinary person (Tran)
2. The accused did, in fact, lose self-control in response to this wrongful act
or insult (subjective element); and
3. The accused did so on the sudden and before there was time for his
passion to cool.
Suddenness has two components:
1) the insult or act must be sudden AND
2) the response must be sudden.
Nahar leaves open door that culture could be considered; Humaid
Cultural background is not part of the ordinary person standard; would be
relevant to the insult portion
The defence of provocation does no more than recognize human
frailties. (Thibert)
The law recognizes that, as a result of human frailties, the accused
reacted inappropriately and disproportionately, but understandably to a
sufficiently serious wrongful act or insult. (Tran (2010), para. 22)
Theory is that people who act in heat of the moment are less
blameworthy. (ie. excuse)
Defence privileges the emotions of anger/rage. Dont privilege other
emotions.

77

Used as a defence by men who kill their wives in murderous rage.

R v Hill 1986 SCC


Ratio: Factors such as age and gender of the accused could be relevant to ordinary person.
Facts: Hill, 16 yr old male, charged with murder of another male after sexual advances.
Appealed on grounds that the trial judge failed to properly instruct the jury on the meaning of
ordinary person in s. 232(2)
Issues: Should jury have been instructed to consider ordinary person as a 16 yr old male?
Decision: Conviction upheld. Trial judges charge to the jury on ordinary person was correct.
Reasons: Dickson -- It was not necessary for the judge to direct the jury that the ordinary
person means an ordinary person of the same age and sex as the accused. It should be
assumed they would do that.
Dissent: Wilson Trial judge should have included an ordinary person of same age as
accused. Reflects reality that youth in development are not held to same degree of
responsibility as adult actors. Appropriate standard is 16 yr old male subject to a homosexual
assault. Sex shouldnt determine the amount of control expected.
R v Thibert 1996 SCC
Ratio: Ordinary person must be of same age, sex and other factors as accused. History of
the interaction may be relevant, including nature of the relationship, which will go to gauge
the gravity of the insult.
The wrongful act or insult must be one which could, in light of the past history of the
relationship between the accused and the deceased, deprive an ordinary person of the same
age, and sex and sharing with the accused such other factors as would give the act or insult in
question a special significance, of the power of self-control. (para. 19) Is culture to be
factored in?
Facts: Wife had an affair with co-worker and husband knew. Accused came armed to work to
speak to wife alone. Killed lover. Raised defence of provocation bc lover taunted him to shoot
him.
Issues: Was the trial judge correct in leaving the defence of provocation with the jury?
Decision: Acquitted.
Reasons: Cory Ordinary person here is married man faced w breakup of marriage. This
ordinary man could have lost self control when deceased taunted him while holding accuseds
wife.
Dissent: Major breakup of marriage should not be considered act that would prompt an
ordinary person to lose self-control. Provocation was not sudden bc accused knew of wifes
affair with deceased and had seen them together that day.
R v Nahar BC CA
Ratio: Role of culture in ordinary person standard.
The question is whether, having regard for the cause and duration of the couple's troubled
relationship, an insult that carried the same emotional impact for an ordinary young married
man of the same cultural background as it apparently carried for Mr. Nahar, would cause such
a man to lose his power of self-control.
Is culture to be factored in?
Facts: Nahar killed his wife bc she was not obeying the Sikh cultural rules by smoking,
drinking and seeing other men. Nahar argued provocation. Trial judge did not consider cultural
factors.
Issues: Should the trial judge have considered cultural background in the determination of

78

whether the ordinary person in that situation would lose self control.
Decision: Trial judge did not err.
Reasons: It was not clear the the judge didnt consider the Sikh cultural values, and even
taking that into account, the ordinary person would not in the circumstances have lost self
control.
R v Humaid Ont CA
Ratio: Role of culture in ordinary person standard.
Facts: Humaid killed wife after becoming concerned that she was having affair with colleague.
Claimed provocation. Defence raised his Islamic culture, which they characterised as male
dominated and concerned with family honour. Accordingly infidelity by woman was a serious
violation of family honour and worthy of punishment by male. Jury was not instructed to
consider cultural background in ordinary person.
Issues: Should jury have considered cultural background in ordinary person?
Decision: Trial judge did not err.
Reasons: No air of reality to provocation defence, but even if there were cultural background
would not be considered in ordinary person. Alleged beliefs which give the insult added gravity
are premised on the notion that women are inferior to men and that violence against women is
in some circumstances accepted, if not encouraged. These beliefs are antithetical to
fundamental Canadian values, including gender equality. It is arguable that as a matter of
criminal law policy, the "ordinary person" cannot be fixed with beliefs that are irreconcilable
with fundamental Canadian values.
[93] In this case, however, the appellant's religious and cultural beliefs are not the
target of the alleged insult. Rather, the appellant's religious and cultural beliefs are said
to render the words spoken by Aysar highly insulting. The difficult problem, as I see it, is
that the alleged beliefs which give the insult added gravity are premised on the notion
that women are inferior to men and that violence against women is in some
circumstances accepted, if not encouraged. These beliefs are antithetical to fundamental
Canadian values, including gender equality. It is arguable that as a matter of criminal law
policy, the "ordinary person" cannot be fixed with beliefs that are irreconcilable with
fundamental Canadian values. Criminal law may simply not accept that a belief system
which is contrary to those fundamental values should somehow provide the basis for a
partial defence to murder.

R v Tran 2010 SCC


Ratio: Adultery is not considered an insult sufficient for the ordinary person to lose self
control.
Facts: Tran killed ex-wifes new lover and seriously injured her after learning she had a new bf.
Issues: Was the new relationship an insult sufficient to deprive the accused of self control?
Decision: 2nd degree murder conviction upheld.
Reasons: No air of reality to the defence of provocation. This was not an insult and Tran did
not act on the sudden. He went there after suspecting the relationship. Only behavior which
comports with contemporary societys norms and values will attract the laws compassion.
(para. 30)

an individualized approach ignores the cardinal principle that criminal law is concerned with
setting standards of human behaviour. (Tran, para. 34)

79

It follows that the ordinary person standard must be informed by contemporary norms of
behaviour, including fundamental values such as the commitment to equality provided for in the
Canadian Charter of Rights and Freedoms. For example, it would be appropriate to ascribe to the
ordinary person relevant racial characteristics if the accused were the recipient of a racial slur,
but it would not be appropriate to ascribe to the ordinary person the characteristic of being
homophobic if the accused were the recipient of a homosexual advance. (para. 34)
Similarly, there can be no place in this objective standard for antiquated beliefs such as
adultery is the highest invasion of property or indeed for any form of killing based on such
inappropriate conceptualizations of honour. (para. 34)
R. v. Tran (2010 SCC 58)
The regulative function of the ordinary person standard:
Only behavior which comports with contemporary societys norms and values will attract
the laws compassion. (para. 30)
The ultimate test
Someone otherwise guilty of murder will have the offence reduced to manslaughter on the basis
of the defence of provocation if:

The accused was subject to a wrongful act or insult which could in light of the past history
of the relationship between the accused and the deceased, deprive an ordinary person of
the same age, and sex and sharing with the accused such other factors as would give the
act or insult in question a special significance, of the power of self-control,

The accused did, in fact, lose self-control in response to that act or insult, and

The accused did so on the sudden, without time for his passion to cool.

SELF DEFENCE
SELF DEFENCE (s. 34)
Affirmative defence
AR and MR are there but shouldnt be punished.
Classic case of justification.
Results in acquittal
At CL, you can respond proportionately to a threat to your safety and you have a defence of
self defence.
Burden of Proof Crown prove elements of offence beyond a reasonable doubt once an air of
reality is established.
s. 34 of CCC
34. (1) A person is not guilty of an offence if
(passed on Nov (a) they believe on reasonable grounds that force is being used against
22, 2012)
them or another person or that a threat of force is being made against them
or another person;
Subjective element

80

(b) the act that constitutes the offence is committed for the purpose of
defending or protecting themselves or the other person from that use or
threat of force; and
(c) the act committed is reasonable in the circumstances.
Basically, if you reasonably believe that the threat of force is
being applied to you, you can respond to that in a reasonable
fashion.
34(2) In determining whether the act committed is reasonable in the
circumstances, the court shall consider the relevant circumstances of the
person, the other parties and the act, including, but not limited to, the
following factors:
(a) the nature of the force or threat;
(b) the extent to which the use of force was imminent and whether there
were other means available to respond to the potential use of force;
(c) the persons role in the incident;
(d) whether any party to the incident used or threatened to use a weapon;
(e) the size, age, gender and physical capabilities of the parties to the
incident;
(f) the nature, duration and history of any relationship between the parties to
the incident, including any prior use or threat of force and the nature of that
force or threat;
(f.1) any history of interaction or communication between the parties to the
incident;
(g) the nature and proportionality of the persons response to the use or
threat of force; and (h) whether the act committed was in response to a use
or threat of force that the person knew was lawful.
34(3) Subsection (1) does not apply if the force is used or threatened by
another person for the purpose of doing something that they are required or
authorized by law to do in the administration or enforcement of the law,
unless the person who commits the act that constitutes the offence believes
on reasonable grounds that the other person is acting unlawfully.
1. Does the accused believed (subjective) on reasonable grounds (objective)
that force or threat of force is being used against them or another
person? (34(1)(a))
2. Was the act committed for the purpose of defending or protecting the
accused or the other person from that use or threat of force? (34(1)(b))
3. Was the act committed reasonable in the circumstances? (34(1)(c))
a. 34(2) In determining whether the act committed is reasonable in
the circumstances, consider the relevant circumstances of the
person, the other parties and the act, including, but not limited to,
the following factors:
(a) the nature of the force or threat;
(b) the extent to which the use of force was imminent and whether there
were other means available to respond to the potential use of force;
(c) the persons role in the incident;

81

(d) whether any party to the incident used or threatened to use a weapon;
(e) the size, age, gender and physical capabilities of the parties to the
incident;
(f) the nature, duration and history of any relationship between the parties to
the incident, including any prior use or threat of force and the nature of that
force or threat;
(f.1) any history of interaction or communication between the parties to the
incident;
(g) the nature and proportionality of the persons response to the use or
threat of force; and
(h) whether the act committed was in response to a use or threat of force
that the person knew was lawful.

This is a process of contextualizing the situation to see whether


actions taken are reasonable.
- 34(2)(b) imminence isnt a requirement under (1) its just a
consideration!
- 34(2)(h)Whether the act committed was in response to a use
or threat of force that the person knew was lawful.
o This used to be a bright line in the old provisions, not just
something to be considered but the determining factor.
o Used to be, couldnt claim self-defense resulting in the
defense of someone if there were other options
o Now, its no longer an exclusion just something to be
considered.
34(3) Subsection (1) does not apply if the force is used or threatened by
another person for the purpose of doing something that they are required or
authorized by law to do in the administration or enforcement of the law,
unless the person who commits the act that constitutes the offence believes
on reasonable grounds that the other person is acting unlawfully.

^^ If a person apprehends a use of force, but its coming from a law


officer, a person has to assume its a lawful use of force.
R v Whynot NSCA 1983
Ratio: no person has the right in anticipation of an assault that may or may not happen, to
apply force to prevent the imaginary assault.
R v Cinous SCC 2002
Ratio: There must be evidence on each element of the defence in order for there to be an air
of reality. Only then should the defence be put to a jury. Ask: Is there evidence on the record
upon which a properly instructed jury acting reasonably could acquit? Ratio re self defence:
must reasonably believe you are subject to a threat of force and have no other means of
protecting yourself.
Facts: Accused charged with murder having shot the deceased in back on head outside a gas
station. He was in a gang and given what he knew about the situation, he became concerned
that he was the target of the killing.
Decision: Guilty.
Reasons: No air of reality to defence of self defence. You dont actually have to have an
assault happen just need a reasonable apprehension of an assault of a threat of an assault.

82

Here, it doesnt matter whether they were actually going to assault Cinous bc he believed on
reasonable grounds that there was a threat of assault. Must be no other means of protecting
yourself. Here, the court rule that he had other options available, including calling the police
and fleeing.

o KEY Q: Did the accused believe, in the situation, that he was about to be
assaulted?
o TEST: Objective/Subjective
Q1: Did the accused reasonably believe he was being
unlawfully assaulted?
Subjective: did accused believe that he was being unlawfully
assaulted?
Objective: if so, was accuseds belief reasonable?
Q2: Did the accused reasonably believe that he faced death
or grievous bodily harm from the attack?
Subjective: did accused believe that he faced death or
grievous bodily harm from the attack?
Objective: if so, was accuseds belief reasonable?
Q3: Did the accused reasonably believe that he could not
preserve himself except by shooting the victim (ie. that there
was no alternative course of action available)?
Subjective: did accused believe that he could not preserve
himself except by shooting the victim?
Objective: if so, was the accuseds belief reasonable? Failed
this one.
o Court: Cinous reasonably apprehended imminent assault.
o HOWEVER, 34(2)(b): Q: Are there other means of responding to the
potential use of force?
A: Before, you couldnt claim the defense if there were other means of
avoiding the force under new provisions, its just a consideration towards
the reasonableness of the view of needed self-defense.
HERE: Court: there were other options available.
Cinous had gone into the gas station, bought wiper fluid, put in
the wiper fluid, borrowed money from one of the guys, and
THEN shot someone in the back of the head.
Court: YOU COULDVE CALLED THE COPS, YA FOOL. You
couldve run!
Cinous: I dont trust the cops! I don't think theyre reliable or
fast enough, and that these guys would eventually find me and
kill me. I couldnt realistically get away!
Court: um, no.
Dissent: Binnie To concede with not calling the police and concede with reasonable criminal
rather than reasonable person is threatening for the public order. This would endorse a
sociopathic approach to dispute resolution. A reasonable person would have seen other
options available.

to allow him to kill in this situation is basically contrary to public order; we cant
give this much space to criminal organizations.
83

This guy didnt feel he had another option


This mightve been reasonable
But were not giving this terrain to this behavior
This would give rise to a sociopathic view of dispute resolution
We dont want to give space to the world Cinous was coming from, so this
is straight up murder.
o Youre a member of a criminal organization, this = murder.
o
o
o
o
o

R v Lavallee 1990 SCC


Ratio: Sometimes the jury needs help (from experts) in understanding what is reasonable or
not in circumstances that are foreign to them. Broadens the scope of factors and better
contextualizes experiences of self defence.
Facts: 22 year old accused had a long history of abuse from the deceased. Killed him as he
was leaving the room after a fight and threat to get you. Trial judge allowed evidence of
battered spouse syndrome and acquitted by jury. CA orders a new trial on grounds that expert
evidence should not have been allowed.
Issues: Was expert evidence required for jury to know what was reasonable in that
situation?
Decision: Acquittal restored.
Reasons: Wilson Expert evidence is permissible to give the jury information that it wont
otherwise have bc of the specialised nature of this experience. Battered spouse syndrome may
not be something that can be appreciated without the assistance of expert evidence. Expert
evidence needed to show why a person in that situation may not see leaving or seeking help
as a reasonable alternative.
Such is the reaction of the average person confronted with the so-called battered wife
syndrome. We need help to understand it and help is available from trained professionals
(RBHS 956)
If it strains credulity to imagine what the ordinary man would do in the position of a
battered spouse, it is probably because men do not typically find themselves in that situation.
Some women do, however. The definition of what is reasonable must be adapted to
circumstances which are, by and large, foreign to the world inhabited by the hypothetical
reasonable man. (RBHS 957)
Precedent: Precedent: Subsequent cases focus on same thing enhancing understanding of
jurors about what is reasonable in that situation.
Concerns/Critques: Syndromization of womens experience; womens experiences must be
included in the objective standard of a reasonable person and have historically been ignored;
battered women who are not the s/t weak, frail, helpless woman not as likely to make use of
this defence; defence isnt very often successful anyway.
Isabel Grant:
It is not necessarily a battered womans helplessness that renders her killing reasonable; it
is the repetition and regularity of the abuse to which she has been subject and her perception
of the threat to her life or safety. By relying on a syndrome explained by a psychiatrist to
interpret the perceived threat we implicitly send the message that, without this evidence,
the womans perception of reality would not be trusted as reasonable. (RHBS 967)
LHeureux-Dub and McLachlin JJ. in Malott:
In Lavallee a majority of the Court accepted that the perspectives of women, which have
historically been ignored, must now equally inform the objective standard of the reasonable

84

person in relation to self-defence. (RBHS 971)


How should the courts combat the syndromization, as Professor Grant refers to it, of
battered women who act in self-defence? The legal inquiry into the moral culpability of a
woman who is claiming self-defence must focus on the reasonableness of her action in the
context of her personal experiences, and her experiences as a woman, not on her status as a
battered woman and her entitlement to claim that she is suffering from battered woman
syndrome. (RBHS 972)

DURESS
DURESS (s. 17 and CL)
Summary
Affirmative defence
AR and MR are there but shouldnt be punished.
Excuse
Results in acquittal

Burden of Proof
Is the accused
the principal
offender?

Elements of CL
of Duress

CL of duress applies to all situations EXCEPT where the accused


is a principal (Paquette), accused of one of the 22 excluded
offences under s. 17 (Ruzic).
Principals who commit any other offence are dealt with under the CL of
duress (most).
If you are a principal offender and accused of one of the excluded
offences, you cannot claim the defence of duress.
Jury should be entitled to decide whether accused is a principal or a
secondary offender. If conclude that secondary, then decide if person has
a defence of duress at common law. (Mena)
Crown prove elements of offence beyond a reasonable doubt once an air of
reality is established.
YES s. 17 Is it one of the 22 excluded offences? Yes No defence of
duress
No Still under s. 17 but same
as CL
defence of duress
(Paquette; Ruzic)
NO Common law defence of duress
Non principal offenders and principals outside of 22 offences go
through CL defence of duress
1. The accused must be subject to a threat of death or serious injury to
himself or to another person (and though it doesnt have to be
immediate, there must be a close temporal connection between the
threat and its execution (Ruzic))
2. The accused must be committing the offence exclusively as a product of
the threat (that is, must believe that the threat will be carried out if
s/he fails to commit the offence)
3. The threat must be of such gravity that it may well cause a reasonable
person placed in the same situation as the accused to respond to the
threat by committing the offence in question. (subjective/objective
ordinary person in same circumstances)
4. The accused must have a reasonable belief in no reasonable avenue

85

of escape (objective/subjective Ruzic; using the context of


Belgrade); and
5. The accused must not be a voluntary member of a criminal association
whereby he or she knew that s/he may be subject to compulsion by
threats.
The common law of duress (R v Ryan)
The elements of the common law defence of duress:
There must be an explicit or implicit threat of present or future death or
bodily harm. (The threat can be directed at the accused or a third party);
The accused must reasonably believe that the threat will be carried out;
There must be no safe avenue of escape. This element is evaluated on a
modified objective standard;
There must be a close temporal connection between the threat and the
harm threatened;
There must be proportionality between the harm threatened and the harm
inflicted by the accused. The harm caused by the accused must be equal to
or no greater than the harm threatened. This is also evaluated on a modified
objective standard;
The accused is not a party to a conspiracy or association whereby the
accused is subject to compulsion and actually knew that threats and coercion
to commit an offence were a possible result of this criminal activity,
conspiracy, or association.
s. 17 of CCC

A person who commits an offence under compulsion by threats of


immediate death or bodily harm from a person who is present when the
offence is committed is excused for committing the offence if the person
believes that the threats will be carried out and if the person is not a party to
a conspiracy or association whereby the person is subject to compulsion, but
this section does not apply where the offence that is committed is
Three Restrictions:
1. The threat has to be of immediate bodily harm (no force or effect, Ruzic)
2. The person posing the threat has to be present (no forece or effect,
Ruzic)
3. 22 offences are excluded, including murder, sexual assault, robbery and
arson. (excluded offences)
s. 17 applies to principals only (Paquette)

Consider making a constitutional argument that those offences. Likely if


someone is charged with one of those offences and has a good duress
claim, they will challenge that restriction.

R. v. Carker (SCC, 1967) BAD LAW


Ratio: s. 17 exhaustively defines the defence of duress. CL defence of duress does not apply.

86

Facts: Accused charged with offence and wanted to rely on duress but couldnt meet
requirements for immediacy and presence. Argues that he should be able to rely on either CL
or s. 17.
Decision: Guilty. No defence of duress available.
Reasons: Court rejects that and holds that s. 17 displaces the CL entirely.
Precedent: Carker has been largely marginalized and CL is now supreme.
Paquette v The Queen SCC 1976
Ratio: s. 17 is applicable to principal offenders only. Secondary offenders are dealt with under
CL where there are no excluded offences.
Facts: Robbery at pop shop in Ottawa and person killed. Paquette is driver of getaway car.
Claims duress bc forced to drive at gunpoint. Carker held that s. 17 is exhaustive and murder
is an excluded offence.
Issues: Is s. 17 applicable?
Reasons: Court reviews language of offence and say a person who commits is only
applicable to principals, not secondary offenders.
R v Mena
Ratio: Jury should be entitled to decide whether accused is a principal or a secondary
offender. If conclude that secondary, then decide if person has a defence of duress at common
law.
R. v. Ruzic (SCC, 2001)
Ratio: Immediacy and presence requirements of s. 17 are unconstitutional bc they allow the
conviction of the morally involuntary thereby breaching the PFJ. Section 7 requires defence of
duress to be available to an accused even when they were not under immediate threat and or
threat is to a third party.
Facts: Marianna Ruzic stalked and harassed by man in Belgrade who threatens to harm her
mother if she doesnt fly heroin to Toronto. She does and is charged. She pleads duress.
Problem: imminence and presence. It was a future threat and the person wasnt present during
the commission. She is under s. 17 bc she is a principal.
Issues: Do the imminence and presence requirements of s. 17 violate s. 7 bc allowing a
morally involuntary person to be convicted is contrary to the PFJ (BC Motor Vehicle).
Decision: Acquitted
Reasons: LeBel - A person acts in a morally involuntary fashion when, faced with perilous
circumstances, she is deprived of a realistic choice whether to break the law. (RBHS 919) Not
morally blameless here bc committed offence. New PFJ that you may not be punished if you
were morally involuntary. Immediacy and presence requirements of s. 17 raise the possibility
of convicting the morally involuntary t/f unconstl.
Once the elements of the offence have been established, the accused can no longer be
considered blameless. This Court has never taken the concept of blamelessness any further
than this initial finding of guilt. (RBHS 920)
Precedent: State of the law after is that the 22 excluded offences still remain along with the
CL defence of duress. Prof: likely if someone is charged with one of those offences and has a
good duress claim, they will challenge that restriction.
Hibbert v The Queen
Ratio: Confirms that duress does not negate mens rea but rather is a stand alone defence (an
excuse)
R v Ryan, 2013
Ratio:

87

The nature of duress


in self-defence, the victim simply attacks or threatens the accused; the motive for the attack
or threats is irrelevant. In duress, on the other hand, the purpose of the threat is to compel
the accused to commit an offence. To put it simply, self-defence is an attempt to stop the
victims threats or assaults by meeting force with force; duress is succumbing to the threats by
committing an offence. (20)
Brief

Note on Party Liability


A person can be guilty of an offence in one of four ways. He or she
actually commits it;
Aids someone else who actually commits it;
Abets (encourages) someone else to actually commit it;
Counsels, procures, or incites someone else to actually commit it.

NECESSITY
necessitas legem non habet necessity knows no
law
NECESSITY
Summary

Burden of Proof
Legal
Requirements
for Necessity
(After Perka)

Affirmative defence (AR and MR are present)


Classic excuse defence (although Wilson J in Perka says should be
available for justification
Results in acquittal
Crown prove elements of offence beyond a reasonable doubt once an air of
reality is established.
1. The situation must be urgent and the peril must be imminent
Modified objective:
o The accused person must, at the time of the act, honestly believe
(subjective), on reasonable grounds (objective), that he faces a
situation of imminent peril that leaves no reasonable alternative
open.

Such that normal human instincts cry out for action and make a counsel of patience
unreasonable.

Foreseeability is not enough it must be on the verge of transpiring and virtually certain
to occur. (Latimer)

2. Compliance with the law must be demonstrably impossible.


That is, there must be no legal way out. No reasonable legal alternative.
(Latimer)
also modified objective assessment
3. There must be proportionality between the harm inflicted and the
harm avoided.
Objective
o Does the community view the harm avoided as at least comparable
to the harm inflicted? only objective

88

The two harms must, at minimum, be of comparable gravity. (Latimer)

Exception

No rational criminal justice system, no matter how humane or liberal, could excuse the
infliction of a greater harm to allow the actor to avert a lesser evil. In such circumstances
we expect the individual to bear the harm and refrain from acting illegally. If he cannot
control himself we will not excuse him. (RBHS 886)

Fact that youre doing something illegal at the time does not disentitle
you from raising defence, unless you created the circumstances of
necessity. (E.g competing keg parties; fasting man who did B&E)

R v Dudley and Stephens


Ratio: necessitas legem non habet necessity knows no law
Facts: Dudley is captain. Stephens is crew member. Cabin boy. Boat sinks in Atlantic and get
off into dingy boat. Not for or water. Weeks pass and crew concludes that one must die to feed
others. Three crew members all have families so they select cabin boy. They kill him and eat
him. Four days later they are rescued. News of what happens spreads and they are arrested
and tried for murder.
Issues: Should they be punished as severely.
Decision: Jury finds them guilty but not sentenced to death. They were given them a
conditional pardon.
Reasons: Had no choice but to act.
R v Morgentaler 1975
Ratio: Put defence on the books in Canada but no test was developed to show what it looked
like.
Facts: M argued he had to perform the abortion bc woman might do something foolish if he
didnt. It was a necessity
Issues: Is the defence of necessity available.
Decision: Necessity not available bc no evidence of this.
Reasons: Dickson J. (as he then was): If the defence does exist, it can go no further than to
justify non-compliance in urgent situations of clear and imminent peril when compliance with
the law is demonstrably impossible. (RBHS 881). Necessity is likely a part of law but not
formally recognized. No other choice but to act.
Precedent: After M, lower courts treated Dicksons statement as a recognition of the defence
of necessity.
Perka v The Queen
Ratio: The Defence Recognized. Explicitly references it and gives criteria but classifies it as an
excuse alone.
Facts: Perka had been sailing boat in international waters. Boat had 33 tons of weed (~67million). Massive engine trouble and storm put him and everyone else in imminent peril.
Redirected ship and landed on Vancouver island. Charged with possession for trafficking. He
says storm was only reason came to Canada and therefore the reason he broke Canadian law
was bc he was at risk of death bc of storm committed offence in your jurisdiction bc of
necessity. Trial acquitted him but then new trial ordered.
Issues: When is the defence of necessity available?
Decision: He was acquitted on procedural issues. Never actually dealt with necessity wrt to
his acquittal
Reasons: Dickson C.J. distinguishes between justifications and excuses. Justifications:
actions that are often praised, as motivated by some great or noble object and therefore
punishment often seems incompatible with the social approval bestowed on the doer. (RBHS
885). Excuses are still viewed as wrongful acts but done in circumstances or for reasons that
the law appreciates and therefore doesnt support punishment. No vindication of the deed (ie.

89

didnt do right thing) but recognize that there was a frailty in human nature that led you to
break the law. Majority recognizes defence of necessity but classifies it only as an EXCUSE The defence of necessity rests on a realistic assessment of human weakness, recognizing that
a liberal and humane criminal law cannot hold people to the strict obedience of laws in
emergency situations where normal human instincts, whether of self preservation or of
altruism, overwhelmingly impel disobedience. (RBHS 885-886)
Dissent: Wilson J Necessity defence should sometimes be available as a justification. (e.g.
parent has to trespass to protect a child who is in danger) Conflicting legal duties exist, but
youre not a weak person for taking care of your child.
Precedent: First Canadian case that recognized existence of CL of necessity. Case wasnt
disposed of on necessity.
Latimer v The Queen
Ratio: Refines the criteria from Perka
Facts: Tracey Latimer suffered from severe form of cerebral palsy, subject to frequent
seizures. Anti-seizure meds prevented her from taking pain killers and she was also subject to
surgeries. Facing a surgery to remove her upper thigh bone. Latimer concludes that rather
than face final surgery, he should end her life. Puts her into cab of truck and kills her. Tells
police and charged with first degree murder. At first trial, convicted of second degree murder
by jury. New trial ordered bc Crown tampered with jury pool. Second trial tried for second
degree murder. Defence argued necessity. Trial judge decided not to put defence to jury bc
said no air of reality. Jury asks if they convict him, if they can have input into sentencing. Judge
tells them not to concern themselves with punishment. But may be some input into
sentencing. If parole ineligibility should exceed 10 years. So jury comes back and finds him
guilty and then judge says min is life in prison. Jury upset and says parole eligibility should be
1 year, even though min is 10-25. Judge uses Charter to do a constitutional exemption (no
longer exists). SCC concludes there is no exemption to mandatory minimum.
Decision: Guilty. Defence of necessary is not available.
Reasons: No proportionality infliction of death for ease of pain.
Precedent: Some indications that murder may never be met under defence of necessity.
R v Kerr
Facts: Kerr is in prison and told he is going to be attacked by other prisoner. Defends himself
with a knife that he had. Charged with possession of weapon from dangerous purpose. Argues
that in prison env where Corrections Canada fails to protect prisoners from violence that he
had to fashion a weapon. Argument about how necessity defence relates to marginalisation
and ability of state to protect.

90

You might also like